Download Dermatovrnrrology

Document related concepts

Disease wikipedia , lookup

Eradication of infectious diseases wikipedia , lookup

Transmission (medicine) wikipedia , lookup

Pandemic wikipedia , lookup

Compartmental models in epidemiology wikipedia , lookup

Public health genomics wikipedia , lookup

Marburg virus disease wikipedia , lookup

Syndemic wikipedia , lookup

Canine parvovirus wikipedia , lookup

Focal infection theory wikipedia , lookup

Infection wikipedia , lookup

Epidemiology of syphilis wikipedia , lookup

Infection control wikipedia , lookup

Transcript
Module 1. Dermatovenerology
Tests
1. Stratum basale is responsible for:
A. Resiliency
B. Skin color
C. Elasticity
D. *Mitotic activity and proliferation
E. All are correct
2. Petechiae is:
A. Elevated solid primary skin lesion
B. Secondary skin lesion
C. *Equal to plane of the skin
D. Above the plane of the skin
E. All are wrong
3. Wheal is characterized by all, except:
A. Sharply circumscribed, raised lesion (dermal edema)
B. Usually erythematous
C. Variable shape and size
D. Transient
E. *Presence of chancre
4. The best method diagnosing of scabies is?
A. Wasserman reaction
B. Tzank smear
C. Dark-field microscopy
D. *Direct visualization
E. Diascopy
5. The most superficial layer of the epidermis is:
A. *Stratum corneum
B. Stratum lucidum
C. Stratum basale
D. Stratum granulosum
E. Stratum spinosum
6. A circumscribed elevation of skin containing purulent fluid is:
A. Vesicle
B. Bullae
C. Tumor
D. Nodule
E. *Pustule
7. Secondary skin lesions are all, except:
A. Fissure
B. Erosion
C. Crusts
D. Ulcer
E. *Bulla
8. Excoriation can result in:
A. Both are wrong
B. *Ulcers
C. Vesicle
D. Papules
E. Patch
9. Pediculosis corporis is caused by:
A. Pubic lice
B. *Body louse
C. Head louse
D. Scabies mite
E. All are correct
10. Crusting is:
A. A collection of dried serum
B. A collection of cellular debris
C. Yellow in colour
D. Brown in colour
E. *All are correct
11. Pediculosis capitis is caused by:
A. Pubic lice
B. Body louse
C. *Head louse
D. Scabies mite
E. Both are correct
12. Burrow is characterized for:
A. Psoriasis
B. Lichen planus
C. Pediculosis
D. *Scabies
E. All are correct
13. Papules are characterized for:
A. Pituriasis versicolor
B. Vitiligo
C. *Lichen planus
D. Urticaria
E. Impetigo
14. Langerhans cell are responsible for:
A. Resiliency
B. Skin color
C. Elasticity
D. Water maintain
E. *Antigen presenting cell in skin
15. Nodules are characterized for all diseases except:
A. Warts
B. Lipoma
C. Erythema nodosum
D. Furnuncle
E. *Pituriasis versicolor
16. Primary skin lesions without cavity are all, except:
A. Macule
B. Papule
C. Patch
D. Nodule
E. *Bulla
17. Mitotic stem cells are characterized for:
A. Stratum corneum
B. *Stratum basale
C. Sratum spinosum
D. Srratum lucidum
E. Stratum granulosum
18. Primary skin lesions are all, except:
A. Macule
B. Papule
C. Nodule
D. *Fissure
E. Urticaria
19. The intercellular connections (desmosomes) are characterized for:
A. Stratum basale
B. Srratum lucidum
C. *ratum spinosum
D. Stratum granulosum
E. Stratum corneum
20. Erosion is vertical loss of:
A. Subcutaneous fat
B. Dermis
C. *Epidermis
D. Dermis and subcutaneous fat
E. Epidermis and subcutaneous fat
21. White striaform symmetric lesions in the buccal mucosa and flat papules on the skin are
characterized for:
A. Primary syphilis
B. Herpes zoster
C. Pemphigus vulgaris
D. *Lichen planus
E. Herpes simplex
22. Dermis is composed of:
A. Fibrous tissue
B. Epithelial tissues
C. *Connective tissue
D. Granular tissue
E. Interstitial tissue
23. Primary skin lesions without cavity are all, except:
A. Macule
B. Papule
C. Patch
D. Nodule
E. *Vesicle
24. Definitive diagnosis of scabies is made by direct visualization of the:
A. Mite
B. Eggs
C. *Mite, eggs
D. Free fluid
E. All are correct
25. Epidermis is composed of:
A. *Epithelial tissues
B. Connective tissue
C. Fibrous tissue
D. Granular tissue
E. Interstitial tissue
26. What layer of the epidermis is characterized by mitotic stem cells?
A. Stratum corneum
B. *Stratum basale
C. Sratum spinosum
D. Srratum lucidum
E. Stratum granulosum
27. Secondary skin lesions are all, except:
A. Fissure
B. Erosion
C. *Nodule
D. Crusts
E. Ulcer
28. Tumor is:
A. Large patch
B. Large fissure
C. *Large nodule
D. Large plaque
E. Large burrow
29. Stratum corneum is formed by:
A. Flattened cells
B. *Dead cells
C. Mitotic stem cells
D. Mercel cells
30. Melanocytes are responsible for:
A. Resiliency
B. Elasticity
C. *Skin color
D. Water maintain
E. All are correct
31. Serpentine burrow localized in:
A. Stratum granulosum
B. Sratum spinosum
C. Srratum lucidum
D. *Stratum corneum
E. Stratum basale
32. Excoriations are characterized for all disease, except:
A. *Pituriasis versicolor
B. Scabies
C. Atopic dermatitis
D. Eczema
E. Allergic contact dermatitis
33. Papulosquamous means presence of such skin lesions:
A. Vesicle
B. Patch
C. Macule
D. *Scales
E. Fissure
34. Pustule is characterized for all disease, except:
A. Impetigo
B. Herpes simplex
C. Acne
D. *Pituriasis versicolor
E. Herpes zoster
35. Rosacea triggering factors means all, except:
A. Alcohol
B. Hot baths or shower
C. Spicy foods
D. *Trauma
E. Humidity
36. Primary skin lesions are all, except:
A. *Erosion
B. Macule
C. Papule
D. Nodule
E. Urticaria
37. Skin appendages are all, except:
A. Sebaceous glands
B. Hair
C. *Subcutis
D. Nails
E. Eccrine sweat glands
38. Acne Conglobata is characterized by such skin lesions:
A. Comedones
B. Pustules
C. Papules
D. *Nodules
E. Vesicles
39. Secondary skin lesions are all, except:
A. *Patch
B. Fissure
C. Erosion
D. Crusts
E. Ulcer
40. Dermatological status of patient include all, except:
A. Inspection
B. Palpation
C. Scraping
D. Diascopy
E. *Tzank smear
41. Dermis is composed of:
A. *Connective tissue
B. Fibrous tissue
C. Epithelial tissues
D. Granular tissue
E. Interstitial tissue
42. The most common bacteria associated with acne is:
A. Corynebacterium
B. Pseudomonas
C. Fusobacterium
D. Acinetobacter
E. *Propionibacterium
43. Skin vessels form:
A. Superficial net
B. *Superficial and deep net
C. Deep net
D. Deep and medium
E. Medium net
44. Nodules are characterized for all diseases, except:
A. Neurofibromatosis
B. Erythema nodosum
C. Furnuncle
D. *Pamphigus vulgaris
E. Hemangioma
45. Vesicles are characterized for all diseases, except:
A. Herpes zoster
B. Herpes simplex
C. Eczema
D. Chicken pox
E. *Erythrasma
46. Tumor means:
A. Large fissure
B. *Large nodule
C. Large patch
D. Large plaque
E. Large burrow
47. Papular minivesicular dermatitis means all, except:
A. Contact dermatitis
B. *Pemphigus vulgaris
C. Atopic dermatitis
D. Scabies
E. Dermatophytosis
48. Macula or patch are characterized for all diseases, except:
A. *Lichen planus
B. Pituriasis versicolor
C. Vitiligo
D. Lentigo
E. Xanthelasma
49. Phthiriasis mens:
A. Scabies
B. *Pediculosis
C. Ascarides
D. Giardiasis
E. Demodicidosis
50. Pediculosis Vestimenti means:
A. *Pediculosis Corporis
B. Pediculosis Capitis
C. Pediculosis Pubis
D. All are correct
E. All are wrong
51. Such layer of the epidermis consists of a single layer of keratinocytes:
A. Stratum corneum
B. Stratum spinosum
C. Stratum lucidum
D. *Stratum basale
E. Stratum granulosum
52. Which of the following methods should be undertaken for scabies diagnosis?
A. Prick test
B. “Burrow-ink test”
C. Tzanck test
D. Balzer test
E. Wood’s light
53. The color of normal skin depends primarily on pigments such as:
A. Melanin
B. Deoxyhemoglobin
C. Carotene
D. Oxyhemoglobin
E. *All are correct
54. Ulcer area of skin loss:
A. Extending into the dermis
B. *All are correct
C. Heals with a scar
D. Associated surface exudates
E. Associated surface crust
55. Connective tissue is present in:
A. Stratum spinosum
B. *Dermis
C. Stratum corneum
D. Stratum lucidum
E. Stratum basale
56. Plaque is:
A. *Above the plane of the skin
B. Associated surface crust
C. Secondary skin lesion
D. Equal to plane of the skin
E. Heals with a scar
57. Acne patients should look for properties in cosmetics with which of the following:
A. Oil-free
B. Hypoallergenic
C. Nonirritating
D. Non-comedogenic
E. *All are correct
58. Which of the followings includes the function of skin?
A. Barrier to infection
B. Sensory receptor
C. Regulate temperature
D. Absorption
E. *All are correct
59. Rosacea symptoms can be aggravated by:
A. Wine and caffeine
B. Exercise
C. Topical steroid
D. Anxiety
E. *All are correct
60. Excoriation can result in:
A. *Erosions
B. Papules
C. Nodules
D. Scales
E. All are correct
61. Which one of the following is self - resolving condition?
A. *Pityriasis Rosea
B. Rosacea
C. Psoriasis
D. Seborrheic Dermatitis
E. Hyperkeratosis
62. Basement membrane selectively filters molecules moving between:
A. *Epidermis and dermis
B. Stratum corneum and stratum lucidum
C. Stratum basale and stratum granulosum
D. Stratum granulosum
E. Stratum spinosum
63. Erosion: a wider tear in the:
A. *Epidermis
B. Dermis
C. Subcutis
D. All are correct
E. All are wrong
64. Patient has well defined 2-Cm localized hair loss in the posterior scalp. This condition
is most likely:
A. Telogium effluvium
B. Anagen effluvium
C. Traction alopecia
D. Traumatic alopecia
E. *Alopecia areata
65. Rosacea triggering factors means all, except:
A. Sun / stress
B. Hot weather
C. *B-blocker
D. Cold wind
E. Exercise
66. The deepest layer of the epidermis is:
A. Stratum corneum
B. Stratum lucidum
C. *Stratum basale
D. Stratum granulosum
E. Stratum spinosum
67. Eccrine glands and deep hair follicles extend to:
A. Stratum lucidum
B. Dermis
C. Stratum basale
D. *Subcutaneous tissue
E. Stratum granulosum
68. Apocrine sweat glands are:
A. Large, deep glands in dermal layer
B. Mostly in groin
C. Mostly in axillae
D. *All are correct
E. All are wrong
69. Scaling - excess dead epidermal cells caused by:
A. Stress
B. *Abnormal keratinisation
C. Overcooling
D. Scrubbing
E. Skin infection
70. Acne Vulgaris is a disease of:
A. Sweat glands
B. *Pilosebaceous follicles
C. Epidermis and sweat glands
D. Dermis and sweat glands
E. All are correct
71. Hidradenitis Suppurativa must be differential with:
A. Cellulitis
B. Pilonidal cysts
C. Bacterial folliculitis
D. *All are correct
E. All are wrong
72. Folliculitis must be differential with all disease, except:
A. Insect bites
B. Scabies
C. *Pituriasis versicolor
D. Rosacea
E. Tinea
73. Telangiectasia is:
A. Associated surface crust
B. Secondary skin lesion
C. *Equal to plane of the skin
D. Above the plane of the skin
E. Heals with a scar
74. Which secondary skin changes are characterized for psoriasis?
A. Cicatricial atrophy
B. *Secondary hyper or hypo pigmentation
C. Secondary atrophy
D. Keloid
E. Atrophy of the hair follicles
75. Morphological variant of psoriasis are all, except:
A. Total psoriasis
B. *Scalp psoriasis
C. Diffuse psoriasis
D. Guttate psoriasis
E. Erythrodermia
76. Pyoderma is a group name for:
A. *Pyococcal dermatoses
B. Allergic dermatoses
C. Contact dermatitis
D. Viral dermatitis
E. All are wrong
77. Phenomenon of “stearic spot” at psoriasis is conditioned by:
A. Acantholysis
B. Spongiosis
C. *Parakeratosis
D. Papillomatosis
E. Disceratosis
78. Variations of psoriasis are all, except:
A. *Secondary psoriasis
B. Scalp psoriasis
C. Generalized psoriasis
D. Guttate psoriasis
E. Palmoplantar psoriasis
79. Skin lesions in case of ecthyma localised in all places, except:
A. Buttocks
B. Thighs
C. Legs
D. *Oral cavity
E. All are correct
80. What symptom is characterized for diagnosis of “psoriatic nail dystrophy”?
A. “Nail pitting”
B. “Oil drop”
C. *Onycholysis
D. Discoloration of the nail
E. Acantholysis
81. Prescription of antibiotics is arbitrary in case of such diagnose.
A. Intertrigenous psoriasis
B. Palmoplantar psoriasis
C. Eruthrodermic psoriasis
D. Scalp psoriasis
E. *Guttate psoriasis
82. Koebner’s phenomenon at psoriasis is:
A. Second appearance of the skin lesions at the same place
B. Appearance of the skin lesions in the open areas of the skin
C. *Appearance of the skin lesions at sites of trauma
D. Appearance of the skin lesions after using of some medicines
E. Appearance of the skin lesions after stress
83. “Oil drop” symptom of the nails is characterized for diagnosis of:
A. Epidermolysis bullosa
B. Lichen planus
C. Onychomycosis
D. *Psoriasis
E. Chronic urticaria
84. Psoriatic phenomenons is:
A. “Christmas tree”
B. “Venera necklace”
C. “Biet collar”
D. *“Terminal film”
E. “Moth eaten”
85. Clinical features of psoriasis in children are:
A. Affection of the palms and soles
B. Frequent affection of the joints
C. *Atypical localization of the skin lesions
D. Absence of itching
E. Eruthrodermic psoriasis
86. Guttate form of psoriasis is often connected with:
A. Stress
B. Bad nourishment
C. Use of medicines
D. *Following group A beta-hemolytic strep pharyngitis
E. Presence of parasites
87. Ecthyma is:
A. Fungal infection
B. Viral infection
C. *Pyoderma
D. Autoimmune disease
E. All are correct
88. Diagnosis of a psoriatic plaque is based on:
A. Diascopy
B. *Grating with a glass slide
C. Skin prick test
D. Dermographism test
E. Tzanck test
89. Lichen planus is characterized by all “P” words, except:
A. Planar
B. Purple
C. Polished
D. *Patch
E. Polygonal
90. “Wickham striae” are characterized for:
A. Psoriasis
B. Atopic dermatitis
C. Scabies
D. *Lichen planus
E. Dermatophytosis
91. Sites of predilection of chronic plaque psoriasis are all, except:
A. Elbows
B. Knees
C. *Mucous membranes
D. Scalp
E. Hands and feet
92. Inverse psoriasis involves:
A. Nails
B. Bones
C. Scalp
D. *Groin
E. All are correct
93. Papules are characterized for:
A. Pituriasis versicolor
B. Vitiligo
C. *Lichen planus
D. Urticaria
E. Impetigo
94. White striaform symmetric lesions in the buccal mucosa and polygonal papules are
characterized for:
A. Primary syphilis
B. Pemphigus vegetans
C. *Lichen planus
D. Pemphigus vulgaris
E. Herpes simplex
95. Hyperpigmentation in psoriasis-result of regress of:
A. *Plaques
B. Vesicle
C. Pustules
D. Bulla
E. All are correct
96. Objective investigation of patients with psoriasis includes all methods, except:
A. Inspection of skin lesions
B. Palpation
C. General patient state
D. *Bulzer test
E. Scrubbing
97. Carbuncle is conglomerate of:
A. Papules
B. *Boils
C. Macula’s
D. Erosions
E. Crusts
98. Localization of furuncle is:
A. Neck
B. Face (chin, upper lip)
C. *All are correct
D. Buttocks
E. Perinea region
99. White striaform symmetric lesions in the buccal mucosa and pruritic papules are
characterized for:
A. *Lichen planus
B. Primary syphilis
C. Pemphigus vulgaris
D. Herpes zoster
E. Herpes simplex
100.
Papulosquamous means presence of such lesions:
A. *Scales
B. Vesicle
C. Patch
D. Macula
E. Fissure
101.
Guttate psoriasis must be diffdiagnosed with:
A. *Secondary syphilis
B. Furuncle
C. Pituriasis versicolor
D. Atopic dermatitis
E. Scabies
102.
Which of the following skin conditions is commonly known as a boil?
A. Ecthyma
B. *Furuncle
C. Psoriasis
D. Impetigo
E. All are correct
103.
Psoriatic nail changes are all, except:
A. Onycholysis
B. “Oil drops”
C. Pitting
D. *“Salmon patches”
E. Subungual hyperkeratosis
104.
Deep pyoderma means all, except:
A.
B.
C.
D.
E.
105.
A.
B.
C.
D.
E.
106.
A.
B.
C.
D.
E.
107.
A.
B.
C.
D.
E.
108.
A.
B.
C.
D.
E.
109.
A.
B.
C.
D.
E.
110.
A.
B.
C.
D.
E.
111.
A.
B.
C.
D.
E.
112.
A.
B.
C.
D.
*Impetigo
Folliculitis of leg
Furuncle
Carbuncle
Sycosis Barbae
Drugs that can exacerbate psoriasis are all, except:
Beta blockers
Lithium
*Corticosteroids
IFN-alpha
Antimalarials
Variants of lichen planus are all, except
Plaque
Atrophic
Erosive
Affection of buccal mucosa
*Patch
Classification of pyoderma according to etiology:
Staphyloderma
*All are correct
Streptoderma
Streptostaphyloderma
Atipical forms
Morphological variant of psoriasis are all, except:
Total psoriasis
Diffuse psoriasis
*Psoriatic arthritis
Guttate psoriasis
Erythrodermia
Papulosquamous means presence of such lesions:
Fissures
Vesicles
*Papules
Patches
Maculae’s
Wickhams striae are characterized for:
Psoriasis
Contact dermatitis
*Lichen planus
Scabies
Dermatophytosis
Involved of oral, other mucous membranes and nails are characterized for:
Pemphigus vulgaris
Pyoderma
Herpes zoster
*Lichen planus
Pituriasis versicolor
Psoriatic phenomenons is:
“Christmas tree”
*“Stearic spot
“Venera necklace”
“Biet collar”
113.
114.
115.
116.
117.
118.
119.
120.
121.
E. “Moth eaten”
Psoriatic phenomenons is:
A. *“Bleeding points”
B. “Christmas tree”
C. “Venera necklace”
D. “Biet collar”
E. “Moth eaten”
Pruritic, purple, polygonal papules are characterized for:
A. Contact dermatitis
B. Primary syphilis
C. Scabies
D. *Lichen planus
E. Psoriasis
Location variants of psoriasis are all, except:
A. Inverse psoriasis
B. Nail psoriasis
C. Psoriatic arthritis
D. Scalp psoriasis
E. *Nummular psoriasis
What kind of infection ecthyma is?
A. Fungal
B. Viral
C. *Bacterial
D. Parasitogenic
E. All are wrong
“Seborrheic area” is:
A. Nails
B. *Central chest
C. Vagina
D. Oral cavity
E. All are correct
Predisposing factors of pyoderma are:
A. Diabetes mellitus
B. Chronic infections
C. *All are correct
D. Diminished of immunity
E. Hypovitaminosis
Causative agent of furuncle is:
A. Treponema pallidum
B. Scabies mite
C. Herpes virus infection
D. *Staphylococcus aureus
E. All are correct
Erythrasma is caused by:
A. Mite
B. *Corynebacterium minutissimum
C. Staphylococcal aureus
D. Streptococcal agent
E. All are correct
Skin lesions of erythrasma are:
A. Papules
B. *Macula’s
122.
123.
124.
125.
126.
127.
128.
129.
130.
C. Erosions
D. Crusts
E. Pustules
What type of infection eythrasma is?
A. Fungal
B. Viral
C. All are correct
D. *Bacterial
E. All are wrong
Morphological variant of psoriasis are all, except:
A. Total psoriasis
B. Diffuse psoriasis
C. Guttate psoriasis
D. Erythrodermia
E. *Nail psoriasis
Location variants of psoriasis are all, except:
A. Inverse psoriasis
B. Nail psoriasis
C. Psoriatic arthritis
D. *Diffuse psoriasis
E. Scalp psoriasis
Inverse psoriasis involves:
A. Nails
B. *Axillae
C. Scalp
D. Bones
E. All are correct
Non-bullous impetigo is a superficial skin infection that manifests as:
A. Nodules
B. Ulcers
C. *Honey-colored crust
D. Clusters of patches
E. Clusters of maculae’s
"Seborrheic area” is
A. Nails
B. *Scalp
C. Vagina
D. Oral cavity
E. All are correct
Lichen planus is characterized by all “P” words, except:
A. *Pustule
B. Planar
C. Purple
D. Polished
E. Polygonal
Clinical improvement of lichen planus include all, except:
A. *Presence of pustules
B. Presence of papules
C. Presence of “Wickham’s striae”
D. Presence of “Koebner’s phenomenon”
E. Affection of nails
“Woronow ring” is characterized for:
A.
B.
C.
D.
E.
131.
A.
B.
C.
D.
E.
132.
A.
B.
C.
D.
E.
133.
A.
B.
C.
D.
E.
134.
A.
B.
C.
D.
E.
135.
A.
B.
C.
D.
E.
136.
A.
B.
C.
D.
E.
137.
A.
B.
C.
D.
E.
138.
A.
B.
C.
D.
Licnen planus
Pyoderma
*Psoriasis
Scabies
Syphilis
Clinical types of impetigo are all, except:
Impetiginous perleche (angular stomatitis)
Annular impetigo
Acute diffuse impetigo
Bullous impetigo
*“Wickham’s striae”
Hypopigmentation in psoriasis-result of regress of:
Erosion
Pustules
Bulla
*Papules
All are correct
Psoriasis of the nails are characterized by all, except:
Nail pitting
“Oil drops”
Onycholysis
All are correct
*Presence of pus
Predisposing factors of pyoderma are all, except:
Diabetes mellitus
*Stress
Chrohic infections
Diminished of immunity
Hypovitaminosis
The main characteristic skin lesion in case of pyoderma is:
*Pustule
Papule
Erosion
Scale
Maculae
Pyoderma means:
Viral infection of the skin
Tinea infection of the skin
All are correct
*Bacterial infection of the skin
All are wrong
Deep pyoderma means all, except:
Folliculitis of leg
Furuncle
Carbuncle
Sycosis Barbae
*Impetigo
Carbuncle means:
*Large deep ulcer
Large deep fissure
Large deep papule
Large deep pustule
E. Large deep erosion
139.
Impetigo is:
A. Very superficial viral infection of the skin
B. *Very superficial bacterial infection of the skin
C. Very superficial fungal infection of the skin
D. Very superficial parasitic infection of the skin
E. Deep bacterial infection of the skin
140.
Deeper skin infection with staphylococcus or streptococcus bacteria are all,
except:
A. Boil
B. Furuncle
C. Abscess
D. Carbuncle
E. *Impetigo
141.
Carbuncle is?
A. Viral infection
B. Fungal infection
C. *Bacterial infection
D. Parasitogenic infection
E. All are wrong
142.
Koebner’s phehomenon is typical for:
A. Allergic dermatitis
B. Impetigo
C. *Psoriasis
D. Furuncle
E. Ecthyma
143.
Location variants of psoriasis are all, except:
A. *Erythrodermia
B. Inverse psoriasis
C. Nail psoriasis
D. Psoriatic arthritis
E. Scalp psoriasis
144.
“Wickham’s striae” are white network on the surface of:
A. Crust
B. Bulla
C. Vesicle
D. *Papule
E. All are correct
145.
Which of the following skin conditions is commonly known as a boil?
A. Ecthyma
B. *Furuncle
C. Abscess
D. Impetigo
E. All are correct
146.
Folliculates means:
A. Deep bacterial inflammation of hair follicles
B. Deep viral inflammation of hair follicles
C. Deep parasitic inflammation of hair follicles
D. *Superficial bacterial inflammation of hair follicles
E. Superficial viral inflammation of hair follicles
147.
Folliculitis means:
A. Superficial infection involving only the whole follicle
B.
C.
D.
E.
148.
A.
B.
C.
D.
E.
149.
A.
B.
C.
D.
E.
150.
A.
B.
C.
D.
E.
151.
A.
B.
C.
D.
E.
152.
A.
B.
C.
D.
E.
153.
A.
B.
C.
D.
E.
154.
A.
B.
C.
D.
E.
155.
A.
B.
C.
D.
E.
*Superficial infection involving only the upper portion of the follicle
Deep infection involving only the whole follicle
Deep infection involving only the upper portion of the follicle
Superficial infection involving only the upper and media portion of the follicle
Psoriasis is characterised by:
Not- demarcated erythematous plaques with silvery scale
Well-demarcated white maculae’s with silvery scale
*Well-demarcated erythematous plaques with silvery scale
Well-demarcated erythematous nodules with silvery scale
Well-demarcated erythematous plaques with red scale
A carbuncle is an infection involving:
*Subcutaneous tissue around several hair follicles
Subcutaneous tissue around one hair follicle
Subcutaneous tissue around sweat gland
Subcutaneous tissue around only sebaceous gland
All are correct
Pustular psoriasis means psoriasis of:
Scalp
Beard zone
Groin
*Soles, palms
Nails
Arrangement of lesions in case of pyoderma means all, except:
Annular
*Chard chancre
Circinate
Linear
Reticulate
Impetigo means:
*Skin infection caused by bacteria
Skin infection caused by HSV
Skin infection caused by CMV
Skin infection caused by EBV
Skin infection caused by mite
Furuncles are skin abscesses caused by staphylococcal infection, which involve:
Stratum spinosum
*A hair follicle and surrounding tissue
Sweat gland and surrounding tissue
Sebaceous gland and surrounding tissue
Stratum corneum
Furuncles (boils) are:
Tender maculae’s or pustules caused by staphylococcal infection
Tender nodules or patch caused by staphylococcal infection
Tender maculae’s or plagues caused by staphylococcal infection
*Tender nodules or pustules caused by staphylococcal infection
Tender nodules or erosions caused by staphylococcal infection
Carbuncles are clusters of:
Maculae’s that are subcutaneously connected
Patches that are subcutaneously connected
*Furuncles that are subcutaneously connected
Vesicles that are subcutaneously connected
Pustules that are subcutaneously connected
156.
Hidradenitis suppurativa means a chronic, scarring inflammation of apocrine
glands of all, except:
A. Of the axillae
B. Of the groin
C. Around the nipples
D. *Of the scalp
E. Around the anus
157.
Non-bullous impetigo is a superficial skin infection that manifests as:
A. *Clusters of pustules
B. Clusters of patches
C. Clusters of maculae’s
D. Scarring
E. Ulcers
158.
Lab. diagnosis of bacterial infection of skin means all, except:
A. Skin biopsy
B. Skin swab
C. Pus swab
D. Nasal / skin swab
E. *Tzanck test
159.
Dermatophytosis- is:
A. Virus infection of the skin
B. Bacteria infection of the skin
C. *Fungal infection of the skin
D. All are correct
E. Parasitogenic disease of the skin
160.
Therapy of dermatophytes include:
A. *Griseofulvin
B. Steroids
C. Cyclosporine
D. Tricyclic antidepressants
E. Leukotriene blockers
161.
Typical skin lesion of tinea is :
A. *Annular or arcuate plaque
B. Ulcer
C. Crust
D. Burrow
E. All are correct
162.
Dermatophytes - keratinophilic fungi, which live in:
A. Derma
B. Vessels
C. Nerves
D. *Superficial dead keratin
E. All are correct
163.
Favus caused by:
A. T.mentagrophytes
B. T.rubrum
C. M.canis
D. *All are wrong
E. All are correct
164.
Trichophyton affect:
A. Only skin
B. Skin, hair
165.
166.
167.
168.
169.
170.
171.
172.
C. Nails, hair
D. Skin, nails
E. *Skin, hair, nails
Skin lesions of pityriasis versicolor are:
A. *Macula's
B. Nodules
C. Fissures
D. Erosions
E. Bullas
ANSWER A
Microsporum is type of dermatophytosis, which affect:
A. Only hair
B. Nails
C. Only skin
D. *Skin, hair
E. Only nails
Wood’s lamp examination - method of diagnosis of:
A. Syphilis
B. Virus infection
C. Scabies
D. Gonococcal infection
E. *Tinea infection
Tinea Versicolor treatment include:
A. *Itraconazole
B. H1 anti-histamines
C. Leukotriene blockers
D. H2 anti-histamines
E. Steroids
Wood’s lamp examination- method of diagnosis of:
A. Lichen planus
B. *Tinea infection
C. Scabies
D. Virus infection
E. Syphilis
Anthropophilic fungi means spread from:
A. *Human to human
B. Animals to man
C. Soil to man
D. All are correct
E. Animal to animal
Zoophilic fungi means spread from:
A. Human to human
B. *Animals to man
C. Soil to man
D. All are correct
E. Animal to animal
Geophilic fungi means spread from:
A. Human to human
B. Animals to man
C. *Soil to man
D. All are correct
E. Animal to animal
173.
A.
B.
C.
D.
E.
174.
A.
B.
C.
D.
E.
175.
A.
B.
C.
D.
E.
176.
A.
B.
C.
D.
E.
177.
A.
B.
C.
D.
E.
178.
A.
B.
C.
D.
E.
179.
A.
B.
C.
D.
E.
180.
A.
B.
C.
D.
E.
181.
A.
B.
C.
Typical skin lesion of tinea is:
Scales
Ulcer
Crust
*Annular or arcuate plaque
All are correct
Scutula skin lesions characterized for:
Furuncle
Carbuncle
*Favus
Lichen planus
Syphilis
Wood’s lamp examination- method of diagnosis of:
Atopic dermatitis
Eczema
Virus infection
Scabies
*Tinea infection
Onychomycosis - means:
*Tinea infection of the nails
Tinea infection of the head
Tinea infection of the skin
Tinea infection of the hand
All are correct
Onychomycosis can be:
Normotrophic
Hypertrophic
Atrophic
All are wrong
*All are correct
Predisposing factors of tinea pedis are:
Occlusive foot wear
Hyperhidrosis of soles
Sharing of wash places
Presence of tinea unguium
*All are correct
Skin lesion of pityriasis versicolor are:
Nodules
Fissures
*Macula's
Erosions
All are correct
The sites of choice in pityriasis versicolor are:
Upper part of the trunk
Upper parts of the arms
Neck
Forehead and cheeks
*All are correct
Main complaint of patients with pityriasis versicolor are:
Severe pain
*Asymptomatic
Hard itch
182.
183.
184.
185.
186.
187.
188.
189.
190.
D. Enlargement of lymphatic nodes
E. All are correct
The clinical picture of pityriasis versicolor is:
A. Well demarcated, indurate plaque
B. Flaccid bulla
C. *Macula
D. Nodule
E. Ulcer
The main sites of choice in pityriasis versicolor are :
A. *Upper part of the trunk and arms
B. Nails
C. Hair
D. Groin
E. All are correct
The causative agent of pityriasis versicolor is:
A. Epidermophyton
B. *Malassezia furfur
C. Microsporum
D. Trichophyton
E. All are correct
Mycoses caused by dermatophytes are called:
A. Dermatophytoses
B. Tinea
C. Ringworm
D. Pityriasis versicolor
E. *All are correct, except pityriasis versicolor
Classification of tinea according site of involvement include all, except:
A. Tinea capitis
B. *Tinea versicolor
C. Tinea barbae
D. Tinea cruris
E. Tinea pedis
Tinear barbae means affection of:
A. Scalp
B. Hands
C. *Beared area
D. Nails
E. Groin
In case of oral candidiasis white adherent plaques are seen over:
A. Buccal mucosa
B. Tongue
C. Palate
D. Gingiva.
E. *All are correct
Kerion is:
A. Inflammatory type of tinea cruris
B. *Inflammatory type of tinea capitis
C. Inflammatory type of tinea pedis
D. Inflammatory type of tinea unguium
E. Inflammatory type of tinea manuum
Favus is:
A. Non-inflammatory type of tinea capitis
B.
C.
D.
E.
191.
A.
B.
C.
D.
E.
192.
A.
B.
C.
D.
E.
193.
A.
B.
C.
D.
E.
194.
A.
B.
C.
D.
E.
195.
A.
B.
C.
D.
E.
196.
A.
B.
C.
D.
E.
197.
A.
B.
C.
D.
E.
198.
A.
B.
C.
D.
E.
Inflammatory type of tinea pedis
Non-Inflammatory type of tinea unguium
*Inflammatory type of tinea capitis
Inflammatory type of tinea manuum
Therapy of dermatophytes include:
*Ketoconazole
Tricyclic antidepressants
Leukotriene blockers
Cyclosporine
Steroids
Black dot type is:
Non-inflammatory type of tinea cruris
*Non inflammatory type of tinea capitis
Non-inflammatory type of tinea manuum
Non-inflammatory type of tinea unguium
Non-inflammatory type of tinea pedis
There are such types of tinea pedis:
Intertriginous
Hyperceratotic
Vesicular
Squamosis
*All are correct
Wood’s lamp examination- method diagnosis of:
Syphilis
Psoriasis
*Pyoderma
Scabies
All are correct
Culture of Sabouraud’s media is used for diagnosis of:
Salmonella
Streptococcus
Treponema pallidum
*Tinea
Chlamydia
KOH preparation is used for diagnosis of:
Streptococcus
Treponema pallidum
Salmonella
Chlamydia
*Tinea
Culture of Sabouraud’s media is used for diagnosis of:
Streptococcus
Treponema pallidum
*Candidiasis
Salmonella
Chlamydia
Wood’s lamp examination- method of diagnosis:
Syphilis
Psoriasis
Lichen planus
*Pityriasis versicolor
All are correct
199.
A.
B.
C.
D.
E.
200.
A.
B.
C.
D.
E.
201.
A.
B.
C.
D.
E.
202.
A.
B.
C.
D.
E.
203.
A.
B.
C.
D.
E.
204.
A.
B.
C.
D.
E.
205.
A.
B.
C.
D.
E.
206.
A.
B.
C.
D.
E.
207.
A.
B.
C.
KOH preparation is used for diagnosis of:
Chlamidia
Streptococcus
Treponema pallidum
Salmonella
*Pityriasis versicolor
Dermatophytosis is superficial fungal infection, which affected:
*Sratum corneum
Stratum lucidum
Stratum spinosum
Stratum basale
Stratum granulosum
Atopic dermatitis is characterized by all, except:
Scaling
Excoriations
*Burrow
Oozing
Lichenification
Dermatophytosis is superficial fungal infection, which affected:
Stratum lucidum
Stratum spinosum
Stratum basale
*Sratum corneum
Derma
Different color of macula’s are characterized for:
Tinea
*Pituriasis versicolor
Psoriasis
Pioderma
All are correct
Dermatophytes which grow inside and outside the hair shaft we call:
*Endothrix and ectothrix
Endothrix
Ectothrix
All are correct
All are wrong
Broken hair which look like as hook (coma) is found in:
Kerion
Secondaty siphilis
Favus
All are correct
*Non inflammatory type of tinea capitis
Pityriasis with variety of color is:
Pityriasis alba
Pityriasis rosea
Pityriasis lichenoides
*Pityriasis versicolor
All are correct
Typical skin lesion of tinea is:
Ulcer
Crust
Bulla
208.
209.
210.
211.
212.
213.
214.
215.
216.
D. *Annular or arcuate plaque
E. All are correct
Balzer test is used for diagnosis of:
A. Candidiasis
B. Tinea
C. *Pityriasis versicolor
D. Dermatomycosis
E. All are correct
Favus is characterized by such forms:
A. Scutular form
B. Squamosis form
C. Impetigo’s form
D. Infiltrarive-purulent form
E. *All are correct
“Stamp symptom” is characterized for:
A. Trichophytia
B. Dermatophytia
C. Rubrophitia
D. All are correct
E. *Microsporia
Skin lesion of pityriasis versicolor are:
A. *Macula's
B. Nodules
C. Fissures
D. Papules
E. Bullas
Favus referred to:
A. *Tinea capitis
B. Tinea barbae
C. Tinea cruris
D. Tinea pedis
E. All are wrong
Site of infection by tinea cruris is:
A. Scalp
B. Feet
C. Face
D. *Groin
E. Nails
Which onychomycosis presentation relates to immunosuppresed condition?
A. Distal subungual
B. *Proximal subungual
C. Nail pitting
D. Onycholysis
E. All are correct
Therapy of dermatophytes include:
A. Leukotriene blockers
B. Cyclosporine
C. Steroids
D. *Terbinafine
E. Tricyclic antidepressants
How many types of onychomycosis you know?
A. 5
B.
C.
D.
E.
4
6
*3
2
217.
What is the main patient complain is characterized for tinea corporis?
A. *Itching
B. Pain
C. Intoxication
D. Fever
E. All are wrong
218.
Which areas are the most common infected in case of tinea pedis?
A. Head
B. *Feet
C. Chest
D. Arms
E. Trunk
219.
In case of acute vesicular tinea pedis specimens for potassium hydroxide
examination should be taken from:
A. Under the vesicle
B. *Root of the vesicle
C. Maculae’s
D. Burrow
E. Around the vesicle
220.
What is the meaning of tinea?
A. *Fungal infection
B. Bacterial infection
C. Sexually transmitted disease
D. All are correct
E. Viral infection
221.
Color of fluorescence in Wood’s light in case of tinea capitis will be:
A. Yellow
B. Coral pink
C. *Green
D. Brown
E. Blue
222.
Tinea Versicolor treatment include:
A. Leukotriene blockers
B. H1 anti-histamines
C. Tricyclic antidepressants
D. Steroids
E. *Fluconazole
223.
Color of fluorescence in Wood’s light in case of erythrasma will be:
A. Green
B. Yellow
C. *Coral red
D. Brown
E. Blue
224.
Treatment of urticaria means all, except:
A. Adrenaline
B. *Itraconazole
C. Terfinadin
D. Desloratidin
225.
226.
227.
228.
229.
230.
231.
232.
233.
E. Hydrocortisone
What factors determine the treatment of tinea infection?
A. Site of lesions
B. Extent of lesions
C. Chronic disease
D. Patient complaints
E. *All are correct
Therapy of dermatophytes include:
A. Cyclosporine
B. Steroids
C. *Itraconazole
D. Tricyclic antidepressants
E. Leukotriene blockers
Atopic dermatitis is characterized by all, except:
A. Dry skin
B. Pruritus
C. Erythema
D. Edema
E. *Hive
“Atopic triad” means:
A. *Asthma, allergic rhinitis, atopic dermatitis
B. Asthma, atopic dermatitis, psoriasis
C. Atopic dermatitis, eczema, urticaria
D. Atopic dermatitis, asthma, urticaria
E. Allergic rhinitis, urticaria, atopic dermatitis
“Jock itch” means:
A. Tinea corporis
B. Tinea barbae
C. *Tinea cruris
D. Tinea capitis
E. Tinea Pedis
“Smooth skin” means:
A. *Tinea corporis
B. Tinea barbae
C. Tinea cruris
D. Tinea capitis
E. Tinea Pedis
In case of Tinea versicolor yeast grows in:
A. Stratum spinosum
B. Stratum lucidum
C. *Stratum corneum
D. Stratum basale
E. Stratum granulosum
Favus is caused by:
A. T.rubrum
B. M.canis
C. E.floccosum
D. *All are wrong
E. Microsporum
Tinea Versicolor treatment include:
A. Cyclosporine
B. H1 anti-histamines
C. H2 anti-histamines
D. *Ketoconazole
E. Steroids
234.
Hyperpigmented or depigmented maculae on chest, back, arms, abdomen are
characterised for:
A. Psoriasis
B. *Tinea Versicolor
C. Tinea corporis
D. Tinea cruris
E. Tinea manuum
235.
In case of oral candidiasis white adherent plaques are seen over:
A. Gingiva
B. Tongue
C. Palate
D. *All are correct
E. All are wrong
236.
Direct microscopic examination of KOH-treated skin scrapings which show a
typical aspect of mycelia and spores described as "spaghetti and meatballs” is
characterized for:
A. Tinea pedis
B. Tinea corporis
C. *Tinea versicolor
D. Tinea cruris
E. Tinea manuum
237.
Therapy of dermatophytes include:
A. Steroids
B. Cyclosporine
C. Tricyclic antidepressants
D. Leukotriene blockers
E. *Miconazole
238.
Such genera comprise group of dermatophytes:
A. Microspourm
B. Trichophyton
C. Epidermophyton
D. *All are correct
E. All are wrong
239.
Dermatophytes which grow only inside the hair shaft we call:
A. *Endothrix
B. Ectothrix
C. Endothrix and ectothrix
D. All are correct
E. All are wrong
240.
Dermatophytes means:
A. *All are correct
B. Microsporum
C. Trichophyton
D. Epidermophyton
E. All are wrong
241.
Superficial mycoses are:
A. *Fungal infections of the outermost keratinized layers of the skin
B. Viral infections of the outermost keratinized layers of the skin
C. Fungal infections of the derma
242.
243.
244.
245.
246.
247.
248.
249.
250.
D. Viral infections of the derma
E. Viral infection of the epidermis
Dermatophytosis involve:
A. Skin
B. Hair
C. Nails
D. All are wrong
E. *All are correct
Tinea versicolor - characterized by a blotchy discoloration of:
A. *Skin
B. Nails
C. Hair
D. All are wrong
E. All are correct
Ringworm of the bearded areas of the face and neck, known as:
A. Tinea corporis
B. Tinea cruris
C. *Tinea barbae
D. Tinea manuun
E. Tinea pedis
Microsporum affected:
A. Only skin
B. Only hair
C. *Skin and hair
D. Skin and nails
E. All are wrong
Epidermophyton affected:
A. Only skin
B. Only hair
C. Only nails
D. *Skin and nails
E. All are wrong
Trichophyton affected:
A. Only skin
B. Only hair
C. Only nails
D. *Skin, hair, nails
E. All are wrong
Tinea cruris means:
A. Tinea of beard zone
B. Tinea of the nails
C. Tinea pedis
D. Tinea of the scalp
E. *All are wrong
Tinea cruris means:
A. Tinea of beard
B. Tinea of the trunk
C. Tinea pedis
D. Tinea of the scalp
E. *Tinea of the groin
Onycholysis means:
A. Tinea of the beard zone
B.
C.
D.
E.
251.
B.
C.
D.
E.
F.
252.
A.
B.
C.
D.
E.
253.
A.
B.
C.
D.
E.
254.
A.
B.
C.
D.
E.
255.
A.
B.
C.
D.
E.
256.
A.
B.
C.
D.
E.
257.
A.
B.
C.
D.
E.
258.
A.
B.
C.
D.
E.
*Separation of the nail from its bed
Tinea of the nails
Discoloration of the nails
All are wrong
Favus caused by:
*T.schoenleinii
T.mentagrophytes
T.rubrum
M.canis
E.floccosum
Predisposing factors of candidiasis are:
Diabetes mellitus
Antibiotics
Obesity (friction)
Poor hygiene
*All are correct
Predisposing factors of candidiasis are:
High humidity
Antibiotics
Obesity (friction)
Poor hygiene
*All are correct
Candida intertrigo localized:
Under the breasts
*All are correct
In the armpits
In the groin
All are wrong
In case of oral candidiasis white adherent plaques are seen over:
Buccal mucosa
Tongue
Palate
*All are correct
Genital organs
Treatment of urticaria means all, except:
Diphenhydramine
Terfinadin
Desloratidin
Hydrocortisone
*Miconazole
Fluorescent “Wood’s lamp” examination can be used to identify:
Microsporum canis
Corynebacterium
Malassia furfur
Microsporum andouinii
*All are correct
Pemphigus vulgaris is:
Viral disease
*Autoimmune disease
Bacterial disease
Sexually transmitted disease
Papulosquamosis disease
259.
Common term used for vesicle or bulla means:
A. *Blister
B. Patch
C. Plaque
D. Crust
E. All are wrong
260.
Nikolsky’s sign is positive in:
A. Psoriasis
B. Scabies
C. *Bullous disorders (Pemphigus)
D. Syphilis
E. Lichen planus
261.
Acantholytic epidermal cells are characterized for:
A. Psoriasis
B. Pyoderma
C. Lichen planus
D. *Pemphigus vulgaris
E. Tinea
262.
Fluid-filled raised lesion > 5 mm across means:
A. Wheal
B. Macula
C. Plaque
D. Patch
E. *All are wrong
263.
Clinical features of Pemphigus Vulgaris are all, except:
A. Tense of flaccid bulla appear on normal skin
B. The lesions may be few and sparse, or extensive
C. The eruption is usually symmetrical
D. They are usually irregular in shape
E. *Nodules are certain skin lesions
264.
Tzanck smear is used as a diagnostic test for which of the following skin
condition?
A. Scabies
B. HIV
C. Psoriasis
D. *Pemphigus Vulgaris
E. Eczema
265.
Common skin lesions above the plane of the skin are:
A. Vesicle
B. Pustule
C. Bulla
D. Nodule
E. *All are correct
266.
Common skin lesions above the plane of the skin are all, except:
A. Papule
B. Nodule
C. Cyst
D. *Patch
E. Vesicle
267.
Clinical features of Pemphigus Vulgaris are all, except:
A. Tense of flaccid bulla appear on normal skin
B. On rupturing, form painful erosions
C. The eruption is usually symmetrical
D. They are usually irregular in shape
E. *Nikolsky’s sign is negative
268.
Fluid-filled raised lesion > 5 mm across means:
A. Wheal
B. *Bulla
C. Plaque
D. Patch
E. Nodule
269.
Therapy of HSV infection include all, except:
A. Acyclovir
B. Valacyclovir
C. Famciclovir
D. H1 anti-histamines
E. *Fluconazole
270.
Clinical features of Pemphigus Vulgaris are all, except:
A. Tense of flaccid bulla appear on normal skin
B. On rupturing, form painful erosions
C. The eruption is usually symmetrical
D. *They are usually regular in shape
E. Nikolsky’s sign is positive
271.
Pemphigus Foliaceus localized in:
A. *Epidermis
B. Derma
C. Subcutis
D. Epidermis, derma
E. All are correct
272.
Tzanck smear is used as a diagnostic test for which of the following skin
condition?
A. HIV
B. Psoriasis
C. *Pemphigus Vulgaris
D. Lichen planus
E. Impetigo
273.
Fluid-filled raised lesion 5 mm or less across means:
A. Nodule
B. Wheal
C. *Vesicle
D. Plaque
E. Patch
274.
Pemphigus Foliaceus is characterised by all, except:
A. Presence of fflaccid bulla and exfoliating scales
B. *Presence of papules
C. Slowly the disease spreads
D. Nikolsky’s sign is positive
E. Bulla rapture rapidly
275.
Therapy of HSV infection include all, except:
A. Acyclovir
B. H2 anti-histamines
C. Valacyclovir
D. *Itraconazole
E. Famciclovir
276.
A.
B.
C.
D.
E.
277.
A.
B.
C.
D.
E.
278.
A.
B.
C.
D.
E.
279.
A.
B.
C.
D.
E.
280.
A.
B.
C.
D.
E.
281.
A.
B.
C.
D.
E.
282.
A.
B.
C.
D.
E.
283.
A.
B.
C.
D.
E.
284.
A.
B.
C.
Blister means:
*Bulla
Plaque
Patch
Nodule
Cyst
Exfoliating scales are characterized for such types of pemphigus:
Pemphigus Vegetans
*Pemphigus Foliaceus
Pemphigus Vulgaris
Pempgigus Erythematosus
All are correct
Acantholytic epidermal cells are characterized for :
Pyoderma
Psoriasis
*Pemphigus foliaceus
Lichen planus
Tinea
Fluid-filled raised lesion 5 mm or less across means:
Patch
Nodule
Wheal
Plaque
*All are wrong
Senear-Usher syndrome is synonym of:
Pemphigus Foliaceus
Pemphigus Vulgaris
Pemphigus Vegetans
*Pempgigus Erythematosus
Phemhigoid
The main characteristic skin lesion of Pemphigus Vulgaris is:
Burrow
Nodule
Cyst
*Bulla
Erosion
Common skin lesions above the plane of the skin are all, except:
Papule
Vesicle
Nodule
Bulla
*Macula
The main characteristic skin lesion of Pemphigus Erythematosus is:
Cyst
Burrow
Plaque
*Bulla
Nodule
Blister means:
*Vesicle
Plaque
Patch
D. Nodule
E. Borrow
285.
Pruritic, polygonal, flat papules and white striaform symmetric lesions in the
buccal mucosa are characterized for:
A. Pemphigus foliaceus
B. Pemphigus vegetans
C. *Lichen planus
D. Pemphigus vulgaris
E. Pemphigus vegetans
286.
Which of the pemphigus never involve mucosal surface?
A. Pemphigus Erythematosus
B. Pemphigus Vegetans
C. *Pemphigus Foliaceus
D. Pemphigus Vulgaris
E. All are wrong
287.
Senear-Usher disease means:
A. *Pemphigus Erythematosus
B. Pemphigus Vegetans
C. Pemphigus Vulgaris
D. Pemphigus Foliaceus
E. All are wrong
288.
Acantholysis means:
A. Intercellular edema between the keratinocytes in the epidermis
B. Is thickening of the stratum corneum, often associated with a qualitative
abnormality of the keratin
C. *Loss of intercellular connection in epidermis
D. Diffuse epidermal hyperplasia
E. The formation of vacuoles within or adjacent to cells
289.
Tzanck smear is used as a diagnostic test for which of the following skin
condition?
A. Psoriasis
B. HIV
C. *HSV
D. Lichen planus
E. Impetigo
290.
Fluid from intact vesicle smeared on a microscope slide which is stained with
either Giemsa’s or Wright’s stain means:
A. Balzer test
B. Auspits sign
C. *Tzanck smear
D. Psoriatic phenomenon
E. Woronow wring
291.
Systemic therapy in dermatology means:
A. Anti fungal
B. Antibacterial agents
C. Anti viral agents
D. *All are correct
E. Antihistamines drugs
292.
Fluid from intact vesicle and smeared on a microscope slide which is stained with
either Giemsa’s or Wright’s stain means:
A. “Auspits” sign
B. “Balzer”test
C. Psoriatic phenomenon
D. “Woronow wring”
E. *All are wrong
293.
Loss of intercellular connections resulting in loss of cohesion between
keratinocytes is:
A. Vacuolization
B. Spongiosis
C. Exocytosis
D. Dyskeratosis
E. *All are wrong
294.
Pemphigus vulgaris is:
A. Papulosquamosis disease
B. Pyoderma
C. Bacterial disease
D. Sexually transmitted disease
E. *ll are wrong
295.
Common skin lesions above the plane of the skin are:
A. Erosion
B. Macula
C. Patch
D. *Bulla
E. Excoriation
296.
Pemphix means:
A. Bubble or papule
B. Bubble or patch
C. *Bubble or blister
D. Nodule
E. Patch
297.
Color of Wood’s light in case of Erythrasma is:
A. Dull yellow
B. *Coral
C. Green
D. Compelete depigmentation
E. All are wrong
298.
Treatment of Pemphigus vulgaris means:
A. Oral steroids
B. Systemic steroids
C. Immunosuppressive agents
D. *All are correct
E. All are wrong
299.
Pemphigus occurs in patients with:
A. Papulosquamosis disease
B. Viral disease
C. *Autoimmune diseases
D. Bacterial disease
E. Sexually transmitted disease
300.
Pemphigus vulgaris can involve:
A. Skin
B. Esophagus
C. Cervix
D. Oral cavity
E. *All are correct
301.
Application of tangential pressure on normal skin results in formation of anew
bulla means:
A. “Auspits” sign
B. *”Nikolsky’s” sign
C. “Balze”r test
D. “Woronow” wring
E. All are wrong
302.
Application on pre-existing bulla results in the spread of bulla, means:
A. “Balzer” test
B. “Auspits” sign
C. “Woronow” wring
D. *“Nikolsky’s” sign
E. All are wrong
303.
In case of Pemphigus vulgaris acantholysis produced:
A. Intraepithelial vesicles or papule
B. *Intraepithelial vesicles or bulla
C. Intraepithelial vesicles or nodule
D. Intraepithelial cyst or bulla
E. All are wrong
304.
In case of Pemphigus vulgaris intraepithelial vesicles or bulla appeared in:
A. Stratum basale
B. Prickle cell layer
C. *Stratum spinosum or the prickle cell layer
D. Dermis
E. Subcutis
305.
Pemphigus vulgaris is characterized with:
A. Flaccid blister
B. Clear fluid inside blister
C. Healthy skin around blister
D. *All are correct
E. Positive Nikolsky’s sign
306.
The main aim in treating Pemphigus vulgaris is to:
A. Decrease blister formation
B. Promote healing of blister
C. Determine the minimum dose of medication necessary to control the disease
process
D. *All are correct
E. Prevent secondary complications
307.
Diet in case of Pemphigus vulgaris means restricted on:
A. Spicy foods
B. Tomatoes
C. Orange juice
D. *All are correct
E. Nuts
308.
Diet in case of Pemphigus vulgaris means restricted on hard foods that may
traumatize oral epithelium mechanically:
A. Nuts
B. Chips
C. Hard vegetables
D. *All are correct
E. All are wrong
309.
Human Herpes Viruses can be:
A.
B.
C.
D.
E.
310.
A.
B.
C.
D.
E.
311.
A.
B.
C.
D.
E.
312.
A.
B.
C.
D.
E.
313.
A.
B.
C.
D.
E.
314.
A.
B.
C.
D.
E.
315.
A.
B.
C.
D.
E.
316.
A.
B.
C.
D.
E.
317.
A.
B.
C.
D.
Alphaherpesvirinae
Betaherpesvirinae
Gammaherpesvirinae
*All are correct
All are wrong
Complications of herpes zoster are:
Post herpetic pain
Ophthalmic zoster -corneal scarring
Loss of vision
*All are correct
All are wrong
“Shingles” means:
Herpes labialis
*Zoster ophthalmicus
Cytomegalovirus
Chicken pox
All are wrong
Therapy of HSV infection include:
Acyclovir
Valacyclovir
Famciclovir
*All are correct
Antihistamines
To inspect skin means look for signs of:
Edema
Moisture
Petechiae
Ecchymosis
*All are correct
We must assess each lesion for:
Asymmetry of shape
Border irregularity
Color variation within one lesion
Diameter
*All are correct
We must inspect and palpate the hair for:
Cleanliness
Distribution, Cleanliness
Correct is answer A
*Correct is answer B
All are wrong
We must evaluate fingernails and toenails for:
Color
Shape
Thickness
Texture
*All are correct
Minor associations with the aging process of the nail include:
Gradual thickening of the nail plate
Presence of longitudinal ridges
Yellowish-gray discoloration
*All are correct
E. All are wrong
318.
Diagnostic tests in dermatology are:
A. Skin biopsy
B. Skin cultures
C. “Tzanck” smear
D. *All are correct
E. “Balzer” test
319.
Classic recurring cold sore is:
A. Varicella Zoster
B. *Type 1 herpes simplex virus
C. Cytomegalovirus
D. Chicken pox
E. Epstein-Barr virus
320.
Classic recurring cold sore is:
A. Type 2 herpes simplex virus
B. Cytomegalovirus
C. Chicken pox
D. Varicella Zoster
E. *All are wrong
321.
Genital herpes is:
A. Cytomegalovirus
B. *Type 2 herpes simplex virus
C. Chicken pox
D. Varicella Zoster
E. All are wrong
322.
After first infection, Herpes Zoster Virus dormant in a:
A. Stratum corneum
B. Stratum basale
C. *Nerve ganglia
D. Stratum pallidum
E. All are wrong
323.
Herpetic whitlow-a form of:
A. Cytomegalovirus
B. Chicken pox
C. *Herpes simplex infection
D. Varicella Zoster
E. Epstein-Barr virus
324.
Multiple lesions occur in a segmental distribution on the skin area innervated by
the infected nerve are characterized for:
A. Epstein-Barr virus
B. Chicken pox
C. *Herpes zoster
D. Herpes simplex
E. Pemphigus Vulgaris
325.
Herpes Zoster caused by reactivation of the dormant virus in clients who have
previously had:
A. Herpes Zoster
B. *Chickenpox
C. Epstein-Barr virus
D. Cytomegalovirus
E. All are correct
326.
Systemic therapy in dermatology means:
A.
B.
C.
D.
E.
Anti viral agents
Antibacterial agents
Anti fungal agents
Antihistamines
*All are correct
327.
Palpation of the skin confirms all, except:
A. The size of the lesions
B. *Psoriatic phenomenon
C. Type of lesions
D. Skin temperature
E. Turgor
328.
Herpes Zoster caused by reactivation of the dormant virus in clients who have
previously had:
A. *Chickenpox
B. Pemphigus erythematous
C. Epstein-Barr virus
D. Cytomegalovirus
E. Pemphigus vulgaris
329.
In case of Herpes Zoster eruption lasts for:
A. Several days
B. *Several weeks
C. Several hours
D. All are wrong
E. Several years
330.
Herpetic whitlow-a form of herpes simplex infection occurring:
A. On the scalp of medical personnel
B. *On the fingertips of medical personnel
C. On the groin of medical personnel
D. All are wrong
E. All are correct
331.
Environmental triggers for re-activation of HSV are:
A. Stress
B. UV radiation
C. Menstruation
D. All are wrong
E. *All are correct
332.
Environmental triggers for re-activation of HSV are:
A. Fever
B. Pregnancy
C. Immunosuppression
D. *All are correct
E. All are wrong
333.
Objective assessment of a patient with a skin complaint (special examination
technique) include:
A. Diascopy
B. Skin-surface microscopy
C. Wood’s light
D. *All are correct
E. Skin scrapping
334.
In diagnosis of skin disease we can use:
A. Routine tests
B. Special tests
335.
336.
337.
338.
339.
340.
341.
342.
343.
C. Skin Biopsy
D. Blood analysis
E. *All are correct
Special tests in dermatology means all, except:
A. Skin scrapings
B. Patch test
C. *Blood analysis
D. Tzank smear
E. Electron microscopy
Therapy of HSV infection include all, except:
A. Acyclovir
B. *Miconazole
C. H1 anti-histamines
D. Valacyclovir
E. Famciclovir
Dermatological status of patient include all, except:
A. Diascopy
B. *Tzank smear
C. Inspection
D. Palpation
E. Scraping
Tzanck preparation means:
A. Culture is taken from a fresh blister.
B. The slide is prepared with Giemsa
C. The slide is prepared with Wright’s stain
D. *All are correct
E. All are wrong
Color of Wood’s light in case of Tinea Versicolor is:
A. *Dull yellow
B. Coral
C. Green
D. Compelete depigmentation
E. All are wrong
Color of Wood’s light in case of tinea capitis is:
A. Dull yellow
B. Coral
C. *Green
D. Compelete depigmentation
E. All are wrong
Treatments of skin disease can be:
A. Topical
B. *All are correct
C. Systemic
D. Surgical
E. All are wrong
Dermatological status of patient include:
A. Inspection
B. Palpation
C. Scraping
D. Diascopy
E. *All are correct
PUVA or Photochemotherapy used mainly for severe:
A.
B.
C.
D.
E.
344.
A.
B.
C.
D.
E.
345.
A.
B.
C.
D.
E.
346.
A.
B.
C.
D.
E.
347.
A.
B.
C.
D.
E.
348.
A.
B.
C.
D.
E.
349.
A.
B.
C.
D.
E.
350.
A.
B.
C.
D.
E.
351.
A.
B.
C.
D.
*Psoriasis
Type 1 herpes simplex virus
Cytomegalovirus
Chicken pox
Varicella Zoster
PUVA or Photochemotherapy used mainly for:
*All are wrong
Type 1 herpes simplex virus
Cytomegalovirus
Chicken pox
Varicella Zoster
Systemic therapy in dermatology means:
Immunosuppressive
Anti viral agents
Antibacterial agents
Anti fungal agents
*All are correct
During skin inspection doctor must look for signs of:
Edema
Moisture
Petechiae
*All are correct
Color
During skin inspection doctor must look for all signs, except:
Petechiae
Ecchymosis
Edema
Moisture
*Tzank smear
Palpation of the skin confirms:
The size of the lesions
Type of lesions
Skin temperature
Turgor
*All are correct
Palpation of the skin confirms all, except:
The size of the lesions
Type of lesions
*Balzer test
Skin temperature
Turgor
Each lesion is necessary to assess for:
Asymmetry of shape
*All are correct
Border irregularity
Color variation within one lesion
Diameter
Punch biopsy: 2-10 mm in diameter, helpful in the diagnosis of:
Discoid lupus
Bullous pemphigoid
Cutaneous T-cell Lymphoma
*All are correct
352.
353.
354.
355.
356.
357.
358.
359.
360.
E. All are wrong
Nail assessment include:
A. Nail bed color
B. *All are correct
C. Texture & Consistency
D. Nail angle
E. Surrounding tissues
Assessment of hair means:
A. Alopecia
B. *All are correct
C. Color
D. Distribution
E. All are wrong
Abnormal color of the nail means:
A. Cyanosis
B. Leukonychia
C. Pink
D. *Correct are A and B
E. Correct are B and C
Abnormal color of the nail means:
A. Cyanosis
B. Leukonychia
C. Melanochyia
D. Correct are A and B
E. *All are correct
Abnormal color of the nail means:
A. Melanochyia
B. Pink
C. Cyanosis
D. Correct are B and C
E. *Correct A and C
Human Herpes Viruses can be:
A. Alphaherpesvirinae
B. Alphaherpesvirinae, Betaherpesvirinae
C. Alphaherpesvirinae, Gammaherpesvirinae, Betaherpesvirinae
D. Correct is answer B
E. *Correct is answer C
Complications of herpes zoster are:
A. Only post herpetic pain
B. Post herpetic pain, ophthalmic zoster -corneal scarring
C. Only loss of vision
D. Only ophthalmic zoster corneal scarring
E. *Correct is answer B
Tinea pedis means:
A. Scabies of the soles
B. Eczema of the soles
C. Dermatitis of the soles
D. *Dermatophytosis of the soles
E. Psoriasis of the soles
Sexually ttransmitted infections includes:
A. Pediculosis Pubis
B. Scabies
361.
362.
363.
364.
365.
366.
367.
368.
369.
C. Condyloma acuminate
D. Condyloma lata
E. *All are correct
“Venera necklace” is :
A. *Combination of hyperpigmentation and depigmentation in secondary syphilis
B. Combination of hyperpigmentation and nodules in secondary syphilis
C. Combination of nodules and depigmentation in secondary syphilis
D. Affection of hair in secondary syphilis
E. Nodules in secondary syphilis
Sexually transmitted infections includes:
A. Genital Herpes
B. Granuloma Inguinale (Donovanosis)
C. Lymphogranuloma Venereum
D. Correct are B and C
E. *All are correct
Sexually transmitted infections includes:
A. Chancroid
B. *All are correct
C. Syphilis
D. Human Immunodeficiency Virus (HIV)
E. Correct are A and C
The main clinical symptoms of primary syphilis are:
A. “Hard chancre”
B. *Hard chancre, regional lymphadenitis
C. Vaginal discharges
D. Roseola
E. Regional lymphadenitis
“Hard chancre” is:
A. Soft
B. Thick
C. Painful
D. Painless
E. *Thick and painless
Variations of the classic chancre are:
A. Non-indurate ulcers
B. Painful ulcers
C. Multiple ulcers
D. *All are correct
E. All are correct, except ”non-indurate ulcers “
“Hard chancre” is clinical symptom of:
A. Gonococcal infection
B. *Primary syphilis
C. Secondary syphilis
D. Late syphilis
E. Congenital syphilis
Typical “chard chancre” is:
A. Macular
B. *Button like , painless lesion
C. Patch
D. Nodule
E. Nodule, maculae, patch
Treponema pallidum is:
A.
B.
C.
D.
E.
370.
A.
B.
C.
D.
E.
371.
A.
B.
C.
D.
E.
372.
A.
B.
C.
D.
E.
373.
A.
B.
C.
D.
E.
374.
A.
B.
C.
D.
E.
375.
A.
B.
C.
D.
E.
376.
A.
B.
C.
D.
E.
377.
A.
B.
C.
D.
Virus
Protozoa
Dermatophytes
*Spirochete
Mite
“Hard chancre” begins as:
A single papule
Well defined papule
Painless papule
*All are correct
Button like papule
“Hard chancre” begins as:
Regular red brown papule
Indurate (button-like) red brown papule
Vaginal discharges
All are correct
*All are correct, except “vaginal discharges”
Variations of the classic chancre are:
Painful ulcers
Multiple ulcers
*All are correct
Secondarily infected ulcers
Correct are A and B
Duration of primary Sero-negative period of syphilis is:
1 week
2 weeks
3 month
Till appearance of roseola
*Till appearance positive results of Wasserman reaction
Syphilis cannot be contracted through:
Toilet seats
Daily activities
Accidental inoculation
*Correct are A and B
All are correct
Syphilis cannot be contracted through:
Through contaminated blood
Hot tubs
Daily activities
*Correct are B and C
All are correct
The mode of transmission of syphilis can be all, except:
Sexual
*Toilet seats
Kissing
Sexual perversion
Accidental inoculation
The mode of transmission of syphilis can be all, except:
Through contaminated blood
Toilet seats
Correct is only answer A
*Correct is only answer B
E. During delivery
378.
The mode of transmission of syphilis can be all, except:
A. Accidental inoculation
B. Through contaminated blood
C. *Daily activities
D. Sex
E. Kissing
379.
Treponema pallidum can be observed by:
A. Gram stain
B. *Dark-field microscopy
C. Direct visualization
D. Prick test
E. All are wrong
380.
Which of the following methods should be undertaken for Treponema pallidum
diagnosis?
A. Prick test
B. “Burrow-ink test”
C. *Dark-field microscopy
D. Balzer test
E. Wood’s light
381.
Clinical improvement of primary syphilis are:
A. Incubation period 3-4 month
B. Presence of subjective sensations
C. All are correct
D. *Wassermann reaction is positive
E. All are wrong
382.
Clinical improvement of primary syphilis are :
A. Button-like, painless ulcer
B. *All are correct
C. Incubation period 3-4 weeks
D. Absence of subjective sensations
E. Wasserman reaction is positive
383.
Treponema pallidum is:
A. Stable in outside
B. Very sensitive to dehumidification
C. Very sensitive to boiling
D. Correct is only answer B
E. *All are correct, except “stable in outside”
384.
Treponema pallidum is:
A. Very sensitive to disinfectants
B. Very sensitive to dehumidification
C. Not stable in outside
D. Very sensitive to boiling
E. *All are correct
385.
Primary sero-negative syphilis means:
A. Presence of chard chancre
B. Enlargement of regional lymphatic node
C. Diagnosis of treponema pallidum by dark-field microscopy
D. *All are correct
E. Wassermann’s reaction – is negative
386.
Complication of hard chancre is:
A. Arthritis
B.
C.
D.
E.
387.
A.
B.
C.
D.
E.
388.
A.
B.
C.
D.
E.
389.
A.
B.
C.
D.
E.
390.
A.
B.
C.
D.
E.
391.
A.
B.
C.
D.
E.
392.
A.
B.
C.
D.
E.
393.
A.
B.
C.
D.
E.
394.
A.
B.
C.
D.
E.
*Phimosis
Bleeding
All are correct
Vaginal discharges
Complication of hard chancre is:
*Gangrene
Arthritis
Vaginal discharges
All are correct
Bleeding
Complications of hard chancre are:
Paraphimosis
Phagedenism
Gangrene
Phimosis
*All are correct
Wassermann’s reaction – specific reaction for:
Psoriasis
Pioderma
Tinea infection
*Syphilis
Gonococcal infection
Wassermann’s reaction is positive after:
2-3 days
*3-4 weeks
1-2 days
7-8 days
2 weeks
Clinical improvement of primary syphilis are all, except:
*Wassermann reaction is negative
Presence of regional lymphadenitis
Absence of subjective sensations
Incubation period 3-4 weeks
Button-like, painless ulcer
Alopecia in secondary syphilis is characterized as :
*“Moth eaten”
“Christmas tree”
“Herald patch”
“Snail-track”
All are wrong
Alopecia in secondary syphilis is characterized as :
“Biet collar”
“Christmas tree”
“Herald patch”
“Snail-track”
*All are wrong
Post-infectious immunity after treatment of syphilis can preserve:
1 year
*Can’t preserve at all
2 years
5 years
The whole life
395.
A.
B.
C.
D.
E.
396.
A.
B.
C.
D.
E.
397.
A.
B.
C.
D.
E.
398.
A.
B.
C.
D.
E.
399.
A.
B.
C.
D.
E.
400.
A.
B.
C.
D.
E.
401.
A.
B.
C.
D.
E.
402.
A.
B.
C.
D.
E.
403.
A.
B.
C.
“Hard chancre” is:
Soft
Thick
Painful
Painless
*Thick and painless
Typical “chard chancre” is:
Erosion
Papule
Nodule
Maculae
*Papule, erosion, ulcer
Typical “chard chancre” is:
*Erosion
Macular
Patch
Nodule
Nodule, maculae, patch
Secondary syphilis is characterized by:
Different types of skin lesions
*All are correct
Regress of syphilides without treatment
Presence of treponema pallidum in syphilides
Serological tests are positive
Papules in secondary syphilis can regress into:
Atrophy
Scar
*Secondary pigmentation
Calcification
All are correct
Syphilitic papules are characterized by:
Copper-red in color
Thickness
Absence of acute inflammation
Absence of subjective sensations
*All are correct
Syphilitic papules are characterized by all, except:
Copper-red in color
Thickness
*Presence of acute inflammation
Absence of subjective sensations
Absence of acute inflammation
Syphilitic papules are characterized by all, except:
Copper-red in color
Thickness
Absence of acute inflammation
*Presence of subjective sensations
Absence of acute inflammation
“Venera necklace” is:
Erosion in late syphilis
Affection of hair in secondary syphilis
Affection of nails in secondary syphilis
D. *Combination of hyperpigmentation and depigmentation in secondary syphilis
E. Nodules in secondary syphilis
404.
“Venera necklace” is:
A. Combination of hyperpigmentation and nodules in secondary syphilis
B. Affection of hair in secondary
C. *Combination of hyperpigmentation and depigmentation in secondary syphilis
D. Affection of nails in secondary syphilis
E. Nodules in secondary syphilis
405.
“Moth eaten” is symptom of:
A. *Alopecia in secondary syphilis
B. Combination of hyperpigmentation and depigmentation in secondary syphilis
C. Combination of hyperpigmentation and nodules in secondary syphilis
D. Combination of nodules and depigmentation in secondary syphilis
E. Nodules in secondary syphilis
406.
To diagnose treponema pallidum in dark-field microscopy, we must take smear
from:
A. Hard chancre
B. Erosions or ulcers from genital organs
C. Erosions or ulcers of anorectal zone
D. Erosions or ulcers of oral cavity
E. *All are correct
407.
“Biet collar” is characterized for:
A. Primary syphilis
B. *Secondary syphilis
C. Congenital syphilis
D. All are correct
E. All are wrong
408.
“Hard chancre” is clinical symptom of:
A. Both are correct
B. Secondary syphilis
C. *Primary syphilis
D. Late syphilis
E. Congenital syphilis
409.
Optimal temperature for treponema pallidum development is:
A. 60˚С
B. 20˚С
C. 70˚С
D. *37˚С
E. 45˚С
410.Treponema pallidum is characterized by all, except:
A. Is not stable in outside
B. Is very sensitive to disinfectants
C. It is very sensitive to dehumidification
D. It is very sensitive for boiling
E. *Can be diagnose by Gram stain
411.
Drugs of choice in syphilis treatment are:
A. *Penicillin
B. Corticosteroids
C. Antiviral drugs
D. Antihistamines
E. All are correct
412.
To precede Wassermann’s reaction, we must take:
A.
B.
C.
D.
E.
Patient urine
Patient saliva
*Patient serum
Patient sweat
All are wrong
413.
"Chard chancre” is characterized by:
A. Itch
B. Pain
C. Hemispherical growth
D. *Well demarcation and painless
E. Well demarcation and itch
414.
Skin lesions in secondary syphilis are connected with:
A. Itch
B. Pain
C. Paresthesia
D. Burning
E. *Subjective sensations are absent
415.
Shorting of incubation period of primary syphilis is characterized for:
A. Children
B. Old patients
C. Pregnancy women
D. All are correct
E. *All are correct, except pregnancy women
416.
Typical lesion of primary syphilis is called:
A. *Hunterian chancre
B. Hunterian scales
C. Hunterian pustules
D. Hunterian maculae’s
E. Hunterian patches
417.Treponema pallidum is characterized by all, except:
A. *Can de diagnose by Balser test
B. Is very sensitive to disinfectants
C. Is not stable in outside
D. It is very sensitive to dehumidification
E. It is very sensitive for boiling
418.
Diagnostic tests for syphilis are all, except:
A. Dark-field examination
B. VDRL
C. FTA_ABS
D. *Prick test
E. RPR
419.
Diagnostic tests for syphilis are all, except:
A. Dark-field examination
B. *Balzer test
C. VDRL
D. FTA_ABS
E. RPR
420.
Syphilis can be:
A. Early syphilis
B. Late syphilis
C. Congenital syphilis
D. Pruritc syphilis
E. *All are correct, except pruritic
421.
Condiloma lata are characterized for:
A. Lichen planus
B. Psoriasis
C. *Syphilis
D. Virus infection
E. Pituriasis versicolor
422.
Condiloma lata are characterized for:
A. *Syphilis
B. Lichen planus
C. Psoriasis
D. Virus infection
E. All are wrong
423.
Diagnostic tests for syphilis are all, except:
A. FTA_ABS
B. Dark-field examination
C. *Tzanck smear
D. VDRL
E. RPR
424.
Alopecia in secondary syphilis is characterized as:
A. “Koebner’s” phenomenon
B. “Christmas tree”
C. “Herald patch”
D. “Snail-track”
E. *All are wrong
425.
“Snail-track” is affection of :
A. Hair in secondary syphilis
B. Skin in secondary syphilis
C. *Mucous membranes in secondary syphilis
D. Bones in secondary syphilis
E. All are wrong
426.
“Snail-track” is affection of :
A. Mucous membranes in primary syphilis
B. Hair in s primary syphilis
C. Skin in secondary syphilis
D. Bones in secondary syphilis
E. *All are wrong
427.
Specific symptom of syphilitic papules is:
A. “Bleeding point”
B. “Terminal film”
C. “Stearetic spot”
D. *“Biet collar”
E. All are wrong
428.Manifestation of secondary syphilis are characterized by:
A. Affection of the skin
B. Affection of mucous membranes
C. Affection of the hair
D. Affection of the nails
E. *Affection of the skin, mucous membranes, hair
429.Manifestation of secondary syphilis are:
A. Affection of the nails
B. Affection of the skin
430.
431.
432.
433.
434.
435.
436.
437.
438.
C. Affection of mucous membranes
D. Affection of the hair
E. *Affection of the skin, mucous membranes, hair
“Biett “collar” and “Venera necklace” are characterized for:
A. Primary syphilis
B. *Secondary syphilis
C. Late syphilis
D. Congenital syphilis
E. All are correct
Primary skin lesion in primary syphilis is called:
A. *“Chard chancre”
B. “Biett “collar”
C. “Venera” necklace”
D. Mite chancre
E. All are wrong
Jarisch-Herxheimer reaction is characterized for:
A. Allergic dermatitis
B. Urticaria
C. *Syphilis
D. Tinea infection
E. All are correct
Incubation period of “classis” syphilis :
A. 2-3 days
B. 1 week
C. *3-4 weeks
D. 2-3 monthі
E. 6-9 month
Connection of such skin lesions can be present during secondary syphilis:
A. Roseola and papule
B. Roseola, papule and pustule
C. Papule and pigmentation
D. Alopecia and pigmentation
E. *All are correct
Secondary syphilis can be present by all skin lesions, except:
A. Roseola and papule
B. *“Chard chancre”
C. Roseola, papule and pustule
D. Papule and pigmentation
E. Alopecia and pigmentation
Immunity in case of syphilis is:
A. Congenital
B. Gain
C. Non-sterility
D. Infective
E. *Non-sterility, infective
“Hard chancre” is:
A. Oval or around
B. Well demarcated
C. Have indurate base
D. Have non-inflammative base
E. *All are correct
“Hard chancre” is characterized by all, except:
A.
B.
C.
D.
E.
439.
A.
B.
C.
D.
E.
440.
A.
B.
C.
D.
E.
441.
A.
B.
C.
D.
E.
442.
A.
B.
C.
D.
E.
443.
A.
B.
C.
D.
E.
444.
A.
B.
C.
D.
E.
445.
A.
B.
C.
D.
E.
446.
A.
B.
C.
D.
Oval or around
*Soft
Well demarcated
Have indurate base
Have non-inflammative base
Regional lymphadenitis in case of primary syphilis is:
Classically rubbery
Painless
Bilateral
*All are correct
All are wrong
Identification of Treponema pallidum in lesions means:
Wasserman reaction
Tzank smear
*Dark-field microscopy
Direct visualization
Diascopy
Identification of Treponema pallidum in lesions means:
*Direct fluorescent antibody
Wasserman reaction
Tzank smear
Direct visualization
Diascopy
Treponemal serologic tests of syphilis are:
TP-PA
FTA-ABS
EIA
*All are correct
All are wrong
Nontreponemal serologic tests of syphilis are all, except:
VDRL
RPR
TRUST
USR
*FTA-ABS
Nontreponemal Serologic tests of syphilis are all, except:
*TP-PA
VDRL
RPR
TRUST
USR
Nontreponemal Serologic tests of syphilis are all, except:
VDRL
*EIA
RPR
TRUST
USR
Jarisch-Herxheimer reaction is:
*Self-limited reaction to anti-treponemal therapy
Self-limited reaction to antifungal therapy
Self-limited reaction to antiviral therapy
All are correct
447.
448.
449.
450.
451.
452.
453.
454.
455.
E. All are wrong
“Hard chancre” and regional lymphadenitis are characterized for:
A. Secondary syphilis
B. *Primary syphilis
C. Congenital syphilis
D. Late syphilis
E. All are correct
“Venera necklace” is combination of hyperpigmentation and depigmentation in:
A. *Secondary syphilis
B. Erythrasma
C. Pituriasis versicolor
D. Psoriasis
E. All are wrong
Clinical improvement of primary syphilis are:
A. Incubation period 3-4 weeks
B. Absence of subjective sensations
C. Button-like, painless ulcer
D. Presence of regional lymphadenitis
E. *All are correct
Treponema pallidum is causative agent of:
A. Gonococci infection
B. *Syphilis
C. Chancroid
D. Favus
E. All are wrong
Jarisch-Herxheimer is an allergic reaction to:
A. Antiviral drugs
B. *Penicillin
C. Antifungal drugs
D. Steroids
E. B-blockers
Clinical improvement of primary syphilis are all, except:
A. Incubation period 3-4 weeks
B. *Presence of subjective sensations
C. Button-like, painless ulcer
D. Presence of regional lymphadenitis
E. Wassermann reaction is positive
Clinical improvement of primary syphilis are :
A. Absence of regional lymphadenitis
B. Presence of subjective sensations
C. Incubation period 3-4 month
D. *All are wrong
E. Large amount of scales
“ Snail-track” are:
A. *Superficial ulcers of the mucous membranes of the oral cavity
B. Deep ulcers of the mucous membranes of the oral cavity
C. Superficial and deep ulcers of the mucous membranes of the oral cavity
D. All are correct
E. All are wrong
“Snail-track” and “Venera necklace” are characterized for:
A. Primary syphilis
B. *Secondary syphilis
456.
457.
458.
459.
460.
461.
462.
463.
C. Late syphilis
D. Congenital syphilis
E. All are wrong
Treatment of syphilis may trigger:
A. Balzer test
B. *Jarisch-Herxheimer reaction
C. Psoriatic phenomenon
D. Woronow wring
E. All, except Jarisch-Herxheimer reaction
The Spirochetes means:
A. Treponema
B. Leptospira
C. Borrelia
D. *All are correct
E. All are wrong
Primary syphilis means:
A. Red or brown rash on skin
B. *Appearance of hard chancre at site of inoculation
C. Both are correct
D. Sore throat
E. Fever
What types of antibodies can pass through placenta?
A. Ig A
B. Ig M
C. *Ig G
D. Ig D
E. Ig E
What types of antibodies can pass through placenta?
A. Ig M
B. Ig D
C. Ig A
D. *Ig G
E. All are correct
In what period of pregnancy treponema pallidum can cross the placenta?
A. *After 17-18 weeks of pregnancy.
B. After 20-28 weeks of pregnancy.
C. After 30-38 weeks of pregnancy.
D. After 40-48 weeks of pregnancy.
E. After 7-8 weeks of pregnancy
Possible results of pregnancy, according to the time of affection of pregnant are:
A. Abortion
B. Stillbirth
C. Birth of child with congenital syphilis
D. Fetus can affected from mother in delivery
E. *All are correct
Diagnosis of congenital syphilis includes all, except:
A. Mother’s anamnesis
B. *Tzanck smear
C. An examination of the placenta and umbilical cord
D. Assessment of typical characteristics of congenital syphilis in different growing
groups
464.
465.
466.
467.
468.
469.
470.
471.
472.
E. Treponema pallidum can be demonstrated in a smear from skin lesions with moist
surface.
Mother’s anamnesis in diagnosis of congenital syphilis means:
A. Abortion
B. Stillbirth
C. Burning of macerated fetus
D. High children lethality after burning
E. *All are correct
Diagnosis of congenital syphilis includes all, except:
A. Clinical assessment of parents and others family members
B. *Balzer test
C. Serological tests of child, his parents and others family children
D. Consultations of others physicians
E. Modern characteristic features of congenital syphilis
Syphilis of placenta is characterized by all features, except:
A. Placenta is edematous
B. Placenta is pale
C. Placenta is greasy
D. Placenta is bulky
E. *Placenta is white
Congenital syphilis can be:
A. Early
B. Late
C. Primary
D. Secondary
E. *Early and late
Congenital syphilis can be:
A. *Early and late
B. Early and secondary
C. Primary and secondary
D. Primary and late
E. Early and secondary
Syphilis of the fetus is characterized by all features, except:
A. Skin of the fetus is macerated.
B. Fetus sometimes dead
C. Little weight of the fetus
D. Enlargement of liver and spleen of the fetus
E. *Enlargement of the fetus weight
Syphilitic chorioretinitis is characterized by such symptom:
A. *“Salt -and -pepper”
B. “Moth eaten”
C. “Snail-track”
D. “Venera necklace”
E. “Christmas tree”
Syphilitic chorioretinitis is characterized by such symptom:
A. “Herald patch”
B. “Christmas tree”
C. *“Salt -and -pepper”
D. “Moth eaten”
E. “Snail-track”
“Salt -and -pepper” symptom is characterized for:
A. Primary syphilis
B.
C.
D.
E.
473.
A.
B.
C.
D.
E.
474.
A.
B.
C.
D.
E.
475.
A.
B.
C.
D.
E.
476.
A.
B.
C.
D.
E.
477.
A.
B.
C.
D.
E.
478.
A.
B.
C.
D.
E.
479.
A.
B.
C.
D.
E.
480.
A.
B.
C.
D.
E.
Secondary syphilis
*Congenital syphilis
Late syphilis
All are correct
Symptom of “salt -and -pepper” is found during assessment of:
*Eye ground
Hair of the head
Mucous membrane of the oral cavity
Mucous membrane of the genital organs
Placenta
In case of late syphilitic ophtalmopathic serological tests are positive in:
*100%
80%
75%
50%
25%
Hutchinson’s teeth are characterized for:
Early congenital syphilis
*Late congenital syphilis
Primary syphilis
Secondary syphilis
Primary syphilis and secondary syphilis
Syphilitic pemphigus is characterized for:
Late congenital syphilis
*Early congenital syphilis
Primary syphilis
Secondary syphilis
Primary and secondary syphilis
Syphilitic rhinitis is characterized for:
Primary syphilis
Secondary syphilis
*Early congenital syphilis
Late congenital syphilis
Primary and late congenital syphilis
Syphilitic chorioretinitis is characterized for:
*Early congenital syphilis
Primary syphilis
Late congenital syphilis
Secondary syphilis
Primary and early congenital syphilis
“Pneumonia alba” is characterized for:
Gonococci infection
*Early congenital syphilis
Primary syphilis
Late congenital syphilis
Secondary syphilis
Eight nerve deafness is characterized for:
Primary syphilis
Gonococci infection
Secondary syphilis
Early congenital syphilis
*Late congenital syphilis
481.
Syphilitic pemphigus is characterized by:
A. *Vesicles and erosions on the front of the palms and soles
B. Maculae and papules on the trunk
C. Vesicles and erosions on the trunk
D. Maculae and papules on the front of the palms and soles
E. Patches on the front of the palms and soles
482.
Syphilitic pemphigus is characterized by:
A. Patches on the front of the palms and soles
B. Maculae and papules on the trunk
C. Vesicles and erosions on the trunk
D. *All are wrong
E. Maculae and papules on the front of the palms and soles
483.
Gumma is characterized for:
A. Primary syphilis
B. Secondary syphilis
C. Early congenital syphilis
D. Late congenital syphilis
E. *Late syphilis
484.
Condiloma lata are characterized for:
A. Early congenital syphilis
B. *Secondary and congenital syphilis
C. Late congenital syphilis
D. Primary syphilis
E. Secondary syphilis
485.
Examination of the placenta and umbilical cord we use to diagnose:
A. Primary syphilis
B. Primary and secondary syphilis
C. *Congenital syphilis
D. Late syphilis
E. Secondary syphilis
486.
Diffuse infiltration in patient with congenital syphilis appeared within the first
weeks or month of birth around all zones, except:
A. Angles of mouth
B. Lips
C. *Scalp
D. Buttocks
E. Palms
487.
Diffuse infiltration in patient with congenital syphilis appeared within the first
weeks or month of birth around:
A. Angles of mouth
B. Lips
C. Buttocks
D. Palms
E. *All are correct
488.
Syphilitic rhinitis is characterized by all, except:
A. Difficult of nasal breathing
B. Presence of highly infectious purulent-serous and hemorrhagic nasal discharges
(snuffles)
C. *Without future deformation of the nose
D. Hoarse breathing
E. Dislocation of nasal septum
489.
Osteochondritis - the main specific and characteristic changes of the bones in
patient with congenital syphilis in children:
A. *Before the age of 1 year
B. After the age of 1 year
C. After the age of 2 year
D. After the age of 3 year
E. After the age of 4 year
490.
Late congenital syphilis can appear in patient with:
A. 1-4 years old
B. *5-20 years old
C. 1-2 years old
D. 1-2 years old
E. All are wrong
491.
Late congenital syphilis is characterized by specific changes in the:
A. Skin
B. Bones
C. Mucous membranes
D. Inner organs
E. *All are correct
492.
Late congenital syphilis is characterized by all specific changes, except:
A. Nerve system
B. Sensitive organs
C. *Hard chancre
D. Bones
E. Mucous membranes
493.
Late congenital syphilis is characterized by all specific changes, except:
A. Mucous membranes
B. *Syphilitic pemphigus
C. Nerve system
D. Sensitive organs
E. Mucous membranes
494.
Hutchinson’s triad means:
A. Interstitial keratitis
B. Hutchinson’s teeth
C. Eight nerve deafness
D. *All are correct
E. All are wrong
495.
Hutchinson’s triad means all, except:
A. Eight nerve deafness
B. Interstitial keratitis
C. Hutchinson’s teeth
D. *Syphilitic pemphigus
E. All are wrong
496.
Hutchinson’s triad means all, except:
A. Hutchinson’s teeth
B. *Osteochondritis
C. Eight nerve deafness
D. Interstitial keratitis
E. All are wrong
497.
Mothers anamnesis (abortion, stillbirth, burning of macerated fetus, high children
lethality after burning) we use to diagnose:
A. Primary syphilis
B.
C.
D.
E.
*Early congenital and late congenital syphilis
Late congenital syphilis
Secondary syphilis
Primary and secondary syphilis
498.
Syphilis of placenta is caused by:
A. Microsporum
B. Chlamidia trachomatis
C. Ureaplasma urealyticum
D. Trichomonas vaginalis
E. *Treponema pallidum
499.
Syphilis of the fetus is caused by:
A. Trichomonas vaginalis
B. *Treponema pallidum
C. Chlamidia trachomatis
D. Ureaplasma urealyticum
E. Dermatophytes
500.
Benzathine pehicillin (extencillin) is:
A. *Antibiotic
B. Corticosteroids
C. Antiviral agent
D. Antihistamines
E. Antifungal drug
501.
Tetracycline is:
A. Antiviral agents
B. *Antibiotic
C. Corticosteroids
D. Antihistamines
E. Antifungal drug
502.
Bicillin-1 is:
A. Antifungal drug
B. Antiviral agents
C. *Antibiotic
D. Corticosteroids
E. Antihistamines
503.
Bicillin-3 is:
A. Antihistamines
B. Antifungal drug
C. Antiviral agents
D. Corticosteroids
E. *Antibiotic
504.
Bicillin-5 is:
A. *Antibiotic
B. Antifungal drug
C. Antiviral agents
D. Corticosteroids
E. Antihistamines
505.
Congenital syphilis of children before the age of 1 year is characterized by all,
except:
A. Syphilitic pemphigus
B. *Hard chancer
C. Diffuse infiltration
D. Syphilitic rhinitis
E. Ostechondritis
506.
Preventive treatment of syphilis means use of all drugs, except:
A. Bicillin-1 (1 200 000 units)
B. Tetracycline, erythromycin
C. Bicillin-3 (1 800 000 units)
D. *Antiviral agents
E. Bicillin-5 (1 500 000 units)
507.
Specific fuciform radial atrophic scars (Robinson-Furnye) are results of:
A. *Diffuse infiltration
B. Syphilitic pemphigus
C. Syphilis of placenta
D. Syphilis of the fetus
E. Syphilitic rhinitis
508.
Diffuse infiltration in patient with congenital syphilis appeared:
A. Recently after birth
B. *Within the first weeks or month of birth
C. 10 year after birth
D. 14 year after birth
E. Never appeared
509.
The spirochetes means:
A. Candida
B. *Leptospira
C. Streptococci
D. All are correct
E. All are wrong
510.
The spirochetes means:
A. *Treponema pallidum
B. Dermatophytes
C. Candida
D. All are correct
E. All are wrong
511.
The spirochetes means:
A. Dermatophytes
B. Streptococci
C. *Borelia
D. All are correct
E. All are wrong
512. A condition of the teeth characteristic of congenital syphilis are:
A. Accessional teeth
B. *Hutchinson’s teeth
C. Successional teeth
D. Temporary teeth
E. Deciduous teeth
513. International classification of late congenital syphilis means all, except:
A. Late syphilitic ophtalmopathic
B. Late congenital neurosyphilis
C. Others form of late congenital syphilis, late congenital neurosyphilis, late
syphilitic ophtalmopathic
D. *Chard chancre
E. Others form of late congenital syphilis
514. Others form of late congenital syphilis means all, except:
A. Affection of bones
515.
516.
517.
518.
519.
520.
521.
522.
B. *Chard chancre
C. Hutchinson’s teeth
D. Affection of ears (suppurative otitis media)
E. Hutchinson’s triad
Others form of late congenital syphilis means all, except:
A. Affection of bones
B. Affection of ears (suppurative otitis media)
C. Hutchinson’s teeth
D. *Accessional teeth
E. Hutchinson’s triad
Others form of late congenital syphilis means all, except:
A. *Deciduous teeth
B. Affection of bones
C. Affection of ears (suppurative otitis media)
D. Hutchinson’s teeth
E. Hutchinson’s triad
Others form of late congenital syphilis means all, except:
A. Hutchinson’s teeth
B. Affection of bones
C. Affection of ears (suppurative otitis media)
D. Hutchinson’s triad
E. *Successional teeth
Non-treponemal tests are all, except:
A. Flocculation tests
B. Venereal disease reaginic laboratory test (V.D.R.L.)
C. Rapid plasma reaginic test
D. Complement fixation test
E. *Treponema pallidum immobilization test (TPI)
Treponemal tests are all, except:
A. Treponema pallidum immobilization test (TPI)
B. Reiter treponeme complement fixation test
C. Fluorescent antibody test (FTA-ABS)
D. *Venereal disease reaginic laboratory test (V.D.R.L.)
E. Treponema pallidum haemagglutination test (TPHA)
What drugs we must prescribe for patient with chronic Chlamydia urethritis?
A. Corticosteroids
B. Anti inflammation drugs
C. Antifungal drugs
D. *Antibiotics
E. Antiviral drugs
Drugs of choice for patients with Chlamydia infection are:
A. Antiviral drugs
B. Antihistamines
C. *Antibiotics
D. Anti inflammation drugs
E. Corticosteroids
The main complications of Chlamydia infection in male are all, except:
A. Prostatitis
B. Epididymitis
C. *Hepatitis
D. Infertility
E. Reiter syndrome
523.
524.
525.
526.
527.
528.
529.
530.
531.
Causative agent of congenital syphilis is :
A. *Bacteria
B. Fungi
C. Virus
D. Mite
E. All are wrong
Causative agent of Chlamydia infection is:
A. Fungi
B. *Bacteria
C. Virus
D. Both are wrong
E. Mite
Causative agent of Gonorrhea is:
A. Virus
B. Fungi
C. *Bacteria
D. Mite
E. All are wrong
Causative agent of Trichomoniasis is:
A. Virus
B. Fungi
C. Mite
D. *Bacteria
E. All are wrong
Causative agent of genital herpes is:
A. *Virus
B. Fungi
C. Bacteria
D. All are wrong
E. Mite
Neisseria gonorrhea survives in all, except:
A. Cervix
B. Mouth
C. *Scalp
D. Throat
E. Eyes
Neisseria gonorrhea survives in all, except:
A. Rectum
B. Throat
C. Eyes
D. *Nails
E. Cervix
You can not catch gonorrhea from all, except:
A. *Vaginal sex
B. Simple kissing
C. Sharing baths
D. Towels
E. Cups
You can not catch gonorrhea from all, except:
A. Toilet seats
B. Cups
C. Simple kissing
532.
533.
534.
535.
536.
537.
538.
539.
540.
D. *Anal sex
E. Towels
Non-specific symptoms of Gonococci cervicitis are all, except:
A. Abnormal vaginal discharge
B. Intermenstrual bleeding
C. Dysuria
D. Lower abdominal pain
E. *Purulent cervical discharge
Clinical manifestations of disseminated gonococci infection are all, except:
A. Skin lesions
B. *Chard chancre
C. Arthralgias
D. Tenosynovitis
E. Arthritis
Gonorrhea is often treated with all drugs, except:
A. Cefixime
B. Ceftnaxone
C. Ciprofloxacin
D. Ofloxacin
E. *Azeleic Acid
Gonorrhea is often treated with all drugs, except:
A. Tetracycline
B. Ceftnaxone
C. Ofloxacin
D. *B-blocker
E. Cefixime
Gonococcal infections in women & men means all, except:
A. *Onychomycosis
B. Proctitis
C. Pharyngeal infections
D. Conjunctivitis
E. Disseminated Gonococcal Infection
Gonococcal infections in women & men means all, except:
A. Pharyngeal infections
B. *Pemphigus
C. Proctitis
D. Conjunctivitis
E. Disseminated gonococcal infection
Gonococcal infections in women & men means all, except:
A. Disseminated gonococcal infection
B. Pharyngeal infections
C. Infertility
D. *Presence of scales
E. Conjunctivitis
Gonococcal infections in women & men means all, except:
A. *Kerion
B. Disseminated gonococcal infection
C. Pharyngeal infections
D. Conjunctivitis
E. Proctitis
Gonococcal Infections in women & men means all, except:
A. Conjunctivitis
541.
542.
543.
544.
545.
546.
547.
548.
B. Disseminated gonococcal Infection
C. Pharyngeal infections
D. *Favus
E. Proctitis
Diagnostic methods of gonococcal infection include all, except:
A. Gram stain smear
B. Culture
C. *Tzanck smear
D. Antigen Detection Tests: EIA & DFA
E. Nucleic Acid Detection Tests
Neisseria gonorrhea means:
A. *Gram negative intracellular diplococci
B. Gram positive intracellular diplococci
C. Gram negative intracellular streptococci
D. Gram positive intracellular streptococci
E. All are wrong
Ophthalmia neonatorum prophylaxis means:
A. *Silver nitrate 1% aqueous solution topical x 1
B. Erythromycin 0.5% ointment topical x 2
C. Tetracycline 1% ointment topical x 4
D. Erythromycin 2.5% ointment topical x 2
E. Tetracycline 10% ointment topical x 4
Ophthalmia neonatorum prophylaxis means:
A. Tetracycline 1% ointment topical x 4
B. Silver nitrate 10% aqueous solution topical x 4
C. *Erythromycin 0.5% ointment topical x 1
D. Erythromycin 2.5% ointment topical x 2
E. Tetracycline 10% ointment topical x 4
Chlamydia unusual presentations mean all, except:
A. Trachoma
B. Inclusion conjunctivitis
C. *Urethritis
D. Lymphogranuloma venereum
E. Trachoma, inclusion conjunctivitis, lymphogranuloma venereum
Prophylaxis of ophthalmia neonatorum means:
A. *Erythromycin 0.5% ointment topical x 1
B. Tetracycline 1% ointment topical x 4
C. Silver nitrate 10% aqueous solution topical x 4
D. Erythromycin 2.5% ointment topical x 2
E. Tetracycline 10% ointment topical x 4
Palmar Syphilides of late syphilis are characterized by all, except:
A. Occur on the palms and soles
B. They are scaly
C. Have great moister surface
D. They are nodular or gummatous
E. *Presence of “Wickham striae”
Diagnostic methods of Gonococcal infection include all, except:
A. Clinical examination
B. Gram stain
C. *Venereal disease reaginic laboratory test (V.D.R.L.)
D. Culture
E. Nucleic acid probes
549.
550.
551.
552.
553.
554.
555.
556.
557.
Diagnostic methods of Gonococcal infection include all, except:
A. Gram stain
B. Clinical examination
C. Nucleic acid probes
D. *Treponema pallidum immobilization test (TPI)
E. Culture
Diagnostic methods of syphilis include all, except:
A. Clinical examination
B. Culture
C. *Dark field microscopy
D. Serology screening test
E. Serology confirmatory test
Diagnostic methods of syphilis include all, except:
A. Dark field microscopy
B. Clinical examination
C. Serology screening test
D. *Gram stain
E. Serology confirmatory test
Gonococcal infection diagnosis means all, except:
A. Clinical examination
B. Gram stain
C. *Venereal disease reaginic laboratory test (V.D.R.L.)
D. Culture
E. Nucleic acid probes
Late stage syphilis principal clinical manifestations are all, except:
A. Destructive gummas
B. Aortic valve injury
C. *Chard chancre
D. Dementia
E. Tabes dorsalis
Late stage syphilis principal clinical manifestations are all, except:
A. Destructive gummas
B. Aortic valve injury
C. Dementia
D. *“Pneumonia alba”
E. Tabes dorsalis
Congenital syphilis clinical manifestations are all, except:
A. Fetal death
B. *Destructive gummas
C. Growth restriction
D. Multiple anomalies
E. Immediately apparent at birth
Congenital syphilis clinical manifestations are all, except:
A. Multiple anomalies
B. Fetal death
C. *Tabes dorsalis
D. Immediately apparent at birth
E. Growth restriction
Genital Herpes possible complications are all, except:
A. Meningitis
B. *“Pneumonia alba”
C. Narrowing of the urethra due to scarring
558.
559.
560.
561.
562.
563.
564.
565.
566.
D. Increased risk of becoming infected with HIV
E. Transfer of virus from mother to infant in childbirth
Culture for Gonorrhea means all, except:
A. Not requires selective media with antibiotics
B. *Sensitive to oxygen and cold temperature
C. Stable to oxygen and sensitive to cold temperature
D. Sensitive to oxygen and stable to cold temperature
E. All are correct
Sweat glands are classified into:
A. Anagen and Telogen
B. *Eccrine and Apocrine
C. Collagen and Apocrine
D. Collagen and Telogen
E. All are wrong
Redness of the skin accompanied by inflammation is known as:
A. Furuncle
B. Follicle
C. *Erythema
D. Milia
E. Hive
The darkening of the skin due to excessive pigment in the skin is called:
A. Impetigo
B. *Hyperpigmentation
C. Keratosis
D. Wheal
E. All are wrong
Enlargement of the nose is known as:
A. *Rhinophyma
B. Hypertrophy
C. Rubella
D. Petechia
E. Nodule
Which of the following conditions refers to Hemangioma?
A. Peeling of skin due to moisture
B. A* harmless tumor consisting of a mass of blood vessels
C. Blisters on the skin
D. Patchy light brown spots on skin
E. Patchy light re spots on skin
Blisters (containing clear fluid) on the skin are called:
A. *Bullae
B. Barnacles
C. Furuncles
D. Carbuncles
E. Nodule
Athlete's Foot or Tinea Pedis is a:
A. *Fungal infection
B. Hair infection
C. Scrotal infection
D. Bacterial infection
E. Viral infection
Lack of melanin in the skin, hair, and eyes is called:
A. Angioma
567.
568.
569.
570.
571.
572.
573.
574.
B. *Albinism
C. Urticaria
D. Blepharitis
E. Nevus
Vitiligo is a condition in which, skin:
A. Becomes allergic
B. Form rashes
C. *Loses its colour
D. Itches
E. Becomes infected
All the following skin infections are bacterial infections except one:
A. Eryspilas
B. Impetigo
C. Boils
D. Ecthyma
E. *Scabies
What is the function of merkel cells
A. Synthesize melanin
B. *For immune reaction
C. For transducers of fine touch
D. All are correct
E. All are wrong
A person who has acne can have any of these blemishes, except:
A. Blackheads
B. Whiteheads
C. Papules
D. Pustules (what many people call pimples)
E. *Hives
A person who has acne can have any of these blemishes, except:
A. Cysts
B. Nodules
C. *Hives
D. Blackheads
E. Whiteheads
Guttate psoriasis is characterized by all, except:
A. Small, red spots (usually on the trunk, arms, and legs but can appear on the scalp,
face, and ears)
B. Spots can show up all over the skin.
C. Spots often appear after an illness, especially strep throat
D. Spots may clear up in a few weeks or months without treatment
E. *A circumscribed, fluid containing, elevated lesion of the skin
Plaque psoriasis is characterized by all, except:
A. Raised reddish plagues on the skin called plaque
B. Plagues may be covered with a silvery scale
C. Plagues can appear anywhere on the skin
D. *Spots often appear after an illness, especially strep throat.
E. Most plagues appear on the knees, elbows, lower back, and scalp
Guttate psoriasis is characterized by all, except:
A. *Plagues vary in size and can appear as separate patches or join together to cover
a large area
B. Small, red spots (usually on the trunk, arms, and legs but can appear on the scalp,
face, and ears)
575.
576.
577.
578.
579.
580.
581.
582.
C. Spots can show up all over the skin.
D. Spots often appear after an illness, especially strep throat
E. Spots may clear up in a few weeks or months without treatment
Plaque psoriasis is characterized by all, except:
A. *Spots may appear where the person had plaque psoriasis
B. Raised reddish plagues on the skin called plaque
C. Plague may be covered with a silvery scale
D. Plague can appear anywhere on the skin
E. Most plagues appear on the knees, elbows, lower back, and scalp
Pustular psoriasis is characterized by all, except:
A. Skin red, swollen, and dotted with pus-filled bumps
B. Bumps usually appear only on the palms and soles
C. *Most plagues appear on the knees, elbows, lower back, and scalp
D. Soreness and pain where the bumps appear
E. Pus-filled bumps will dry, and leave behind brown dots and/or scale on the skin.
Inverse psoriasis means presence of lesions in all places, except:
A. Armpits
B. Around the groin
C. *Elbows
D. Genitals
E. Buttocks
Inverse psoriasis means presence of lesions in all places, except:
A. *Knees
B. Armpits
C. Around the groin
D. Genitals
E. Buttocks
Erythrodermic psoriasis is also called:
A. *Exfoliative psoriasis
B. Flexural psoriasis
C. Intertriginous psoriasis
D. Plaque psoriasis
E. Guttate psoriasis
Erythrodermic psoriasis is characterized by all, except:
A. Skin looks like it is burned
B. Most (or all) of the skin on the body turns bright red
C. Body cannot maintain its normal temperature of 98.6° F. Person gets very hot or
very cold
D. Heart beats too fast
E. *Raised reddish patches on the skin called plaque
Erythrodermic psoriasis is characterized by all, except:
A. Intense itching
B. Intense pain
C. *Chard chancre
D. Skin looks like it is burned.
E. Most (or all) of the skin on the body turns bright red
Inverse psoriasis is also called:
A. Exfoliative psoriasis
B. Erythrodermic psoriasis
C. *Intertriginous psoriasis
D. Plaque psoriasis
E. Guttate psoriasis
583. Inverse psoriasis is also called:
A. *Flexural psoriasis
B. Exfoliative psoriasis
C. Erythrodermic psoriasis
D. Plaque psoriasis
E. Guttate psoriasis
584. Plaque psoriasis is also called:
A. Flexural psoriasis
B. Exfoliative psoriasis
C. *Psoriasis vulgaris
D. Pustular psoriasis
E. Guttate psoriasis
585. What lesions many people call pimples?
A. Blackheads
B. Whiteheads
C. Papules
D. *Pustules
E. Cysts
586. Types of rosacea are all, except:
A. Erythematotelangiectatic rosacea
B. *Late congenital rosacea
C. Papulopustular rosacea
D. Phymatous rosacea
E. Ocular rosacea
587. Signs of tinea versicolor are all, except:
A. The spots are lighter than the surrounding skin.
B. *Skin red, swollen, and dotted with pus-filled bumps
C. The color of the spots can be white, pink, salmon, red, tan, or brown
D. The spots can appear anywhere on the body
E. Spots can be dry and scaly
588. Signs of tinea versicolor are all, except:
A. Skin may itch where the spots appear
B. Spots become more noticeable as the skin tans. The yeast prevents the skin from
tanning
C. Spots grow slowly
D. As the yeast grows, the spots can combine and form patches of lighter (or darker)
skin
E. *Skin red, swollen, and dotted with pus-filled bumps
589. The most common signs of hives are all, except:
A. Slightly raised, pink or red swellings on the skin
B. Welts that occur alone or in a group, or connect over a large area
C. Skin swelling that subsides
D. Skin swelling goes away within 24 hours at one spot
E. *Blackheads
590. The most common signs of hives are all, except:
A. *Whiteheads
B. Slightly raised, pink or red swellings on the skin
C. Welts that occur alone or in a group, or connect over a large area
D. Skin swelling that subsides
E. Skin swelling goes away within 24 hours at one spot
591. Mouth (oral lichen planus) means all, except:
A. Patches of tiny white dots and lines that can look like lace
592.
593.
594.
595.
596.
597.
598.
599.
B. Redness and swelling
C. Peeling on the gums
D. Painful sores (can hurt or burn)
E. *Skin looks like it is burned
Nails lichen planus means all, except:
A. Ridges or grooves on the nails
B. Splitting or thinning
C. Temporary loss of nails
D. Permanent loss of nails
E. *“Oil drops”
Scalp (lichen planopilaris) means al, except:
A. Redness and irritation
B. Tiny bumps
C. *Scars appear quickly
D. Thinning hair or patches of hair loss
E. Scars appear slowly
Scabies in children means all, except:
A. The scabies rash can cover most of the body
B. Even a child’s palms, soles, and scalp can be infested with mites
C. The rash often appears on the palms
D. *Widespread soft chancre
E. The rash often appears on the soles
Crusted scabies means all, except:
A. Is a severe form of scabies
B. *The rash often appears on the palms
C. Crusted scabies develops in people who have a weak immune system
D. Crusted scabies develops when the person’s body cannot develop any resistance
to the mites
E. A common sign of crusted scabies is widespread crusts on the skin
Shingles also called:
A. *Herpes zoster
B. Herpes labialis
C. Cytomegalovirus
D. Chicken pox
E. All are wrong
Signs and symptoms of Shingles are all, except:
A. Rash
B. Blisters
C. Pain
D. Flu-like symptoms
E. *Hives
“Mother patch” is a large patch characterized for:
A. *Pityriasis rosea
B. Tinea versicolor
C. Shingle
D. Lichen planus
E. Exfoliative psoriasis
Which of the following is an extracutaneous manifestation of psoriasis?
A. Lymphadenopathy
B. *Arthritis
C. Schizophrenia
D. Eczema
E. Gottron's nodules
600. Large flat hyperpigmented lesion is an example of a:
A. *Macule
B. Plaque
C. Cyst
D. Bulla
E. Nodule
601. Small fluid-filled lesion on an erythematous base is a:
A. Bulla
B. *Vesicle
C. Papule
D. Macule
E. Cyst
602. A raised itchy large lesion that when you came back in the room from looking it up was
GONE! It is a:
A. *Hive
B. Wheal
C. Patch
D. Plaque
E. Nodule
603. The slide you’ve been waiting for and don’t deny it! This fellow has an itchy heel which
he keeps rubbing. This has caused the skin to thicken and the skin lines to accentuate. This is a
secondary skin lesion. It is:
A. Excoriation
B. A scar
C. Atrophy
D. *Lichenification
E. Vesicle
604. It is a thickened heaped up lesion whereas if you palpate it feels thin, slightly indented
and there is an absence of skin lines. What are they respectively?
A. *Keloid and atrophy
B. Scar and atrophy
C. Scar and lichenification
D. Keloid and lichenification
E. All are wrong
605. What is the difference between HIV and AIDS?
A. HIV is a virus and AIDS is a bacterial disease
B. There is no difference between HIV and AIDS
C. *HIV is the virus that causes AIDS
D. All are wrong
E. HIV is a virus and AIDS is a parasitogenic disease
606. HIV can make a person ill because:
A. It makes a person lose weight very suddenly
B. It reduces the body's core temperature
C. *It attacks the immune system
D. All are correct
E. All are wrong
607. HIV is believed to have evolved from a similar virus found in which animal?
A. Baboon
B. *Chimpanzee
C. Elephant
D. Guinea pig
E. Cats
608. Which of these drugs is most commonly used on its own to reduce mother-to-child HIV
transmission?
A. Aspirin
B. Tenofovir
C. Paracetamol
D. *Nevirapine
E. All are wrong
609. Which normally rare cancer is often associated with AIDS?
A. Squamous Cell Carcinoma
B. Mesothelioma
C. *Kaposi's Sarcoma
D. All are correct
E. All are wrong
610. What is the medical term for a condition, known as “hives”?
A. Impetigo
B. *Urticaria
C. Bulla
D. Patch
E. Papule
611. How does urticaria usually manifest itself on one’s skin?
A. *With pink weal’s with pale center
B. With red and scaly skin plaque
C. With green round dots on the skin
D. With narrow skin scratches resembling letter “Z”
E. All are wrong
612. Urticaria wheals are known to be able to vanish on their own at one part of the body and
appear later on another. How do you think: how long do individual wheals usually last?
A. *Between 1 and 24 hours
B. Between 10 and 60 minutes
C. Between 1 and 3 days
D. Between 1 and 3 weeks
E. Between 1 and 3 months
613. It is known that hives can be either acute or chronic. Which of the following situations
represent a case of chronic hives?
A. Hives appear in response to certain medication
B. Hives do not respond to topical treatment
C. Hives affect more than 50 % of skin surface
D. Hives last more than 6 weeks
E. *All are right
614. What are the most common triggers of short-termed or acute urticaria?
A. Infections
B. Drugs and foots
C. Pollen and dust
D. Insect bites
E. *All the mentioned
615. It is known that even cold or hot weather may cause hives outbreak. What other
“specific” factors may trigger urticaria?
A. *Touching the skin of predisposed person
B. Watching certain TV shows
C. Doing certain types of jobs, like preparing a monthly report
D. None of the above
E. All the mentioned
616. Statistics says that about 80% of all hives cases are idiopathic. What does this mean?
A. It means the disorder is difficult to cured
B. It means the disorder cannot be properly diagnosed
C. *It means no exact cause of the disorder can be identified
D. It means the disorder is not contagious
E. All are right
617. What body substance is said to be the major participant in the process of hives
development?
A. *Histamine
B. Serotonin
C. Blood plasma
D. Adrenaline
E. All the mentioned
618. How often should one take long-acting antihistamines, as Zyrtec, Allegra, Hismanal, in
order to relieve the symptoms of hives?
A. Every 3-6 hours
B. Every 6-12 hours
C. *Every 12-24 hours
D. Every 24-48 hours
E. All are wrong
619. What side effect, associated with antihistamines use, is considered to be the most
bothersome?
A. *Drowsiness
B. Headache
C. Sexual dysfunction
D. Weight again
E. All the mentioned
620. Psoriasis can also cause inflammation of the joints, which is known as:
A. Lymphadenopathy
B. Erythema nodosum
C. *Psoriatic arthritis
D. Cellulitis
E. All are wrong
621. A non-contagious rash of thick plaques and silvery scales usually affects the scalp,
elbows, knees, and lower back. is known as:
A. *Psoriasis
B. Tinea infection
C. Pituriasis versicolor
D. All are correct
E. All are wrong
622. Small, painful, fluid-filled blisters on the mouth or nose, cold sores are caused by the:
A. Type 2 herpes simplex virus
B. *Type 1 herpes simplex virus
C. Varicella Zoster virus
D. Cytomegalovirus
E. All are correct
623. Blackhead or open comedones often seen on the:
A. Face
B. Chest
C. Back
D. *All are correct
E. All are wrong
624. A whitehead or closed comedones often seen on the:
A. Face
B. Chest
C. Back
D. *All are correct
E. All are wrong
625. Which of the following is the most common adverse reaction to transdermal drug
delivery systems?
A. Allergic contact dermatitis
B. Anaphylaxis
C. *Irritant contact dermatitis
D. Urticaria
E. Viral superinfection
626. Acantholysis is seen in:
A. Pemphigus vulgaris
B. Darier’s disease
C. Pemphigus foliaceus
D. *All are correct
E. All are wrong
627. Syphilis is also called as the:
A. Silent disease
B. Silent killer
C. *The great imitator
D. Contagious disease
E. All are wrong
628. Which is the first symptom that is manifested by this disease?
A. *A sore called "chancre"
B. Fever
C. Swollen lymph nodesd
D. Pain in the affected area
E. All are wrong
629. The painless sore called "chancre":
A. Is round, firm, and small ulcer
B. Is found in the penis, the vagina or the rectum
C. Disappears within a few weeks regardless of whether treatment is obtained
D. *All are correct
E. All are wrong
630. What are some of the symptoms that are manifest in the secondary phase of syphilis?
A. Flu-like symptoms
B. Whole-body rash
C. Patchy hair loss
D. *All are correct
E. All are wrong
631. Which of the diseases listed is NOT an STD:
A. Ectoparasitic infection
B. *Hand-foot-mouth disease
C. Urethritis
D. Chancroid
E. Scabies
632. Which of the following is a possible symptom of an STD?
A. Bumps, sores, or warts near the mouth, anus, or vagina
B. Painful urination
C. Painful sex
D. *All are correct
E. All are wrong
633. The bacterium Chlamydia trachomatis can cause:
A. Lymphogranuloma venereum (LGV) and orchitis
B. Epididymitis and urethritis
C. Chlamydia
D. *All are correct
E. All are wrong
634. Where, in the skin, are the cells which divide to form new cells?
A. Stratum corneum
B. Stratum lucidum
C. *Stratum basale
D. Stratum granulosum
E. Stratum spinosum
635. What eventually happens to the cells of the epidermis?
A. *They die and flake off
B. They are reabsorbed into the skin
C. They remain on the surface of the skin
D. All are correct
E. All are wrong
636. Name the structures in the dermis which produce oil?
A. Pigment cells
B. Hair follicles
C. *Sebaceous glands
D. Sweat glands
E. All are wrong
637. Which material is the main component of surface skin cells?
A. *A type of protein called keratin
B. A type of protein called albumen
C. A type of lipid called trans-fatty acid
D. All are correct
E. All are wrong
638. What kind of substance is secreted by the sebaceous glands?
A. Sweat or perspiration
B. A milky substance consisting of mineralised water
C. *An oily substance called sebum
D. A milky substance consisting of fat
E. All are wrong
639. What are fingernails and toenails made of?
A. Calcium
B. Magnesium
C. Sheets of cartilage
D. *Dead skin cells containing keratin
E. Minerals
640. Which layer of the epidermis consists of cells that are undergoing active cell reproduction
and eventually migrate to the stratum corneum layer to be sloughed off?
A. *Stratum basale
B. Stratum corneum
C. Stratum lucidum
D. Stratum granulosum
E. Stratum spinosum
641. When we look at our fingers, we can see fingerprints. Which of the following layers of
skin are we looking at in order to see the fingerprints?
A. *Ppapillary layer of the dermis
B. Stratum corneum
C. Stratum lucidum
D. Stratum granulosum
E. Stratum spinosum
642. Which of the following layers is the thickest of the cutaneous layer?
A. Ppapillary layer of the dermis
B. Stratum corneum of the epidermis
C. Stratum lucidum of the epidermis
D. Stratum granulosum of the epidermis
E. *Reticular layer of the dermis
643. When we give a patient an injection of medication, many times we place the needle earest
the location of major blood vessels in the skin. This needle is referred to as a “hypodermic eedle”
because it's of the right length to enter the:
A. Dermis
B. Epidermis
C. *Hypodermis
D. All are correct
E. All are wrong
644. Which is the correct sequence of epidermal layers of thick skin, deep to superficial?
A. Stratum spinosum, stratum lucidum, stratum corneum, stratum basale, stratum
granulosum
B. Stratum basale, stratum spinosum, stratum granulosum, stratum lucidum, stratum
corneum
C. *Stratum granulosum, stratum corneum, stratum lucidum, stratum spinosum,
stratum basale
D. Stratum basale, stratum granulosum, stratum corneum, stratum spinosum, stratum
lucidum
E. Stratum corneum, stratum lucidum, stratum spinosum, stratum granulosum,
stratum basale
645. Which is the correct sequence of epidermal layers of thick skin, superficial to deep?
A. Stratum spinosum, stratum lucidum, stratum corneum, stratum basale, stratum
granulosum
B. Stratum basale, stratum spinosum, stratum granulosum, stratum lucidum, stratum
corneum
C. Stratum granulosum, stratum corneum, stratum lucidum, stratum spinosum,
stratum basale
D. Stratum basale, stratum granulosum, stratum corneum, stratum spinosum, stratum
lucidum
E. *Stratum corneum, stratum lucidum, stratum spinosum, stratum granulosum,
stratum basale
646. Which of the following are NOT derivatives of the epidermis?
A. Hair and hair follicles
B. Sebaceous glands
C. Sweat glands
D. *Dermal papillae
E. Nails
647. Which area of the skin contains the most blood vessels?
A. Stratum basale
648.
649.
650.
651.
652.
653.
654.
655.
B. Epidermis
C. Reticular layer
D. Papillary layer
E. Hypodermis
ANSWER D
Which of the following cells is part of the immune system?
A. Merkel cell
B. Melanocyte
C. Keratinocyte
D. Merkel cell
E. *Langerhans' cell
Which of the following thick skin strata is the thickest?
A. Stratum basale
B. *Stratum corneum
C. Stratum lucidum
D. Stratum granulosum
E. Stratum spinosum
Which of the following strata is absent in epidermis?
A. Stratum corneum
B. *Stratum basale
C. Reticular layer
D. Stratum granulosum
E. Stratum spinosum
Which of the following is NOT a function of the skin?
A. *Production of discharges
B. Protection and insulation
C. Preventing unnecessary water loss
D. Excretion
E. Regulation of body temperature
Seborrhea is caused by
A. Overactive sweat glands
B. *Overactive oil glands
C. Overactive mammary glands
D. Overactive ceruminous glands
E. All are wrong
How is genital herpes treated?
A. *With medication
B. With radiation
C. With surgery
D. All are correct
E. All are wrong
How is genital herpes diagnosed?
A. *Physical exam
B. X-ray
C. Ultrasound
D. All are correct
E. All are wrong
How many types of herpes simplex virus are there?
A. One
B. *Two
C. Three
D. Four
656.
657.
658.
659.
660.
661.
662.
663.
664.
E. Five
What should a pregnant woman with HSV do to avoid passing the virus on to her baby?
A. Stay in close contact with her obstetrician during pregnancy
B. Be prepared to have a cesarean delivery if an outbreak occurs at the time of labor
C. Take an antibiotic
D. *A and B
E. B and C
The most common type of psoriasis is:
A. *Plaque psoriasis
B. Guttate psoriasis
C. Erythrodermic psoriasis
D. Pustular psoriasis
E. All are wrong
What plays a major role in psoriasis?
A. The digestive system
B. The circulatory system
C. *The immune system
D. The nervous system
E. All are wrong
What type of psoriasis is the most common form?
A. Pustular
B. *Plaque
C. Guttate
D. Inverse
E. All are wrong
HIV is an abbreviation for:
A. Human immune virus
B. Human immunodeficiency virus
C. Humanoid immunodeficiency virus
D. *Humanus immunocompromisation virus
For what infections people with HIV and AIDS are largely prone to:
A. Systemic infections
B. *Opportunistic infections
C. Superficial infections
D. Hospital-acquired infections
E. All are wrong
Which is not considered a common method of transmission for HIV?
A. Blood
B. Genital secretions
C. Breast milk
D. *Urine
E. All are wrong
Stage 3 of HIV infection is referred to as:
A. Immunocompromised
B. *AIDS
C. Progressive AIDS
D. Remission
E. All are wrong
People with HIV and AIDS are largely prone to:
A. *Opportunistic infections
B. Systemic infections
C. Superficial infections
665.
666.
667.
668.
669.
670.
671.
672.
673.
D. Hospital-acquired infections
E. All are wrong
Signs and symptoms of shingles are generally:
A. Burning pain and sensitive skin
B. Oozing blisters that crust over
C. Pain without blisters
D. *All are correct
E. All are wrong
What is the most common complication of shingles?
A. *Nerve pain
B. Joint pain
C. Back pain
D. Neck pain
E. All are wrong
Medical prescription treatments that combat shingles are called:
A. *Antiviral medications (antivirals)
B. Antiherpetic medications (antiherpetics)
C. Antizoster medications (antizosters)
D. Antivaricella medications (antivaricellas)
E. All are wrong
The shingles vaccine is available for people:
A. Of any age
B. Ages 13 to 55
C. *Over the age of 60
D. Babies under 2 years of age
E. All are wrong
A shingles outbreak typically lasts:
A. One to two weeks
B. One to five weeks
C. *Three to four weeks
D. Five to six weeks
E. All are wrong
In what part of the body does the Varicella zoster virus (VZV) settle?
A. In cranial nerves
B. *In nerve roots and ganglia
C. In the sciatic nerve
D. In the digestive tract
E. All are wrong
What can happen in the body to cause or reactivate shingles?
A. Emotional stress
B. Acquired immunodeficiency syndrome (AIDS)
C. Cancer and chemotherapy
D. *All are correct
E. All are wrong
Most doctors reach a diagnosis of shingles by which method?
A. Visual diagnosis
B. Laboratory analysis of blister fluid
C. Wasserman reaction
D. *Correct answer A and B
E. Correct answer B and C
In general, the cure for shingles is:
A. The shingles vaccine
674.
675.
676.
677.
678.
679.
680.
681.
B. Zovirax, Valtrex, or Famvir as antiviral medications
C. Allowing the illness to run its course
D. *There is no cure for shingles
E. All are wrong
How does a shingle normally appear on the body?
A. *On one side of the back, chest, face, and/or legs
B. On both sides of the back, chest, face, and/or legs
C. Mainly on the chest or back
D. All over the face and body
E. All are wrong
What is believed to be one of the possible rosacea causes?
A. *Demodex folliculorum
B. Propionibacterium acnes
C. Staphylococcus aureus
D. Streptococcus pyogenes
E. All are correct
Due to what process does facial skin become red resulting in rosacea development?
A. Aneurysm
B. *Vasodilation
C. Erection
D. Constipation
E. All are correct
What is not a complication of the untreated later stages of rosacea?
A. Sebaceous hyperplasia
B. Teleangiectasia
C. Rhinophyma
D. *Trigeminal neuralgia
E. All are correct
What is ocular rosacea?
A. *It is a disease that affects eyelids and skin
B. It is type of rosacea , which looks like red circles on the skin
C. It is rosacea combined with acne
D. It is conjunctivitis and rosacea combined
E. All are wrong
What class of living organisms do fungi belong to?
A. Plants
B. Animals
C. Viruses
D. *Fungi form their own separate class
E. Bacteria
What part of human body is most vulnerable for fungi?
A. *Skin
B. Brain
C. Heart
D. Stomach
E. Liver
What does a medical term “dermatophytosis” mean?
A. Specific type of fungi
B. *Fungal skin infection
C. Anti-fungal medication
D. The process of fungi reproduction
E. All are wrong
What are the key features of athlete’s foot?
A. The infection usually affects the skin between toes
B. The infection causes whitening and scaling of the skin
C. The infection is very contagious
D. *All the mentioned is true for the athlete’s foot
E. All are wrong
683. What part of human body does jock itch usually affect?
A. *Groin area
B. Feet
C. Armpits
D. Scalp
E. Palms
684. Who are at greater risks to develop jock itch?
A. Men
B. People wearing tight underwear
C. People who sweat a lot
D. Obese people
E. *All are correct
685. Why is the fungal skin infection “tinea corporis” commonly called “ringworm”?
A. Because it is caused by worms
B. Because it is caused by fungi of round shape
C. *Because it causes round ring-like lesions on the skin
D. Because it is easier to pronounce
E. All are wrong
686. How do fungi affect human nails?
A. Fungi make nails yellow or discolored
B. Fungi make mails thicker
C. Fungi make nails deformed
D. *All are correct
E. All are wrong
687. What factors contribute to the development of fungal infections, including nail
infections?
A. *Warmth and moisture
B. Cold and dryness
C. Sun light and wind
D. Smoking and drinking
E. All are correct
688. What is the name for a class of medications, commonly used for the treatment of fungal
skin infections?
A. Antibiotics
B. Analgesics
C. *Antifungal
D. Antidepressants
E. Antivirus
689. What can one do to prevent fungal skin infections?
A. Dry skin after bathing
B. Wear loose underwear and clothing
C. Avoid sharing towels or hair brushes
D. Avoid tight shoes
E. *All the above mentioned
690. Mark the true statement:
A. *Rosacea is a treatable disorder
682.
B. Rosacea is a curable condition
C. Rosacea is a contagious disease
D. Rosacea is a mental disorder
E. All are wrong
691. Define which group of people is more likely to develop rosacea:
A. Children
B. Adults
C. *Older people
D. It does not depend on the age
E. Children and adults
692. What is the most significant rosacea symptom?
A. Pimples
B. Shortness of breath
C. Sweating
D. *Red face
E. All are wrong
693. What part of the body cannot be affected with rosacea?
A. *Legs
B. Chest
C. Neck
D. Back
E. All are wrong
694. What are clinical features of athlete’s foot?
A. The infection usually affects the skin between toes
B. The infection causes whitening and scaling of the skin
C. All are wrong
D. The infection is very contagious
E. *All the mentioned is true for the athlete’s foot
695. What are the main signs and symptoms of shingles?
A. Burning pain and sensitive skin
B. Oozing blisters that crust over
C. Pain without blisters
D. *All are correct
E. All are wrong
696. Sebaceous glands secreted:
A. Sweat
B. A milky substance consisting of mineralised water
C. *An oily substance called sebum
D. A milky substance consisting of fat
E. All are wrong
697. Pimples are also called:
A. Blackheads
B. Whiteheads
C. Papules
D. *Pustules
E. Cysts
698. Patient with acne can have any of these skin lesions, except:
A. Cysts
B. Nodules
C. *Hives
D. Blackheads
E. Whiteheads
699.
Baby 4 years old has erosion on the lip. What primary skin lesion was previously?
A. Nodule
B. Papule
C. Urticaria
D. *Vesicle
E. Macule
700. During prophylactic assessment of kindergarten children 2 cases of pediculosis were
diagnosed. What drug we must use for their treatment?
A. *Nittiphor
B. АРРІВО
C. Chloramine
D. Chlorophos
E. Neopin
701. 8 year old girl diagnose is scabies. What lab test we must use for diagnosis of Sarcoptes
scabiei?
A Prick test
B Diascopy
C Dark-field microscopy
D *Skin scraping
E Skin biopsy
702. 40 year old male patient presents with follicular comedones with or without
inflammatory papules, pustules, and nodules. Most likely diagnosis is:
A Seborrheic dermatitis
B Systemic Lupus Erythematosus
C Rosacea
D *Acne vulgaris
E Atopic dermatitis
703. 44 year old female patient suffered from folliculitis for 10 years. Possible causative
agent of this can be all, except:
A Shaving
B *Hypovitaminosis
C Staphylococcus aureus
D Chemical irritation
E Skin injury
Situatinal tasks
1. Patient 23 years old has scar on the left leg. What primary skin lesion was previously?
A Crust
B Lichenification
C *Ulcer
D Erosion
E Vegetation
2. Patient 18 years old has scar on the left leg. What primary skin lesion was previously?
A Crust
B Lichenification
C *Nodule
D Erosion
E Vegetation
3. Patient 29 years old has hypo pigmentation on the left leg. What primary skin lesion was
previously?
A Nodule
B Lichenification
C Ulcer
D *Papule
E Vegetation
4. Patient 20 years old has crusts on the left forearm. What primary skin lesion was previously?
A Nodule
B Papule
C Urticaria
D *Vesicle
E Macule
5. Patient 10 years old has yellow-honey crusts on the right forearm. What primary skin lesion
was previously?
A Nodule
B Papule
C Urticaria
D Macule
E *Pustule
6. Patient 29 years old has hyper pigmentation on the left leg. Its characterized for all skin
lesions, except:
A *Urtica
B Nodule
C Plaque
D Ulcer
E Vesicle
7. Patient 24 years old has itching, more severe at night. Upon examination: on the skin of
glands mummer, abdomen, on the edges of the fingers, toes or sides of the hands and feet
little papules, hemorrhage crusts. Excoriations are absent. Most likely diagnosis is:
A Eczema
B *Scabies
C Pediculosis
D Allergic dermatitis
E Urticaria
8. Patient 6 years old, during 2 weeks, has skin lesions connected with hard itching, more
severe at night. Upon examination: in the interdigital webs forearm little red papules,
hemorrhage crusts, excoriations. Most likely diagnosis is:
A Eczema
B Atopic dermatitis
C Allergic dermatitis
D *Scabies
E Pediculosis
9. 57 year old female patient use corticosteroids for 2 years. Which of the following is a
possible adverse side effect of chronic use of topical steroid medications?
A Striae
B *All are correct
C Atrophy of the skin
D Folliculitis
E Ectasias
10. Mother of 8-month baby complaints are: presence of skin lesions and itching more severe at
night. Upon examination: on the palms and soles – little vesicles. On the skin of face, neck,
trunk – little vesicles, which lie near each other, inflamated papules, pustules, crusts,
excoriations. Most likely diagnosis is:
A *Scabies
B Dermatitis
C Pediculosis
D Atopic dermatitis
E Syphilitic pemphigus
11. 36 year old female patient diagnose is scabies. What lab test we must use for diagnosis of
Sarcoptes scabiei?
A Wasserman reaction
B Prick test
C *Skin scraping
D Tzank smear
E Skin biopsy
12. Patent 17 years old complaints are: itching more severe at night. Upon examination: on the
skin of the palms and soles – little vesicles, red papules, hemorrhage crusts, excoriations;
erythema, papules and ectyma on the genital organs. Most likely diagnosis is:
A Pediculosis
B Syphilis
C *Scabies
D Candidal infection
E Contact dermatitis
13. Women 26 years old complaints are: itching more severe at night. Upon examination: on the
skin of the palms and soles – little vesicles, red papules, hemorrhage crusts, excoriations.
Ardy symptom is positive. Most likely diagnosis is:
A *Scabies
B Pediculosis
C Contact dermatitis
D Spread pyoderma
E Allergic dermatitis
14. Women 35 years old complaints are: itching of the head. Trip by train, from anamnesis.
Upon examination: purulent crusts, excoriations on the skin of the head; great plaques
closely connected with hair follicle. Most likely diagnosis is:
A Pyoderma of the scalp
B Seborrhea dermatitis
C Tinea capitis
D *Pediculosis
E Neurodermitis
15. Women 45 years old complaints are erythematic skin lesions on the face, connected with
itching, which increase after washing of the face. Upon examination: little papules and zone
of scales on the erythematic base on the skin of the face. Most likely diagnosis is:
A *Demodecosis, erythematic and squamosis form
B Acne vulgaris
C Demodecosis, papule form
D Seborrhea dermatitis
E Contact dermatitis
16. Upon examination: on the skin of the face pustules and nodules based on the erythema,
connected with scaling. Laboratory: scabies mite during scrubbing of the beard zone. Put
correct diagnose, please.
A Demodecosis, papule and pustules form
B Acne vulgaris
C Demodecosis, papule form
D Seborrhea dermatitis
E *Scabies
17. Women 50 years old complaints are: erythema of the face, itching. Upon examination:
erythema and scaling. Laboratory: demodex folliculorum during scrubbing of the beard zone.
Put correct diagnose, please.
A Demodecosis, papule and pustules form
B Acne vulgaris
C Allergic dermatitis
D Seborrhea dermatitis
E *Demodecosis, erythematic and squamosis form
18. Patient has well defined 2-Cm localized hair loss in the posterior scalp. This condition is
most likely:
A Telogium effluvium
B Anagen effluvium
C Traction alopecia
D Traumatic alopecia
E *Alopecia areata
19. Patient 23 years old has of impetigo with itching, more severe at night. What is the diagnose?
A Dermatitis
B Psoriasis
C Eczema
D *Scabies
E Tinea
20. Patient 37 years old has hard itching, more severe at night. Upon examination: little
vesicles, inflammative papules, excoriations, on the trunk, interdigital space of the palms.
Most likely diagnosis is:
A. Urtica
B. *Scabies
C. Dermatitis
D. Eczema
E. Pituriais versicilor
21. 67 year old female patient presents with nocturnal pruritus, red macules on nape of neck and
scalp. Most likely diagnosis is:
A *Pediculosis Capitis
B Dermatitis
C Tinea capitis
D Psoriasis
E Pediculus humanus
22. 18 year old female patient presents with open and closed comedones, papules, pustules,
nodules, and cysts of varying severity, found on face, neck, shoulders, and upper trunk. Most
likely diagnosis is:
A *Acne vulgaris
B Atopic dermatitis
C Seborrheic dermatitis
D Systemic Lupus Erythematosus
E Rosacea
23. 45 year old female patient presents with flushing and persistent central facial erythema with
telangiectasia. Most likely diagnosis is:
A Scabies
B Acne vulgaris
C Atopic dermatitis
D Seborrheic dermatitis
E *Rosacea
24. 40 year old male patient presents with central facial erythema with transient papules and
pustules. Most likely diagnosis is:
A Seborrheic dermatitis
B Eczema
C *Rosacea
D Acne vulgaris
E Atopic dermatitis
25. 38 year old female patient presents painful, tender, erythematous papule with central pustule
single lesions in the beard area. Most likely diagnosis is:
A Insect bites
B Scabies
C Rosacea
D Tinea
E *Folliculitis
26. 25 year old female patient suffered from folliculitis for 5 years. Possible causative agent of
this can be all, except:
A Staphylococcus aureus
B Chemical irritation
C Friction
D *Scabies mite
E Perspiration
27. 64 year old female patient suffered from obesity for 10 years. Upon examination: recurrent
boils on the axillae areas. Most likely diagnosis is:
A Cellulitis
B *Hidradenitis suppurativa
C Pilonidal cysts
D Bacterial folliculitis
E Squamous cell carcinoma
28. 56 year old male patient suffered from Hidradenitis Suppurativa for 6 month. Possible
methods of treatment includes all, except:
A Incision and drainage
B Antibiotics
C Isotretinoin
D *Antiviral drugs
E Corticosteroids
29. 44 year old male patient diagnose is scabies. What lab test we must use for diagnosis of
Sarcoptes scabiei?
A Prick test
B *Skin Scraping
C Tzank smear
D Balzer test
E Skin biopsy
30. 45 year old female patient presents with central facial erythema with transient papules and
pustules. Those symptoms can be aggravated by:
A Wine and caffeine
B Exercise
C Topical steroid
D Anxiety
E *All are correct
31. 56 year old female patient diagnose is scabies. What lab test we must use for diagnosis of
Sarcoptes scabiei?
A *Oil preparation
B KOH prep
C Wood’s light
D Skin biopsy
E Tzanck prep
32. Skin lesions appeared 5 days ago. Skin assessment: little inflammatory erythematous pustules.
Except this, erosions, this appeared after pustules breakdown, crusts. About what disease we can
thing?
A Eczema
B *Impetigo vulgaris
C Herpes simplex
D Contact dermatitis
E Candidiasis
33. Boy 5 years old, during 5 days have little inflammatory erythematous pustules, erosions, which
appeared after pustules breakdown, crusts. About what dermatosis we can thing?
A *Іmpetigo
B Dermatitis herpetiphormic
C Pemphigus vulgaris
D Eczema
E Herpes zoster
34. Two children from one family have yellow-honey crusts, on the face, amount of which
rapidly increase. About what disease we can thing?
A Dermatitis
B *Impetigo
C Tinea
D Furuncle
E Candidiasis
35. What ointment we must prescribe for patient with impetigo?
A Herpevir
B Prednizolon
C Clotrimazole
D *Tetracycline
E Synaphlan
36. What drugs we must prescribe for patient with furuncle?
A Corticosteroids
B Antiinflammative drugs
C Antifungal drugs
D *Antibiotics
E Vitamins
37. Boy 16 years old has carbuncle. What type of infection it is?
A *Bacterial
B Fungal
C Viral
D Parasitogenic
E All are wrong
38. Women 24 years old have eythrasma. What type of infection it is?
A *Bacterial
B Fungal
C Viral
D All are correct
E All are wrong
39. Old women 75 years old have ecthyma. What type of infection it is?
A *Bacterial
B Fungal
C Viral
D Parasitogenic
E All are wrong
40. Patient has lichen planus. What we must use for topical treatment?
A Cream
B Aerosol
C *Ointment
D Lead water
E Mixed drugs
41. Patient 6 years old complaints are: red plaques covered with scales on the pressing zones.
About what disease we can thing?
A Dermatitis
B Eczema
C Tinea
D *Psoriasis
E Lichen planus
42. What type of drugs we must prescribe for patients with psoriatic plaques?
A Paste
B *Ointment
C Lead water
D Cream
E Mixed drugs
43. Patient has red, flat, umbilicated, pearly colored. About what disease we can thing?
A Psoriasis
B Dermatitis herpetiphormic
C Tinea
D *Lichen planus
E Eczema
44. Patient 21 year old has zone of lichenification on the hair, surrounded by papules,
excoriations. Except this, itching. Put diagnose.
A *Lichen planus
B Dermatitis
C Eczema
D Psoriasis
E Syphilis
45. Doctor diagnose on the patient skin Koebner’s phehomenon. For what disease it is
characterized?
A *Psoriasis
B Scabies
C Pituriasis rosea
D Tumor
E Acne vulgaris
46. Doctor diagnose on the patient skin ”Herald patch”. For what disease it is characterized?
A Psoriasis
B *Pituriasis rosea
C Pituriasis lichenoid chronic
D Acne vulgaris
E Acne vulgaris
47. Doctor diagnose on the patient skin “Woronow ring”. For what disease it is characterized?
A Licnen planus
B Pyoderma
C Scabies
D *Psoriasis
E Syphilis
48. Patient 45 year old has psoriasis. What type of the disease it is?
A Sexually transmitted disease
B Virus disease
C Fungal disease
D *Papulosquamous disease
E Pyoderma
49. Young man 38 years old have ltitle numerous zones of alopecia with incomplete hair loss
(“moth eaten”). For what disease it is characterised?
A Psoriasis
B Tinea capitis
C *Syphilitic alopecia
D Partial alopecia
E Discoid LE
50. 23 year old female patient presents with sharply-defined but irregular brown, scaly patches in
the groin. For what disease it is characterised?
A Tinea of the groin
B Eczema
C Dermatitis
D *Erythrasma
E Candidiasis
51. 5 year old boy suffered from this disease for 5 days. Upon examination: thin-walled vesicle
on erythematous base, that soon ruptures & the exuding serum dries to form yellowishbrown (honey-color) crusts. Most likely diagnosis is:
A *Impetigo
B Pemphigus vulgaris
C Eczema
D Herpes zoster
E Herpetiform dermatitis
52. 28 year old female patient presents with raised areas of inflamed skin covered with silvery
white scaly skin on the extensor surfaces, back, sacrum, hairline, knees, elbows. Most likely
diagnosis is:
A *Psoriasis vulgaris
B Eczema
C Allergic dermatitis
D Atopic dermatitis
E Guttate psoriasis
53. 45 year old male patient presents with nail pitting, onycholysis of distal nail bed and “oil
drops”. Most likely diagnosis is:
A Eczema
B Lichen planus
C Onychomycosis
D *Nail psoriasis
E All are wrong
54. 68 year old male patient presents with vesicles on soles of hands & feet, painful rather than
itchy. Patient suffered from this disease for 25 years. Most likely diagnosis is:
A Contact dermatitis
B *Palmoplantar psoriasis
C Eczema
D Allergic dermatitis
E Tinea of the palms and soles
55. 47 year old male patient presents with smooth inflamed patches in skin folds of skin and
under the breasts. Most likely diagnosis is:
A Psoriasis vulgaris
B Pustular psoriasis
C Guttate psoriasis
D Erythrodermic psoriasis
E *Flexural psoriasis
56. 58 year old female patient presents with smooth inflamed patches in skin folds of skin and
under the breasts. Most likely diagnosis is:
A Erythrodermic psoriasis
B Psoriasis vulgaris
C Pustular psoriasis
D Guttate psoriasis
E *Flexural psoriasis
57. 34 year old female patient presents with plaques covered with silvery white scaly skin on the
extensor surfaces, back, sacrum, hairline, knees, and elbows. When the plaques are scraped,
one can see pinpoint bleeding from the skin below. This sign is:
A *Auspitz's sign
B Bulzer test
C Prick test
D Tzanck test
E All are wrong
58. 49 year old male patient presents with plaques covered with silvery white scaly skin on the
extensor surfaces, back, sacrum, hairline, knees, and elbows. Auspitz's sign is positive. Most
likely diagnosis is:
A Tinea of the skin
B Secondary syphilitic
C Eczema
D Allergic dermatitis
E *Psoriasis
59. 63 year old female patient presents with plaques covered with silvery white scaly skin on the
extensor surfaces, back, sacrum, hairline, knees, and elbows. Koebner’s phenomenon is
positive. Most likely diagnosis is:
A Tinea of the skin
B *Psoriasis
C Secondary syphilitic
D Eczema
E Allergic dermatitis
60. 68 year old female patient presents with reddish-purple, flat-topped bumps that may be very
itchy and thinning and surface roughnesses of the nail plate with longitudinal idges. Most
likely diagnosis is:
A Tinea of the skin
B Psoriasis
C *Lichen planus
D Eczema
E Allergic dermatitis
61. 45 year old male patient presents with involves the mucous membranes of the mouth and
vulvovaginal area and reddish-purple, flat-topped bumps. Most likely diagnosis is:
A Candidiasis
B Primary syphilis
C Secondary syphilis
D *Lichen planus
E Pemphigus vulgaris
62. Patient 20 years old has tinea pedis. What type of ointment we can prescribe
prophylactically?
A Antivirus ointment
B Antibacterial ointment
C *Antifungal ointment
D Corticosteroids
E Vitamin
63. Baby 6 years old has tinea capitis (inflammative-purulent form).What is source of infection
in this case?
A Cow
B Pigeon
C Horse
D *Cat
E Rabbit
64. Patient 35 years old has such complaints: well – demarcated inflammative papules with
elevate board, scales in the groins. Hard itching is present. This condition is most likely:
A Dermatitis
B Erythrasma
C Eczema
D *Tinea of the groin
E Fixed erythema
65. What type of the drug we can prescribe 45 years patient with onychomycosis?
A *Antifungal
B Antivirus
C Antiinflammative
D Antibacterial
E Corticosteroids
66. Patient 20 years old has pain vesicles on the palms, appearance of which connected with visit
of swimming – pool. Clinically: little confluence vesicles, erosions with part of epidermis.
This condition is most likely:
A Dermatitis
B Eczema
C Pemphigus
D *Tinea of the pedis
E Psorasis
67. What type of ointment we must prescribe 25 years female with tinea corporis?
A *Antifungal
B Antivirus
C Antibacterial
D Antiinflammative
E Corticosteroids
68. During 3 month patient has changing of the color of the nails. During pressing – pus. Most
likely diagnosis is:
A Onychomycosis
B Pannaricium
C Impetigo
D *Candidal paronychia
E Chronic eczema
69. 28 year old female patient presents with paronychia. What type of drug we must prescribe for
this patient?
A Antibacterial
B Antiinflammative
C *Antifungal
D Corticosteroids
E Antiinflammative
70. 46year old male patient presents with Candida paronychia. What type of drug we must
prescribe for this patient?
A Antibacterial
B Antivirus
C Corticosteroids
D *Antifungal
E Antiinflammative
71. On the upper part of the thorax patient has well-demarcated, different in color macules 1-1,5
sm. in dm. Scales are present on the surface of these lesions. Balzer test is positive. This
condition is most likely:
A Psoriasis
B Dermatitis
C *Pituriasis versicolor
D Syphilitic roseola
E Eczema
72. 32 years patient presents with a large amount of non-inflammative maculae’s with hidden
scales on the surface. Subjective sensations are absent. This condition is most likely:
A Psoriasis
B Dermatitis
C Eczema
D Tinea of the skin
E *Pituriasis versicolor
73. 57 years patient presents with candidal vulvovaginatis. What drug we must prescribe?
A Herpevir
B Prednisilon
C Clotrimazole
D *Fluconasol
E Synaphlan
74. 34 years patient presents with favus. What kind of infection it is?
A Bacterial
B *Fungal
C Viral
D Parasitogenic
E All are wrong
75. Young woman 24 years old have pityriasis versicolor. What kind of infection it is?
A Bacterial
B *Fungal
C Viral
D Parasitogenic
E All are wrong
76. 34 years patient presents with infiltrative-purulent form of tinea capitis. What type of drugs
we can use?
A Ointment
B Cream
C Paste
D *Water-lead
E All are correct
77. 27 years patient presents with eczema. What we must use for topical treatment?
A Paste
B Ointment
C *Lead water
D Cream
E Mixed drugs
78. 43 years patient presents with atopic dermatitis. What type of ointment we must prescribe for
this patient?
A Antivirus ointment
B Antibacterial ointment
C Antifungal ointment
D *Corticosteroids
E Vitamins
79. 43 years patient presents with acute contact dermatitis. What type of ointment we must
prescribe for patient in this case?
A Antifungal ointment
B Antibacterial ointment
C Antivirus ointment
D Antiinflammative type
E *Corticosteroids
80. 45 years patient presents with eczema. What type of medicine for topical treatment we must
prescribe for this patient?
A Cream
B *Lead water
C Ointment
D Aerosol
E Vitamins
81. Patient 31 years old, during 1 year has skin lesions on the palms which connected with hard
itching. Upon examination: symmetrical erythema with little vesicles, erosions, weeping,
surrounded by inflammative papules, crusts. This condition is most likely:
A Herpes simplex
B Scabies
C *Eczema
D Atopic dermatitis
E Dermatitis
82. Patient 67 years old has severe itching, which he connected with allergy. What drug we must
prescribe?
A Lamizile
B *Phencarolum
C Delagil
D Metronidazole
E Doxycyclini hydrochloridum
83. In case of acute vesicular tinea pedis of 34 years old patient specimens for potassium
hydroxide examination should be taken from:
A Under the vesicle
B Root of the vesicle
C *Both are correct (A and B)
D Both are wrong (A and B)
E All are correct
84. 23 year old female patient presents with worsening pain on her toenails during regular
jogging. Upon exam, yellowish discolored lifted nails with distal sub-ungal hyperkeratosis
involving all the nails on her right foot. In addition, scaling in between the toes was also
noted. Name the disease:
A Psoriasis
B *Onychomycosis / Tinea Pedis
C Lichen planus
D Eczema
E All are wrong
85. 45 year old female patient presents with pain with a vesicular eruption followed by scabbing
and skin scarring in the region of the coetaneous distribution of the ophthalmic maxillary
divisions of the trigeminal nerve. Most likely diagnosis is:
A *Zoster ophthalmicus
B Herpes zoster
C Herpes simplex
D Chicken pox
E Herpetiform dermatitis
86. 27 year old female patient presents with clusters and groups of vesicles on erythematous
base. Most likely diagnosis is:
A *Pemphigus Erythematosus
B Pemphigus Vegetans
C Pemphigus Foliaceus
D Pemphigus Vulgaris
E Herpes Simplex Viruses
87. 47 year old male patient presents with erosions in gingiva with small striations at the
periphery of the lesion. On the forearms: purple, polygonal flat-topped papules. Most likely
diagnosis is:
A Pemphigus Vulgaris
B *Erosive Lichen Planus
C Pemphigus Foliaceus
D Pemphigus Erythematosus
E Bullous lichen Planus
88. 49 year old female patient suffered from Pemphigus Vulgaris for 8 years. Diet in case of
Pemphigus vulgaris means restricted on hard foods that may traumatize oral epithelium
mechanically:
A Nuts
B Chips
C Hard vegetables
D *All are correct
E All are wrong
89. 33 year old female patient presents with erosions in buccal mucosa with small striations at
the periphery of the lesion. On the trunk: purple, polygonal flat-topped papules. Most likely
diagnosis is:
A *Erosive Lichen Planus
B Pemphigus Vulgaris
C Pemphigus Foliaceus
D Pemphigus Erythematosus
E Bullous lichen Planus
90. 45 year old female patient presents with painful shallow ulcer in the buccal mucosa.
Previously: bulla, which rupture. On the trunk: purple, polygonal flat-topped papules. Most
likely diagnosis is:
A *Bullous lichen Planus
B Erosive Lichen Planus
C Pemphigus Foliaceus
D Pemphigus Erythematosus
E Dermatitis Herpetiformis
91. 38 year old male patient presents with white areas that ulcerate in the mouth. On the
forearms: purple, polygonal flat-topped papules. Most likely diagnosis is:
A Pemphigus Vegetans
B Pemphigus Foliaceus
C Pemphigus Erythematosus
D *Bullous lichen Planus
E Dermatitis Herpetiformis
92. 48 year old female patient presents with numerous flaccid bulla on the normal skin of the
trunk. The eruption is symmetrical. Nikolsky’s sign is positive. Name the disease:
A Bullous lichen Planus
B *Pemphigus Vulgaris
C Erosive Lichen Planus
D Impetigo
E Both are wrong
93. 35 year old male patient diagnose - HSV infection. Therapy of this disease include all,
except:
A Acyclovir
B Valacyclovir
C Famciclovir
D H1 anti-histamines
E *Fluconazole
94. 59 year old male patient presents with numerous irregular in shape flaccid bulla on the
normal skin of the trunk. The eruption is symmetrical. In the mucous membranes of the oral
cavity painful erosions. Nikolsky’s sign is positive. Name the disease:
A Bullous lichen Planus
B Erosive Lichen Planus
C *Pemphigus Vulgaris
D Impetigo
E All are wrong
95. 47 year old female patient suffered from this disease for 25 years. Upon examination:
symmetrical numerous irregular in shape flaccid bulla on the normal skin of the trunk. In the
mucous membranes of the oral cavity painful erosions. Nikolsky’s sign is positive. Name the
disease:
A Bullous lichen Planus
B *Pemphigus Vulgaris
C Erosive Lichen Planus
D Impetigo
E All are wrong
96. 54 year old male patient suffered from this disease for 28 years. Upon examination: painful
erosions in the mucous membranes of the oral cavity. On the normal skin of the trunk
symmetrical numerous irregular in shape flaccid bulla. Nikolsky’s sign is positive. This
condition is most likely:
A Bullous lichen Planus
B *Pemphigus Vulgaris
C Erosive Lichen Planus
D Impetigo
E Tinea corporis
97. 58 year old female patient suffered from Pemphigus Vulgaris for 38 years. What type of
disease is it?
A Viral disease
B *Autoimmune disease
C Bacterial disease
D Sexually transmitted disease
E Papulosquamosis disease
98. 47 year old male patient diagnosed Pemphigus Vulgaris. To improve this diagnose it is
necessary to:
A Take skin biopsy for histology
B Take serum for indirect immunofluorescence
C Make Tzanck smear
D *All are correct
E All are wrong
99. 29 year old female patient diagnose - HSV infection. What drug we can prescribe?
A Acyclovir
B Valacyclovir
C Famciclovir
D *All are correct
E All are wrong
100. 47 year old female patient diagnose - HSV infection. Therapy of HSV infection include all,
except:
A Acyclovir
B H2 anti-histamines
C Valacyclovir
D *Itraconazole
E Famciclovir
101. 46 year old male patient suffered from Pemphigus Vulgaris for 8 years. Diet in case of
Pemphigus vulgaris means restricted on:
A Orange juice
B Spicy foods
C Tomatoes
D All are wrong
E *All are correct
102. Objective assessment of a patient with a skin complaint (special examination technique)
include:
A Diascopy
B *All are correct
C Skin-surface microscopy
D Wood’s light
E All are wrong
103. In dermatology we can use:
A *All are correct
B Routine tests
C Special tests
D Skin Biopsy
E All are wrong
104. Special tests in dermatology means all, except:
A Skin scrapings
B Patch test
C *Blood analysis
D Tzank smear
E Electron microscopy
105. 32 year old female patient presents with thick painless ulcer on glens penis covered with
black crust. Not far from it, one lymphatic node without pain. Most likely diagnosis is:
A Ectyma
B Impetigo
C Furuncle
D Carbuncle
E *Syphilis
106. 28 year old male patient presents with little scar on the glens penis. From anamnesis,
presence of painless ulcer on the same place, which heal without treatment. Not far from
previous ulcer remains one thick mobile lymphatic node without pain. Most likely diagnosis
is:
A Herpes virus
B Impetigo
C *Syphilis
D Gonococal infection
E Furuncle
107. 26 year old male received course of treatment against little wound on the glens penis. One
week later, after recovering, on the skin of thorax and abdomen red maculae’s 0,5 sm in dm,
without any subjective sensation appeared.. Name the disease:
A Pituriasis versicolor
B Lichen planus
C *Syphilis
D Psoriasis
E Dermatitis
108. 28 years old female patient presents with papules on the vulvae lips 1 sм in dm., thick with
smooth surface, tendency to confluent, without subjective sensation. Name the disease:
A Herpes infection
B Condiloma accuminata
C *Syphilitic condiloma lata
D Candidiasis
E Chlamidiosis
109. 43 year old male presents with numerous papules 0.5 sm in dm on the palms and soles,
without subjective sensations, which are didn’t confluent, not scaling. Name the disease:
A Psoriasis
B Warts
C *Syphilis
D Tinea
E Lichen planus
110. 32 years old male presents with painless thick ulcer on glens penis, 1 х 1,5 sm, covered
with black crust. Not far from it, one lymphatic node without pain. Most likely diagnosis is:
A Impetigo
B Furuncle
C Ectyma
D *Syphilis
E Carbuncle
111. 24 years old female patient presents with chard chancre. It is clinical symptom of:
A *Primary syphilis
B Secondary syphilis
C Congenital syphilis
D Gonococcal infection
E Late syphilis
112. 46 years old female patient presents with chard chancre. This symptom begins as:
A A single papule
B Well defined papule
C Painful papule
D All are correct
E *All are correct, except “painful papule”
113. Patient 29 years old diagnose – primary syphilis. What are the main clinical symptoms of
this disease?
A Hard chancre
B Polyadenitis
C Roseola
D Regional lymphadenitis
E *Hard chancre, regional lymphadenitis
114. Patient 24 years old has syphilis. What type of immunity will remain after it?
A Congenital
B Gain
C Non-sterility
D Infective
E *Non-sterility, infective
115. Patient 24 years old has syphilis. To diagnose treponema pallidum in dark-field
microscopy, we must take smear from:
A Hard chancre
B Erosions and ulcers from genital organs
C Anorectic zone
D Oral cavity
E *All are correct
116. Patient 37 years old has syphilis. Drugs of choice in syphilis treatment are:
A Coticosteroids
B *Penicillin
C Antiviral drugs
D Antihistamins
E Antiinflammatine drugs
117. 28 years old young man has symptoms of syphilis. To precede Wassermann’s reaction,
we must take:
A Patient urine
B Patient saliva
C *Patient serum
D Patient sweat
E Patient feces
118. 43 year old male presents with numerous zones of alopecia with incomplete hair loss
(“moth eaten”). Its characterized for:
A Psoriasis
B Tinea capitis
C *Syphilis
D Partial alopecia
E Discoid LE
119. What is the main danger reaction for 34 years old patient, who treated syphilis with
penicillin?
A Urticaria
B Fever
C Anaphylactic shock
D *Jarish-Herxheimer reaction
E Allergic dermatitis
120. 24 year old male presents with syphilis. What kind of infection it is?
A Fungal
B Viral
C Parasitogenic
D *Bacterial
E All are wrong
121. 36 year old male presents with button-like, well demarcated painless ulcer on axilla zone.
This patient most likely has:
A Hydradenitis
B *Primary syphilis
C Furuncle
D Carbuncle
E Ecthyma
122. 25 year old male presents with generalized papulosquamous body rash and positive
Wassermann reaction. This patient most likely has:
A Psoriais
B Lichen planus
C *Secondary syphilis
D Allergic dermatitis
E Eczema
123. 44 years old male presents with numerous zone of alopecia and positive Wassermann
reaction. This patient most likely has:
A Tinea capitis
B Discoid LE
C *Syphilis
D Psoriasis
E Lichen planus
124. 35 years old male presents with numerous plantar rash and positive Wassermann reaction.
This patient most likely has:
A Warts
B Primary syphilis
C Psoriais
D *Secondary syphilis
E Lichen planus
125. 28 years old male present with hard chancre on glens penis. Complications of this skin
lesion can be all, except:
A Gangrene
B Phimosis
C Paraphimosis
D Phagedenism
E *Accessory infection of Bartholin’s glands
126. Patient 39 years old diagnose – primary syphilis. The mode of transmission of this disease
can be all, except:
A Sexual
B *Toilet seats
C Kissing
D Sexual perversion
E Accidental inoculation
127. 22 years old female present with hard chancre on the lips. The mode of transmission of
syphilis can be all, except:
A Accidental inoculation
B Through contaminated blood
C *Daily activities
D All are correct
E All are wrong
128. 42 years old male present with numerous papules on the trunk. Wassermann reaction is
positive. Syphilitic papules are characterized by all, except:
A Copper-red in color
B Thickness
C *Presence of acute inflammation
D Absence of subjective sensations
E Absence of acute inflammation
129. 65 years old male present with numerous papules on the palms and soles. Wassermann
reaction is positive. Syphilitic papules are characterized by:
A Copper-red in color
B Thickness
C Absence of acute inflammation
D Absence of subjective sensations
E *All are correct
130. 39 years old female diagnose - “Venera necklace”. These lesions are:
A Erosion in late syphilis
B Affection of hair in secondary syphilis
C Affection of nails in secondary syphilis
D *Combination of hyperpigmentation and depigmentation in secondary syphilis
E Nodules in secondary syphilis
131. 23 year old female patient presents with chlamydial urethritis. What drug we must
prescribe?
A. Biseptol
B. *Doxycyclini hydrochloridium
C. Metronidazole
D. Dexamethasonum
E. Indomethacin
132. 19 years old patient has discharges from urethra during 3 weeks. What type of bacteria can
cause this urethritis?
A. *Chlamydia
B. Pneumococcus
C. Salmonella
D. Yersinia
E. All are wrong
133. 20 years old patient present with urethral discharges during 2 weeks. What type of bacteria
can cause this urethritis?
A. Streptococcus
B. Pneumococcus
C. Salmonella
D. Yersinia
E. *Chlamydia
134. 35 years old patient present with urethral discharges, caused by chlamydia during 3 weeks.
What type of drug we must prescribe for this patient?
A Corticosteroids
B Antiinflammative drugs
C Antifungal drugs
D *Antibiotics
E Vitamins
135. 18 years old patient present with itching, disorders in sexual function during, sometimes
slight discharges from urethra in the morning. What type of bacteria can cause this urethritis?
A. *All are correct
B. Mycoplasma
C. Trichomonada
D. Chlamydia
E. Gonococcus
136. 21 years old patient present with discharges from urethra, pain during emiction during 3
days after sexual contact. Upon examination: enlargement, edema of glens penis. Presence of
yellow-greenish purulent discharges. Name the disease:
A. *Acute gonococci urethritis
B. Trichomoniasis
C. Chlamydia infection
D. Genital herpes
E. Syphilis
137. 20 years old patient present with discharges from urethra during 3 weeks. What type of
bacteria can cause this urethritis?
A. *Gonococcus
B. Pneumococcus
C. Salmonella
D. Yersinia
E. Diphtgeria
138. 34 years old patient diagnose is Hutchinson’s triad. It is means:
A Hutchinson’s teeth
B VIII nerve deafness
C Thickness of the part of clavicle
D Syphilitic interstitial keratitis
E *All are right, except thickness of the part of clavicle
139. 28 years old patient present with rhagades, which are cracks or fissures in the skin around
the mouth. This condition is most likely:
A Primary syphilis
B Secondary syphilis
C *Congenital syphilis
D Late syphilis
E All are wrong
140. 1-week-old infant boy has syphilitic pemphigus. It is characterized for:
A Primary syphilis
B Late congenital syphilis
C *Early congenital syphilis
D Secondary syphilis
E Primary and secondary syphilis
141. 2 years old patient present syphilitic chorioretinitis. Assessment of the eye ground of this
patient shows specific symptom:
A “Herald patch”
B “Christmas tree”
C *“Salt -and -pepper”
D “Moth eaten”
E “Snail-track”
142. 3 years old female patient present syphilitic chorioretinitis. It is characterized for:
A *Early congenital syphilis
B Primary syphilis
C Late congenital syphilis
D Secondary syphilis
E Primary and secondary syphilis
143. Fetus present with Pneumonia Alba. It is characterized for:
A *Early congenital syphilis
B Primary syphilis
C Secondary syphilis
D Late congenital syphilis
E Tertiary syphilis
144. Assessment of newborn placenta show: the chorionic villi are enlarged, and contain dense
laminated connective tissue, and the capillaries distributed throughout the villi are
compressed by this connective tissue proliferation. Such a photomicrograph revealing
cytoarchitectural changes of the placenta seen in:
A Primary syphilis
B *Early congenital syphilis
C Secondary syphilis
D Late congenital syphilis
E Tertiary syphilis
145. 2-week-old infant boy has hepatosplenomegaly. The infant kept his upper extremities in a
flail-like position because of painful periostitis. It is caused by:
A Secondary relapse syphilis
B Neurosyphilis
C Primary syphilis
D *Congenital syphilis
E Secondary syphilis
146. 1-week-old infant girl has perianal excoriation with hepatosplenomegaly is characterized
for:
A *Congenital syphilis
B Neurosyphilis
C Tertiary syphilis
D Primary syphilis
E Secondary syphilis
147. 2-month-old infant boy has nasal discharges (snuffles). They are the earliest sign of:
A Primary syphilis
B Candidiasis
C *Congenital syphilis
D Pemphigus Vulgaris
E Pempgigus Erythematosus
148. 7-year-old girl present with Condyloma latum. It is characterized for:
A Primary syphilis
B *Early congenital syphilis
C Neurosyphilis
D Late congenital syphilis
E Tertiary syphilis
149. 9-year-old boy present with Condyloma latum (congenital syphilis).It is caused by:
A *Bacteria
B Virus
C Mite
D Dermatophyte
E All are wrong
150. 10-year-old boy present with perforated hard palate. It is characterized for:
A *Syphilis
B Eczema
C Candidiasis
D Lochen planus
E Herpes simplex type 1
151. 25 years old female patient present Bartholin’s Abscess. Possible causative agent of this
disease is:
A Mite
B Herpes simplex
C Candidal infection
D *Gonorrhea infections
E Chlamydia infection
152. 4-days-old boy present with tense bulla with a thick base on the on the palms and soles.
Name the disease:
A Pempgigus Erythematosus
B *Syphilitic pemphigus
C Pemphigus Vegetans
D Pemphigus Foliaceus
E Pemphigus Vulgaris
153. 1- month-old girl present with erosive papules on genital organs. This condition is most
likely:
A Candidiasis
B Herpes
C *Congenital syphilis
D Impetigo
E Dermatitis
154. 21 years old male patient present with grouped burning, painful vesicles on the glens penis
during 4 days with inflammative base. Name the disease:
A Hard chancre
B Iimpetigo
C *Herpes simplex type 2
D Dermatitis
E Genital warts
155. Doctor consult 29 years old male patient. Upon examination: on presence of painful
erosions and grouped vesicles on the glens penis. This condition is most likely:
A *Genital herpes
B Secondary syphilis
C Primary syphilis
D Scabies
E Chlamydia infection
156. Patient 30 years old has mucocutaneus candidiasis (vulvovaginatis, stomatitis, paronychii)
for many years. Symptoms of other diseases are absent. What can be causative agent of this
disease?
A Hypovitaminosis
B Local infection
C *AIDS
D Diabetes mellitus
E Hypothyroidism
157. Patient 29 years old has bacillary angiomatosis. What another disease can be present in this
case?
A Hypovitaminosis
B Local infection
C *AIDS
D Diabetes mellitus
E Syphilis
158. Patient 28 years old has herpes simplex virus for 2 years which regress with ulcer and scar.
What can be causative agent of this disease?
A Chronic pneumonia
B Diabetes mellitus
C Local infection
D *AIDS
E Non-rational therapy
159. Patient 25 years old has mucocutaneus candidiasis (vulvovaginatis, stomatitis, paronychii)
for many years. Symptoms of other diseases are absent. What can be causative agent of this
disease?
A *AIDS
B Non-rational therapy
C Hypovitaminosis
D Local infection
E Diabetes mellitus
160. Patient 40 years old has recurrent herpes zoster infection on the same place. What can be
causative agent of this disease?
A Diabetes mellitus
B *AIDS
C Hypovitaminosis
D Chronic candidiasis
E Local infection
161. Patient 30 years old has mucocutaneus candidiasis (vulvovaginatis, stomatitis, paronychii)
for many years. Symptoms of other diseases are absent. What can be causative agent of this
disease?
A Hypovitaminosis
B Local infection
C Hypothyroidism
D *AIDS
E Diabetes mellitus
162. Patient 29 years old has bacillary angiomatosis. What another disease can be present in this
case?
A Hypovitaminosis
B Local infection
C Diabetes mellitus
D Syphilis
E *AIDS
163. Patient 28 years old has herpes simplex virus for 2 years which regress with ulcer and scar.
What can be causative agent of this disease?
A Chronic pneumonia
B Diabetes mellitus
C *AIDS
D Local infection
E Non-rational therapy
164. Patient 25 years old has mucocutaneus candidiasis (vulvovaginatis, stomatitis, paronychii)
for many years. Symptoms of other diseases are absent. What can be causative agent of this
disease?
A Non-rational therapy
B Hypovitaminosis
C Local infection
D *AIDS
E Diabetes mellitus
165. Patient 40 years old has recurrent herpes zoster infection on the same place. What can be
causative agent of this disease?
A Diabetes mellitus
B Hypovitaminosis
C *AIDS
D Chronic candidiasis
E Local infection
166. 4 years child has scratches a lot! There is a family history of both asthma and hay fever.
What medication should be used?
A Triamcinalone 0.1% ointment twice
B Lidex ointment (fluocinonide 0.05%) twice daily
C *Hydrocortisone 1% ointment twice daily
D Temovate ointment (clobetasol 0.05%) twice daily
E All are wrong
167. 5 month child was tended by a pregnant woman on the day he broke out with the eruption,
and she was very worried about risk to her fetus. Disease and advice:
A Roseola, and no risk to the fetus
B Rubella and risk is significant during about the first 5 months of gestation; check
fetal date of conception, and then serum rubella antibody titers of the mother if
appropriate, etc.
C *Erythema infectiosum, and the child has stopped shedding virus by the time the
eruption occurs, i.e., no risk to fetus
D Erythema infectiosum, and the fetus should be followed carefully because of
parvovirus attack of the red cell line, and subsequent possible in utero severe
anemia, congestive heart failure, and even death
E All are wrong
168. 25 year old woman has severe acne and is probably best treated with “Accutane” at:
A *1 mg/kg/day for 3-5 months
B 2 mg/kg/day for 3-5 months
C 3 mg/kg/day for 3-5 months
D 10 mg/kg/day for 3-5 months
E All are wrong
169. The 36 years old men present with “Boils” on his penis, which broke down to form ulcers.
Prescribe a possible treatment, please.
A *Antibiotics
B Delagil
C Cyclosporinee
D Dexamethasonum
E Indomethacin
170. The 24 years old men present with “Boils” on left leg, which broke down to form ulcers.
Prescribe a possible treatment, please.
A Phencarolum
B Delagil
C *Antibiotics
D Dexamethasonum
E Indomethacin
171. 25 years old woman presented on a bank holiday after noticing pale patches on her trunk
following a holiday in Spain. She has no other symptoms. A fine superficial scale was seen
when the surface was lightly scraped with a fingernail. What is the diagnosis?
A Seborrheic dermatitis
B Systemic Lupus Erythematosus
C Rosacea
D Acne vulgaris
E *Pituriais versicilor
177 . 18 years old woman presented on a bank holiday after noticing pale patches on her trunk
following a holiday in Spain. She has no other symptoms. A fine superficial scale was seen when
the surface was lightly scraped with a fingernail. What is the cause?
A Streptococcus
B Pneumococcus
C Salmonella
D Yersinia
E *Piturosporum orbiculare
178 .18 years old woman presented on a bank holiday after noticing pale patches on her trunk
following a holiday in Spain. She has no other symptoms. A fine superficial scale was seen when
the surface was lightly scraped with a fingernail. What is the cause?
A *Malassezi furfur
B Pneumococcus
C Yersinia
D Salmonella
E Streptococcus
179 38 years old woman presented on a bank holiday after noticing pale patches on her trunk
following a holiday in Spain. She has no other symptoms. A fine superficial scale was seen when
the surface was lightly scraped with a fingernail. What is characteristic appearance?
A *Slightly scaly light brown or pink patches on light colored skin
B Slightly scaly papules on light colored skin
C Slightly scaly nodules on light colored skin
D Slightly scaly papules and nodules on light colored skin
E Slightly scaly light brown or pink patches and nodules on light colored skin
180 26 years old woman presented on a bank holiday after noticing pale patches on her trunk
following a holiday in Spain. She has no other symptoms. A fine superficial scale was seen when
the surface was lightly scraped with a fingernail. What test could help?
A Prick test
B Skin Scraping
C Tzank smear
D *Balzer test
E Skin biopsy
181 Middle aged woman had suffered from psoriasis since the age of 19. This time the soles
of her feet and palms of her hands were affected by the rash, which was painful and at the times
caused a burning sensation. What is the likely diagnosis?
A *Pustular psoriasis
B Exfoliative psoriasis
C Flexural psoriasis
D Intertriginous psoriasis
E Plaque psoriasis
182 45 years old woman suffered from pustular psoriasis since the age of 21. This time the
soles of her feet and palms of her hands were affected by the rash, which was painful and at the
times caused a burning sensation. What is the differential diagnosis?
A Fungal infection
B Contact dermatitis
C Eczema
D *All are correct
E All are wrong
183 20 years old woman suffered from Hidradenitis Suppurativa. What treatment does this
patient need?
A Corticosteroids
B *Surgery and antibiotics
C Antifungal drugs
D Antibiotics
E Vitamins
184 28 years old woman suffered from Hidradenitis Suppurativa. What are characteristic
features of this condition?
A Sinus formation
B Abscesses
C Scarring of the deep tissue
D *All are correct
E All are wrong
185 25 years old woman suffered from pustular psoriasis since the age of 17.
Which of the following medications can WORSEN psoriasis?
A Topical steroids
B Methotrexate
C Cyclosporin
D *Beta-blockers
E All are wrong
186 On a routine physical, you find that this healthy young man has an asymptomatic rash on
his back. There is scale when scratched. He has no travel history. A scraping of the scale is
shown. What is the most appropriate treatment?
A No treatment is necessary as this is a normal variation in darkly pigmented
individuals
B *Initiate treatment for tinea versicolor
C Initiate treatment for vitiligo
D No treatment is necessary as this is most consistent with acne scarring
E Initiate treatment for tinea corporis
187 Patient 19 years old presents to you with acne. Which of the following points in her
history could BEST help you in your diagnosis and management?
A Chocolate and milk in her diet
B *Excessive coffee drinking
C Hirsutism and irregular menses
D Poor hygiene
E Lack of exercise
188 25 years old man was evaluated for a red indurate ulcerated plaque on the mid penile
shaft following a week of painless inguinal adenopathy. Three weeks later he developed a
widespread papulosquamous eruption on the trunk and extremities including the palms and soles.
Condylomata lata lesions were noted on the scrotum. His serological tests for syphilis were
positive. What is the likely diagnosis?
A Primary syphilis
B *Secondary syphilis
C Early congenital syphilis
D Late congenital syphilis
E All are wrong
189 35-year-old man had a 2 year history of symmetric large red scaly patches on the scalp,
face, postauricular areas, chest and upper back. What is the likely diagnosis?
A *Dermatitis, seborrheic
B Psoriasis, plaque
C Lupus erythematosus, discoid
D Granuloma annulare
E Keratosis follicularis
190 47 years old woman complained of a generalized progressive pruritic papulopustular
eruption with burrows on the wrists, web spaces of the feet and hands, elbows and knees. She
had a close friend with a similar eruption and itching. What is the diagnosis?
A Urticaria pigmentosa
B Chicken pox
C *Scabies
D Eczema (dermatitis, atopic)
E Contact dermatitis
191 22 years old woman was evaluated for a diffuse scaly papular eruption involving her
chest, abdomen and back. A potassium hydroxide preparation demonstrated spores and
pseudohyphae typical of Malassezi furfur. What is the diagnosis?
A Pityriasis rosea
B Burn, sun
C *Lichen planus
D Tnea versicolor
E Folliculitis, eosinophilic
192 3 year old girl developed an itchy painful vesicular eruption in a dermatomal pattern. She
had a history of varicella at 6 months of age. What is the diagnosis?
A Roseola
B Pityriasis rosea
C *Herpes zoster
D Psoriasis, guttate
E Eczema (dermatitis, atopic)
193 70-year-old man presented with a 2 month history of diffuse pruritic symmetric
violaceous lichenoid papules affecting his hands and feet. Note the longitudinal ridging and
dystrophy of his finger and toe nails. What is the diagnosis?
A *Lichen planus
B Tinea pedis
C Paronychia, acute
D Psoriasis, erythrodermic
E Pityriasis rosea
194 20-year-old man complained of generalized pruritus. Burrows were noted on his wrists,
elbows, and the web spaces of his hands. Please select the correct diagnosis:
A *Scabies
B Eczema (dermatitis, atopic)
C Contact dermatitis
D Lichen planus
E Tinea pedis
195 Symmetric linear papules and pustules covered 3 months infants’ chest and abdomen. His
parents were itching, but their rash was eczematous without intact burrows. Scraping from
several papules on the infant revealed viable female mites. Please select the correct diagnosis:
A Tinea corporis
B *Scabies
C Dermatitis, contact allergic
D Lichen planus
E Eczema
196 This 65-year-pold man developed a cluster of itchy vesicles on his upper lip and nose
after recovering from an upper respiratory tract infection. The rash cleared without treatment
within a week. Please select the correct diagnosis:
A Eczema herpeticum
B *Herpes simplex virus infection
C Tinea corporis
D Dermatitis, atopic
E Impetigo, bullous
197 A 58-year-old woman complained of painful bleeding ulcers in her mouth following the
rupture of collapsed bullae. A skin biopsy with immunofluorescence confirmed the clinical
suspicion of pemphigus vulgaris. What is the diagnosis?
A Lichen planus
B Impetigo, bullous
C *Pemphigus vulgaris
D Varicella
E Cheilitis
198 45-year-old man had an itchy red scaly eruption which continued to spread despite
treatment with a topical steroid for 1 month. The lesions resolved within 2 weeks of initiating
oral griseofulvin. What is the diagnosis?
A Erythrasma
B *Tinea cruris
C Eczema
D Pityriasis rosea
E Dermatitis, seborrheic
199 20-year-old man complained of generalized pruritus. Burrows were noted on his wrists,
elbows, and the web spaces of his hands. Please select the correct diagnosis:
A *Scabies
B Psoriasis
C Dermatitis herpetiformis
D Lichen planus
E Eczema
200 38-year-old man developed a painless ulcer on the distal shaft of the penis. The lesion
cleared with oral doxycycline. The incidental pearly penile papules present at the base of the
glans. What is the diagnosis?
A Herpes simplex virus infection
B Candidiasis
C *Syphilis, primary
D Condiloma acuminate
E Wart, anogenital
«Test questions to figures»
1. ?Based on the photo (Fig. 1), choose diagnosis:
A. Psoriasis nummular
B. Microsporia of the skin
C. Tinea barber
D. * Impetigo vulgaris
E. Tuberculosis cutis colliquvativa
2. Clinical types of the disease on (Fig. 1) are all, except:
A. Impetiginous perleche (angular stomatitis)
B. Annular impetigo
C. Acute diffuse impetigo
D. Bullous impetigo
E. * Impetigo of the bones
3. Predisposing factors of the disease on (Fig. 1) are all, except:
A. Diabetes mellitus
B. * Stress
C. Chrohic infections
D. Diminished of immunity
E. Hypovitaminosis
4. Based on the photo (Fig. 2), choose diagnosis:
A. Tuberculosis cutis colliquvativa
B. Psoriasis nummular
C. Dermatophitia of the skin
D. * Ecthyma
E. Microsporia of the skin
5. The main clinical symptom of the disease on (Fig. 2) is:
A. * A small blister with a red border that may be filled with pus
B. Nodule
C. Maculae
D. Hive
E. Burrow
6. What type of infection is disease (Fig. 2)?
A. Tinea infection
B. * Bacterial infection
C. Viral infection
D. Sexually transmitted infection
E. Parasitogenic infection
7. Based on the photo (Fig. 3), choose diagnosis:
A. Erythema nudosum
B. Psoriasis nummular
C. Dermatophitia of the skin
D. * Carbuncle
E. Allergic contact dermatitis
8. This disease (Fig. 3) means:
A. * Large deep ulcer
B. Large deep fissure
C. Large deep papule
D. Large deep pustule
E. Large deep erosion
9. For what type of infection the disease on (Fig. 3) belong?
A. Fungal
B. * Bacterial
C. Viral
D. Parasitogenic
E. Sexually transmitted
10. The disease on (Fig. 3) is an infection involving:
A. Subcutaneous tissue around several hair follicles
B. * Subcutaneous tissue around one hair follicle
C. Subcutaneous tissue around sweat gland
D. Subcutaneous tissue around sebaceous gland
E. Sebaceous gland
11. Based on the photo (Fig. 4), choose diagnosis:
A. Plaques psoriasis
B. Seborrheal dermatitis
C. Scabies
D. * Sycosis vulgaris
E. Infiltrativepurulent trychophitia of the skin
12. What type of skin lesions are most characterized for the disease on (Fig. 4)?
A. Vesicles
B. * Infiltration with pustule on the surface
C. Scars
D. Maculae’s
E. Papules
13. The disease on photo (Fig. 4) is a:
A. * Piogenic infection of the whole hair follicles
B. Viral infection of the whole hair follicles
C. Piogenic infection of the upper part of the hair follicles
D. Viral infection of the upper part of the hair follicles
E. Sexually transmitted disease
14. Based on the photo (Fig. 5), choose diagnosis:
A. Exudative psoriasis
B. Tinea of the skin
C. Contact dermatitis
D. * Annular impetigo
E. Pemphigus vulgaris
15. What type of skin lesions are most characterized for the disease on photo (Fig. 5)?
A. Vesicles
B. * Yellowbrown crusts, erosions
C. Ulcers
D. Maculae’s
E. Papules
16. Lab. diagnosis of the disease on photo (Fig. 5) means all, except:
A. Skin biopsy
B. Skin swab
C. Pus swab
D. Nasal/skin swab
E. * Balzer test
17. Based on the photo (Fig. 6), choose diagnosis:
A. Psoriasis nummular
B. Tinea of the pulms
C. Pemphigus vulgaris
D. * Bullous impetigo
E. Tuberculosis cutis colliquvativa
18. What type of skin lesions are most characterized for the disease on photo (Fig. 6)?
A. Vesicles
B. * Yellowhoney crusts, erosions
C. Ulcers
D. Maculae’s
E. Papules
19. The disease on photo (Fig. 6) is characterized by the rapid progression of:
A. * Vesicles to flaccid bullae
B. Macule or patch
C. Papule or plaque
D. Papule or nodule
E. Excoriation or lichenification
20. For what type of infection the disease on (Fig. 6) belong?
A. Fungal
B. * Bacterial
C. Viral
D. Parasitogenic
E. Sexually transmitted
21. Based on the photo (Fig. 7), choose diagnosis:
A. Tuberculosis cutis colliquvativa
B. Lichen planus
C. Infiltrativepurulent trychophitia of the skin
D. * Chronic ulcerativevegitans pyoderma
E. Pemphigus vulgaris
22. Based on the photo (Fig. 8), choose diagnosis:
A. Eczema
B. Sycosis vulgaris
C. Dermatitis
D. * Hidradenitis purulent
E. Exudative psoriasis
23. Such multiple violaceous indurated sinus tracts, nodules, and ulcers with purulent and
necrotic drainage photo (Fig. 8) are characterized for:
A. * Hidradenitis purulent
B. Eczema
C. Sycosis vulgaris
D. Dermatitis
E. Exudative psoriasis
24. What kind of infection is the disease on photo (Fig. 8)?
A. Fungal
B. Viral
C. * Bacterial
D. Parasitogenic
E. All are wrong
25. Causative agent of the disease on photo (Fig. 9) is:
A. Mite
B. * Corynebacterium minutissimum
C. Staphylococcal aureus
D. Streptococcal agent
E. All are correct
26. Skin lesions of the disease on photo (Fig. 9) are:
A. Papules
B. * Macula’s
C. Erosions
D. Crusts
E. Pustules
27. Based on the photo (Fig. 9), choose diagnosis:
A. Exudative psoriasis
B. Hidradenitis purulent
C. Pituriasis versicolor
D. * Erythrasma
E. Toxicodermia
28. Based on the photo (Fig. 10) chooses diagnosis:
A. Pituriasis versicolor
B. Scabies
C. Exudative psoriasis
D. * Erythrasma
E. Zhiber’s pink lichen
29. Skin lesions of the disease on photo (Fig. 10) are:
A. Papules
B. * Macula’s
C. Erosions
D. Crusts
E. Pustules
30. Causative agents of the disease on photo (Fig. 10) are:
A. Mite
B. * Corynebacterium minutissimum
C. Staphylococcal aureus
D. Streptococcal agent
E. All are correct
31. Based on the photo (Fig. 11) chooses diagnosis:
A. Pemphigus vulgaris
B. * Impetigo vulgaris
C. Tinea of the skin
D. Eczema
E. Dermatitis
32. What secondary skin lesions are most characterized for the disease on photo (Fig. 11)?
A. Vesicles
B. * Erosions, yellowhoneybrown crusts
C. Ulcers
D. Scales
E. Papules
33. The disease on photo (Fig. 11) is:
A. Very superficial viral infection of the skin
B. * Very superficial bacterial infection of the skin
C. Very superficial fungal infection of the skin
D. Very superficial parasitic infection of the skin
E. Deep bacterial infection of the skin
34. Serpentine burrow, characterized for the disease on photo (Fig. 12), localized in:
A. Stratum granulosum
B. Sratum spinosum
C. Srratum lucidum
D. * Stratum corneum
E. Stratum basale
35. Definitive diagnosis of the disease on photo (Fig. 12) is made by direct visualization of the:
A. Mite
B. Eggs
C. * Mite, eggs
D. Free fluid
E. All are correct
36. Based on the photo (Fig. 12) chooses diagnosis:
A. Exudative psoriasis
B. Tinea of the skin
C. Impetigo vulgaris
D. * Scabies
E. Eczema
37. Based on the photo (Fig. 13) chooses diagnosis:
A. Psoriasis of the scalp
B. Tinea capitis
C. Atopic dermatitis
D. * Impetigo of the hair
E. Seborrheal dermatitis
38. The main characteristic primary skin lesion for the disease on photo (Fig. 13) is:
A. * Pustule
B. Papule
C. Erosion
D. Scale
E. Maculae
39. The disease on photo (Fig. 13) is a type of:
A. Viral infection
B. Tinea infection
C. Sexually transmitted disease
D. * Bacterial infection
E. Parasitogenic disease
40. Based on the photo (Fig. 14) chooses diagnosis:
A. Plague psoriasis
B. Seborrheal dermatitis
C. Impetigo
D. * Scabies
E. Eczema
41. The best method diagnosing of the disease on photo (Fig. 14) is:
A. Wasserman reaction
B. Tzank smear
C. Darkfield microscopy
D. * Direct visualization
E. Diascopy
42. Definitive diagnosis of the disease on photo (Fig. 14) is made by direct visualization of the:
A. Mite
B. Eggs
C. * Mite, eggs
D. Free fluid
E. All are correct
43. Based on the photo (Fig. 15) chooses diagnosis:
A. * Ecthyma vulgaris
B. Tuberculosis cutis colliquvativa
C. Nummular psoriasis
D. Erythema nudossum
E. Microsporia of the skin
44. What type of infection is of the disease on photo (Fig. 15)?
A. Fungal
B. Viral
C. * Bacterial
D. Parasitogenic
E. Sexually transmitted
45. Based on the photo (Fig. 16) chooses diagnosis:
A. Psoriasis of the scalp
B. Tinea capitis
C. * Impetigo vulgaris
D. Seborrheal dermatitis
E. Eczema
46. This type of the disease on photo (Fig. 16) is a superficial skin infection that manifests as:
A. Nodules
B. Ulcers
C. * Honeycolored crust
D. Clusters of patches
E. Clusters of maculae’s
47. Based on the photo (Fig. 17) chooses diagnosis:
A. Ringle impetigo
B. Exudative psoriasis
C. Tinea corporis
D. * Scabies, complicated by pyoderma
E. Contact dermatitis
48. What type of infection is of the disease on photo (Fig. 17)?
A. Fungal
B. Viral
C. Bacterial
D. * Parasitogenic
E. Sexually transmitted
49. Which of the following methods should be undertaken for diagnosis of the disease on photo
(Fig. 17)?
A. Prick test
B. * “Burrowink test”
C. Tzanck test
D. Balzer test
E. Wood’s light
50. Based on the photo (Fig. 18) chooses diagnosis:
A. * Bullous impetigo
B. Tinea corporis
C. Eczema
D. Pemphigus vulgaris
E. Dermatitis
51. The disease on photo (Fig. 18) is a superficial skin infection that manifests as:
A. Nodules
B. Ulcers
C. * Bulla, honeycolored crust
D. Clusters of patches
E. Clusters of maculae’s
52. Based on the photo (Fig. 19) chooses diagnosis:
A. Psoriasis
B. Sycosis vulgaris
C. Dermatitis
D. * Scabies. Ardy symptom
E. Atopic dermatitis
53. This Ardy symptom on photo (Fig. 19) is characterized for:
A. Psoriasis
B. Sycosis vulgaris
C. Dermatitis
D. * Scabies
E. Atopic dermatitis
54. Based on the photo (Fig. 20) chooses diagnosis:
A. * Scabies
B. Exudative psoriasis
C. Lichen planus
D. Erythrasma
E. Eczema
55. Definitive diagnosis of the disease on photo (Fig. 20) is made by direct visualization of the:
A. Free fluid
B. Mite
C. Eggs
D. * Mite, eggs
E. All are correct
56. Based on the photo (Fig. 21) chooses diagnosis:
A. Guttate psoriasis
B. Contact dermatitis
C. Eczema
D. * Scabies
E. Lichen planus
57. Definitive diagnosis of the disease on photo (Fig. 21) is made by direct visualization of the:
A. Free fluid
B. Mite
C. Eggs
D. * Mite, eggs
E. All are correct
58. Which of the following methods should be undertaken for diagnosis of the disease on photo
(Fig. 21)?
A. Prick test
B. * “Burrowink test”
C. Tzanck test
D. Balzer test
E. Wood’s light
59. Which of the following methods should be undertaken for diagnosis of the disease on photo
(Fig.21)?
A. Wasserman reaction
B. Tzanck test
C. Woods light
D. V.D.R.L.
E. * “Burrowinktest”
60. Based on the photo (Fig. 22) chooses diagnosis:
A. * Impetigo vulgaris
B. Nummular dermatitis
C. Pemphigus vulgaris
D. Eczema
E. Tinea barber
61. What type of infection is of the disease on photo (Fig. 22)?
A. Fungal
B. * Bacterial
C. Viral
D. Parasitogenic
E. All are wrong
62. The main characteristic primary skin lesion for the disease on photo (Fig. 22) is:
A. * Pustule
B. Papule
C. Erosion
D. Scale
E. Maculae
63. The main characteristic secondary skin lesion for the disease on photo (Fig.22) is:
A. Papule
B. Vesicle
C. Patch
D. Plaque
E. * Yellow crusts
64. Based on the photo (Fig. 23) chooses diagnosis:
A. Nummular dermatitis
B. Sycosis vulgaris
C. Eczema
D. * Rhinitis impetigo’s
E. Tinea barber
65. What type of infection is of the disease on photo (Fig. 23)?
A. Fungal
B. Viral
C. Parasitogenic
D. * Bacterial
E. Sexually transmitted disease
66. Based on the photo (Fig. 24) chooses diagnosis:
A. Nummular dermatitis
B. Tinea barber
C. Eczema
D. * Furuncle
E. Pemphigus vegetans
67. What type of infection is of the disease on photo (Fig. 24)?
A. Sexually transmitted
B. Fungal
C. Viral
D. Parasitogenic
E. * Bacterial
68. Causative agent of the disease on photo (Fig. 24) is:
A. Pubic lice
B. Treponema pallidum
C. Scabies mite
D. Herpes virus infection
E. * Staphylococcus aureus
69. Based on the photo (Fig. 25) chooses diagnosis:
A. Molluscum contagiosum
B. * Post scabies skin lymphoplasia
C. Guttate psoriasis
D. Tinea corporis
E. Lichen planus
70. Based on the photo (Fig. 27) chooses diagnosis:
A. * Furuncle
B. Infiltrativepurulent tinea corporis
C. Erythema nudossum
D. Pustule psoriasis
E. Eczema
71. The disease from photo (Fig. 26) is:
A. Viral infection
B. Fungal infection
C. * Bacterial infection
D. Parasitogenic infection
E. Sexually transmitted
72. The disease from photo (Fig. 26) is formed by:
A. Tender maculae’s or pustules caused by staphylococcal infection
B. Tender nodules or patch caused by staphylococcal infection
C. Tender maculae’s or plagues caused by staphylococcal infection
D. * Tender nodules or pustules caused by staphylococcal infection
E. Tender nodules or erosions caused by staphylococcal infection
73. Causative agent of the disease on photo (Fig. 26) is:
A. Pubic lice
B. * Staphylococcus aureus
C. Treponema pallidum
D. Scabies mite
E. Herpes virus infection
74. Based on the photo (Fig. 27) chooses diagnosis:
A. Scabies
B. Chancreform pyoderma
C. Tinea corporis
D. Post scabies skin lymphoplasia
E. *Furunculosis
75. Based on the photo (Fig. 28) chooses diagnosis:
A. Pemphigus vulgaris
B. Tinea corporis
C. Vasculitis hemorrhagic
D. *Piodermia
E. Tuberculosis cutis colliquvativa
76. Based on the photo (Fig. 29) chooses diagnosis:
A. Pustular psoriasis
B. Lichen planus
C. Congenital keratoderma of the pedis
D. * Tinea pedis, dishydrotic form
E. Tuberculosis cutis colliquvativa
77. Predisposing factors of the disease on photo (Fig. 29) are:
A. Occlusive foot wear
B. Hyperhidrosis of soles
C. Sharing of wash places
D. Presence of tinea unguium
E. * All are correct
78. The disease from photo (Fig. 29) is:
A. Viral infection
B. * Fungal infection
C. Bacterial infection
D. Parasitogenic infection
E. Sexually transmitted
79. Based on the photo (Fig. 30) chooses diagnosis:
A. Osteal follicululitis
B. Eczema
C. Carbuncle
D. * Infiltrativepurulent trichophytosis of the hair
E. Tuberculosis cutis colliquvativa
80. The disease from photo (Fig. 30) is:
A. * Fungal infection
B. Viral infection
C. Bacterial infection
D. Parasitogenic infection
E. Sexually transmitted
81. Based on the photo (Fig. 31) chooses diagnosis:
A. Pemphigus vulgaris
B. Tinea corporis
C. * Chronic mucocutaneous candidiasis
D. Vasculitis hemorrhagic
E. Tuberculosis cutis colliquvativa
82. Causative agent of the disease on photo (Fig. 31) is:
A. Pubic lice
B. * Candida albicans
C. Treponema pallidum
D. Scabies mite
E. Herpes virus infection
83. Based on the photo (Fig. 32) chooses diagnosis:
A. Exudative psoriasis
B. * Vulva and vagina candidiasis
C. Eczema
D. Lichen planus of mucous membranes
E. Leukoplakia of mucous membranes
84. Causative agent of the disease on photo (Fig. 32) is:
A. Pubic lice
B. * Candida albicans
C. Treponema pallidum
D. Scabies mite
E. Herpes virus infection
85. What type of ointment you must prescribe for the patient with disease on photo (Fig. 33)?
A. Antibiotic
B. Viral
C. Corticosteroid
D. With salicylic acid
E. * Antifungal
86. Typical skin lesion of the disease on photo (Fig. 33) is:
A. * Annular or arcuate plaque
B. Ulcer
C. Crust
D. Burrow
E. All are correct
87. Based on the photo (Fig. 33) chooses diagnosis:
A. Psoriasis
B. Eczema
C. Lichen planus
D. Warts
E. * Tinea manuum
88. Based on the photo (Fig. 34) chooses diagnosis:
A. Nummular psoriasis
B. * Rubrophytosis of the manuum
C. Eczema
D. Impetigo vulgaris
E. Dermatitis
89. The disease from photo (Fig. 34) is:
A. * Fungal infection
B. Viral infection
C. Bacterial infection
D. Parasitogenic infection
E. Sexually transmitted
90. Based on the photo (Fig. 35) chooses diagnosis:
A. Nummular psoriasis
B. * Interdigital candidiasis
C. Eczema
D. Impetigo vulgaris
E. Dermatitis
91. The disease from photo (Fig. 35) is:
A. Bacterial infection
B. * Fungal infection
C. Viral infection
D. Parasitogenic infection
E. Sexually transmitted infection
92. What secondary skin lesions are most characterized for the disease on photo (Fig. 36)?
A. Vesicles
B. * Erosions
C. Ulcers
D. Maculae’s
E. Papules
93. Based on the photo (Fig. 37) chooses diagnosis:
A. Nail psoriasis
B. Lichen planus of the nails
C. Onychodystrophy of the nails
D. * Onychomycosis, hypertrophic type
E. Congenital keratoderma
94. The disease from photo (Fig. 37) is:
A. Bacterial infection
B. * Fungal infection
C. Viral infection
D. Parasitogenic infection
E. Sexually transmitted infection
95. Based on the photo (Fig. 38) chooses diagnosis:
A. Osteal follicululitis
B. * Infiltrativepurulent trichophytosis of the skin
C. Microsporia of the skin
D. Tinea cruris
E. Tuberculosis cutis colliquvativa
96. The disease from photo (Fig. 38) is:
A. Parasitogenic infection
B. Bacterial infection
C. Sexually transmitted infection
D. * Fungal infection
E. Viral infection
97. Based on the photo (Fig. 39) chooses diagnosis:
A. Eczema
B. Toxicodermia
C. Dermatitis
D. * Rubrophytia manuum
E. Atopic dermatitis
98. The disease from photo (Fig. 39) is:
A. Viral infection
B. Parasitogenic infection
C. Bacterial infection
D. Sexually transmitted infection
E. * Fungal infection
99. Based on the photo (Fig. 40) chooses diagnosis:
A. Psoriasis of palms and soles
B. Lichen planus
C. Congenital keratoderma
D. * Tinea pedis, hyperceratotic form
E. Bullous epidermolysis
100. The disease from photo (Fig. 40) is:
A. Viral infection
B. Parasitogenic infection
C. Bacterial infection
D. Sexually transmitted infection
E. * Fungal infection
101. Based on the photo (Fig. 41) chooses diagnosis:
A. * Tinea
B. Nummular dermatitis
C. Eczema
D. Impetigo vulgaris
E. Exudative psoriasis
102. What type of ointment you must prescribe for this patient with disease from photo (Fig.
41)?
A. Antibiotic
B. Viral
C. * Antifungal
D. Corticosteroid
E. With salicylic acid
103. The disease from photo (Fig. 41) is:
A. Sexually transmitted infection
B. Viral infection
C. Parasitogenic infection
D. Bacterial infection
E. * Fungal infection
104. What ointment you must prescribe for the patient with disease from photo (Fig. 41)?
A. Acyclovir
B. Kremgen
C. Mifungar
D. * Econazole
E. Latycort
105. What type of ointment you must prescribe for this patient with disease from photo (Fig.
42)?
A. Antibiotic
B. Viral
C. * Antifungal
D. Corticosteroid
E. With salicylic acid
106. Based on the photo (Fig. 42) chooses diagnosis:
A. Nummular dermatitis
B. Eczema
C. Impetigo vulgaris
D. Psoriasis
E. * Tinea manuum
107. The disease from photo (Fig. 42) is:
A. Parasitogenic infection
B. Sexually transmitted infection
C. * Fungal infection
D. Viral infection
E. Bacterial infection
108. What ointment you must prescribe for the patient with disease from photo (Fig. 43)?
A. Kremgen
B. Acyclovir
C. Herpevir
D. * Pimaphucort
E. Betasalyc
109. Causative agent of the disease on photo (Fig. 43) is:
A. Pubic lice
B. * Candida albicans
C. Treponema pallidum
D. Scabies mite
E. Herpes virus infection
110. What ointment you must prescribe for the patient with disease from photo (Fig. 44)?
A. Acyclovir
B. Kremgen
C. * Mifungar
D. Pimafucort
E. Latycort
111. The disease from photo (Fig. 44) is:
A. Parasitogenic infection
B. Sexually transmitted infection
C. * Fungal infection
D. Viral infection
E. Bacterial infection
112. The disease from photo (Fig. 44) is:
A. Parasitogenic infection
B. Sexually transmitted infection
C. * Fungal infection
D. Viral infection
E. Bacterial infection
113. Based on the photo (Fig. 45) chooses diagnosis:
A. * Rubrophytosis of cruris
B. Exudative psoriasis
C. Seborrheal dermatitis
D. Eczema
E. Pituriasis versicolor
114. The disease from photo (Fig. 45) is:
A. Viral infection
B. Parasitogenic infection
C. Sexually transmitted infection
D. * Fungal infection
E. Bacterial infection
115. What type of ointment you must prescribe for this patient with disease from photo (Fig.
45)?
A. Antibiotic
B. Viral
C. * Antifungal
D. Corticosteroid
E. With salicylic acid
116. Based on the photo (Fig. 46) chooses diagnosis:
A. Exudative psoriasis
B. Seborrheal dermatitis
C. * Rubrophytosis corporis
D. Eczema
E. Pituriasis versicolor
117. The disease from photo (Fig. 46) is:
A. Viral infection
B. Parasitogenic infection
C. Sexually transmitted infection
D. * Fungal infection
E. Bacterial infection
118. What type of ointment you must prescribe for this patient with disease from photo (Fig.
46)?
A. Antibiotic
B. Viral
C. * Antifungal
D. Corticosteroid
E. With salicylic acid
119. Based on the photo (Fig. 47) chooses diagnosis:
A. * Tinea pedis, intertrigenous form
B. Eczema
C. Pemphigus vulgaris
D. Congenital keratoderma
E. Tinea pedis, hyperceratotic form
120. Predisposing factors of disease from photo (Fig. 47) are:
A. Occlusive foot wear
B. Hyperhidrosis of soles
C. Sharing of wash places
D. Presence of tinea unguium
E. * All are correct
121. The disease from photo (Fig. 47) is:
A. * Fungal infection
B. Parasitogenic infection
C. Sexually transmitted infection
D. Viral infection
E. Bacterial infection
122. Based on the photo (Fig. 48) chooses diagnosis:
A. Eczema
B. * Onychomycosis, normotrophic variant
C. Nail psoriasis
D. Nail lichen planus
E. Congenital keratoderma
123. Disease from the photo (Fig. 48) can be:
A. Normotrophic
B. Hypertrophic
C. Atrophic
D. All are wrong
E. * All are correct
124. The disease from photo (Fig. 48) is:
A. Bacterial infection
B. Viral infection
C. Parasitogenic infection
D. Sexually transmitted infection
E. * Fungal infection
125. What symptom is characterized for nails color for the patient with disease from photo
(Fig. 49)?
A. Bolter symptom
B. Oil drops
C. Mouth eaten
D. Snail track
E. * All are wrong
126. What ointment you must prescribe for the patient with disease from photo (Fig. 49)?
A. Acyclovir
B. Kremgen
C. Mifungar
D. * Econazole
E. Latycort
127. The disease from photo (Fig. 49) is:
A. Viral infection
B. Parasitogenic infection
C. Sexually transmitted infection
D. * Fungal infection
E. Bacterial infection
128. Based on the photo (Fig. 50) chooses diagnosis:
A. Nummular psoriasis
B. Scabies
C. * Folliculonodular rubrophytosis
D. Microsporia of the skin
E. Osteal follicululitis
129. The disease from photo (Fig. 50) is:
A. Viral infection
B. Parasitogenic infection
C. Sexually transmitted infection
D. * Fungal infection
E. Bacterial infection
130. What ointment you must prescribe for the patient with disease from photo (Fig. 50)?
A. Acyclovir
B. Kremgen
C. * Mifungar
D. Prednisolon
E. Betasalyc
131. Based on the photo (Fig. 51) chooses diagnosis:
A. Exudative psoriasis
B. * Anthropophilic microsporia
C. Contact dermatitis
D. Bullous impetigo
E. Pemphigus vulgaris
132. The disease from photo (Fig. 51) is:
A. * Fungal infection
B. Viral infection
C. Parasitogenic infection
D. Sexually transmitted infection
E. Bacterial infection
133. What type of ointment you must prescribe for this patient with disease from photo (Fig.
51)?
A. Antibiotic
B. Viral
C. * Antifungal
D. Corticosteroid
E. With salicylic acid
134. What symptom is characterized for this disease from photo (Fig. 51)?
A. “Mouth eaten”
B. * All are wrong
C. “Woronow ring”
D. “Koebner phenomenon”
E. “Bolter symptom”
135. The causative agent of the disease from photo (Fig. 51) is:
A. Epidermophyton
B. Malassezia furfur
C. * Microsporum
D. Trichophyton
E. All are wrong
136. Based on the photo (Fig. 52) chooses diagnosis:
A. Nail lichen planus
B. Nail psoriasis
C. * Onychomycosis, atrophic variant
D. Eczema
E. Congenital keratoderma
137. Disease from the photo (Fig. 52) can be:
A. Normotrophic
B. Hypertrophic
C. Atrophic
D. All are wrong
E. * All are correct
138. How many types of the disease from photo (Fig. 52) you know?
A. 5
B. 4
C. 6
D. * 3
E. 2
139. The disease from photo (Fig. 52) is:
A. * Fungal infection
B. Viral infection
C. Parasitogenic infection
D. Sexually transmitted infection
E. Bacterial infection
140. Based on the photo (Fig. 53) chooses diagnosis:
A. Osteal follicululitis
B. * Infiltrativepurulent trichophytosis of the scalp
C. Limited neurodermatitis
D. Carbuncle
E. Favus
141. The disease from photo (Fig. 53) is:
A. Viral infection
B. Parasitogenic infection
C. * Fungal infection
D. Sexually transmitted infection
E. Bacterial infection
142. What symptom is characterized for this disease from photo (Fig. 53)?
A. “Mouth eaten”
B. All are wrong
C. “Woronow ring”
D. “Koebner phenomenon”
E. * “Bolter symptom”
143. Based on the photo (Fig. 54) chooses diagnosis:
A. Nail psoriasis
B. Nail lichen planus
C. * Candidal paronychia
D. Congenital keratoderma
E. Eczema
144. The disease from photo (Fig. 54) is:
A. * Fungal infection
B. Viral infection
C. Parasitogenic infection
D. Sexually transmitted infection
E. Bacterial infection
145. Causative agent of the disease on photo (Fig. 54) is:
A. Pubic lice
B. * Candida albicans
C. Treponema pallidum
D. Scabies mite
E. Herpes virus infection
146. Based on the photo (Fig. 55) chooses diagnosis:
A. Psoriasis
B. * Zoophilic microsporia
C. Contact dermatitis
D. Bullous impetigo
E. Lichen planus
147. This “Stamp symptom” from photo (Fig. 55) is characterized for:
A. Trichophytia
B. Dermatophytia
C. Rubrophitia
D. Syphilis
E. * Zoophilic microsporia
148. What type of ointment you must prescribe for this patient with disease from photo (Fig.
55)?
A. Antibiotic
B. Viral
C. * Antifungal
D. Corticosteroid
E. With salicylic acid
149. The disease from photo (Fig. 56) is:
A. * Fungal infection
B. Viral infection
C. Parasitogenic infection
D. Sexually transmitted infection
E. Bacterial infection
150. What type of ointment you must prescribe for this patient with disease from photo (Fig.
56)?
A. Antibiotic
B. Viral
C. * Antifungal
D. Corticosteroid
E. With salicylic acid
151. Based on the photo (Fig. 57) chooses diagnosis:
A. Psoriasis
B. * Tinea capitis
C. Primary Syphilis
D. Bullous impetigo
E. Lichen planus
152. Based on the photo (Fig. 57) chooses diagnosis:
A. Nummular psoriasis
B. * Superficial tinea capitis
C. Favus
D. Impetigo vulgaris
E. Pemphigus erythematous
153. Based on the photo (Fig. 57) chooses diagnosis:
A. Osteal follicululitis
B. * Tinea capitis
C. Limited neurodermatitis
D. Carbuncle
E. Favus
154. Based on the photo (Fig. 58) chooses diagnosis:
A. Psoriasis
B. * Tinea
C. Impetigo bullous
D. Pemphigus erythematous
E. Wart
155. The disease from photo (Fig. 58) is:
A. * Fungal infection
B. Viral infection
C. Parasitogenic infection
D. Sexually transmitted infection
E. Bacterial infection
156. Based on the photo (Fig. 59) chooses diagnosis:
A. Nail lichen planus
B. * Onychomycosis, hypertrophic variant
C. Nail psoriasis
D. Eczema
E. Congenital keratoderma
157. The disease from photo (Fig. 59) is:
A. * Fungal infection
B. Viral infection
C. Parasitogenic infection
D. Sexually transmitted infection
E. Bacterial infection
158. Based on the photo (Fig. 60) chooses diagnosis:
A. Nail lichen planus
B. Nail psoriasis
C. Eczema
D. Congenital keratoderma
E. * Onychomycosis, atrophic variant
159. The disease from photo (Fig. 60) is:
A. * Fungal infection
B. Viral infection
C. Parasitogenic infection
D. Sexually transmitted infection
E. Bacterial infection
160. How many types of the disease from photo (Fig. 60) you know?
A. 5
B. 4
C. 6
D. * 3
E. 2
161. Based on the photo (Fig. 61) chooses diagnosis:
A. Geographical psoriasis
B. Scabies
C. * Pituriasis versicolor
D. Rubrophytosis corporis
E. Dermatitis
162. The disease from photo (Fig. 61) is:
A. * Fungal infection
B. Viral infection
C. Parasitogenic infection
D. Sexually transmitted infection
E. Bacterial infection
163. The main complaints of patients with the disease from the photo (Fig. 61) include:
A. Severe pain
B. * Asymptomatic
C. Hard itch
D. Enlargement of lymphatic nodes
E. Fever
164. Based on the photo (Fig. 62) chooses diagnosis:
A. Nummular dermatitis
B. Eczema
C. Erythrasma
D. * Tinea cruris
E. Exudative psoriasis
165. The disease from photo (Fig. 62) is:
A. * Fungal infection
B. Viral infection
C. Parasitogenic infection
D. Sexually transmitted infection
E. Bacterial infection
166. What ointment you must prescribe for the patient with disease from photo (Fig. 62)?
A. Acyclovir
B. Kremgen
C. Mifungar
D. * Econazole
E. Latycort
167. What orally drug must take patient with disease from photo (Fig. 63)?
A. * Orungal
B. Dexamethazone
C. Valtrex
D. Phencarol
E. Azitromycine
168. Based on the photo (Fig. 64) chooses diagnosis:
A. Osteal follicululitis
B. Sycosis vulgaris
C. Dermatitis
D. * Infiltrativepurulent trichophytosis of the beard zone
E. Tuberculosis cutis colliquvativa
169. The disease from photo (Fig. 64) is:
A. Viral infection
B. * Fungal infection
C. Parasitogenic infection
D. Sexually transmitted infection
E. Bacterial infection
170. Based on the photo (Fig. 65) chooses diagnosis:
A. Rubrophytosis corporis
B. * Zoophilic micrisporia
C. Contact dermatitis
D. Bullous impetigo
E. Lichen planus
171. What symptom is characterized for this disease from photo (Fig. 65)?
A. “Mouth eaten”
B. * “Stamp symptom”
C. “Woronow ring”
D. “Koebner phenomenon”
E. “Bolter symptom”
172. The disease from photo (Fig. 65) is:
A. Viral infection
B. * Fungal infection
C. Parasitogenic infection
D. Sexually transmitted infection
E. Bacterial infection
173. Based on the photo (Fig. 66) chooses diagnosis:
A. * Pituriasis versicolor
B. Contact dermatitis
C. Syphilis
D. Impetigo vulgaris
E. Lichen planus
174. The disease from photo (Fig. 66) is:
A. Viral infection
B. * Fungal infection
C. Parasitogenic infection
D. Sexually transmitted infection
E. Bacterial infection
175. The clinical picture of the disease from photo (Fig. 66) is:
A. Well demarcated, indurate plaque
B. Flaccid bulla
C. * Macula
D. Nodule
E. Ulcer
176. The disease from photo (Fig. 66) is:
A. Viral infection
B. * Fungal infection
C. Parasitogenic infection
D. Sexually transmitted infection
E. Bacterial infection
177. Based on the photo (Fig. 67) chooses diagnosis:
A. Pemphigus vulgaris
B. * Candidiasis of the groin
C. Tinea of the groin
D. Erythrasma
E. Dermatitis
178. The disease from photo (Fig. 67) is:
A. Parasitogenic infection
B. Viral infection
C. * Fungal infection
D. Sexually transmitted infection
E. Bacterial infection
179. Based on the photo (Fig. 68) chooses diagnosis:
A. * Tinea capitis
B. Osteal follicululitis
C. Limited neurodermitis
D. Carbuncle
E. Pemphigus erythematous
180. The disease from photo (Fig. 68) is:
A. Parasitogenic infection
B. Viral infection
C. * Fungal infection
D. Sexually transmitted infection
E. Bacterial infection
181. Based on the photo (Fig. 69) chooses diagnosis:
A. Psoriasis of the scalp
B. Contact dermatitis
C. Eczema
D. * Tinea capitis
E. Seborrheal dermatitis
182. The disease from photo (Fig. 69) is:
A. * Fungal infection
B. Viral infection
C. Parasitogenic infection
D. Sexually transmitted infection
E. Bacterial infection
183. Based on the photo (Fig. 70) chooses diagnosis:
A. Toxicodermia
B. Scabies
C. * Pituriasis versicolor
D. Rubrophytosis corporis
E. Dermatitis
184. The disease from photo (Fig. 70) is:
A. * Fungal infection
B. Viral infection
C. Parasitogenic infection
D. Sexually transmitted infection
E. Bacterial infection
185. Skin lesions of the disease from photo (Fig. 70) are:
A. Nodules
B. Fissures
C. * Macula's
D. Erosions
E. Excoriations
186. The main complaints of patients with the disease from the photo (Fig. 70) include:
A. Severe pain
B. * Asymptomatic
C. Hard itch
D. Enlargement of lymphatic nodes
E. Fever
187. Based on the photo (Fig. 71) chooses diagnosis:
A. Pustular psoriasis
B. * Rubrophytia of the face
C. Atopic dermatitis
D. Impetigo vulgaris
E. Contact dermatitis
188. The disease from photo (Fig. 71) is:
A. Viral infection
B. Parasitogenic infection
C. * Fungal infection
D. Sexually transmitted infection
E. Bacterial infection
189. What ointment you must prescribe for the patient with disease from photo (Fig. 73)?
A. * Lamycon
B. Acyclovir
C. Carboderm
D. Zovirax
E. Spregal
190. What ointment you must prescribe for the patient with disease from photo (Fig. 74)?
A. Orungal
B. Dexamethazone
C. Valtrex
D. * Nizoral
E. Azitromycine
191. Based on the photo (Fig. 75) chooses diagnosis:
A. Exudative psoriasis
B. * Tinea cruris
C. Eczema
D. Erythrasma
E. Contact dermatitis
192. The disease from photo (Fig. 75) is:
A. Viral infection
B. Parasitogenic infection
C. * Fungal infection
D. Sexually transmitted infection
E. Bacterial infection
193. Based on the photo (Fig. 76) chooses diagnosis:
A. Contact dermatitis
B. * Pituriasis versicolor
C. Atopic dermatitis
D. Toxicodermia
E. Syphilis
194. The disease from photo (Fig. 76) is:
A. Viral infection
B. Parasitogenic infection
C. * Fungal infection
D. Sexually transmitted infection
E. Bacterial infection
195. What is the disease on the photo (Fig. 76)?
A. Syphilis
B. Contact dermatitis
C. * Pituriasis versicolor
D. Atopic dermatitis
E. Toxicodermia
196. Treatment of the disease from the photo (Fig. 76) includes:
A. * Itraconazole
B. H1 antihistamines
C. Leukotriene blockers
D. H2 antihistamines
E. Steroids
197. The main complaints of patients with the disease from the photo (Fig. 76) include:
A. Severe pain
B. * Asymptomatic
C. Hard itch
D. Enlargement of lymphatic nodes
E. Fever
198. The sites of choice of the disease from the photo (Fig. 76) are:
A. Upper part of the trunk
B. Upper parts of the arms
C. Neck
D. Forehead and cheeks
E. * All are correct
199. The disease from photo (Fig. 76) is:
A. Viral infection
B. Parasitogenic infection
C. * Fungal infection
D. Sexually transmitted infection
E. Bacterial infection